Sunteți pe pagina 1din 87

FONDATĂ în anul 1996

ANUL XIII nr. 5 – 6 / Iulie - Decembrie 2008


1. Judeţul BISTRIŢA-NĂSĂUD: Nicolae Sanda, Maria Sas, Vasile Negruşeri, Oprea
Angelica-Lucia,
2. Judeţul BRAŞOV: Mihaly Bencze
3. Judeţul BRĂILA: Gheorghe Alexe, Nazeli Boicescu, Daniela Covaci, Victoria Negulescu,
LucicaVisalon, Marius Damian, Valentin Damian, Mihaela Giurcă, Roxana Murea,
Georgeta Roşca, Viorica Păun, Nicolae Stănică, Daniela Tilincă, Neculai Carnaru, Narcis
Turcu, RaduVasile, Carmen Folea, Mădălina Teodorescu, Iugulescu Nicolae
4. BUCUREŞTI: Dumitru Săvulescu, Cristina Ichim
5. Judeţul BUZĂU:Costică Negoiţă
6. Judeţul CLUJ: Dorin Andrica, Dorel I. Duca, Vasile Şerdean
7. Judeţul CONSTANŢA: Ion Cucurezeanu, Gheoghe Andrei, Andreea Păunescu
8. Judeţul DOLJ: Lucian Tuţescu, Nicolae Ivăşchescu, Ioan Purcaru, Eliza Chile, Ani
Drăghici, Nicoleta Bran, Silvia Olteanu, Ionuţ Ivănescu, Teodora Papoe, Anca Tuţescu,
Roxana Vasile, Ion Casiu, Cristian Casiu, Liviu Smarandache, Monica Matei, Sabău
Niculina, Ion Vişan, Margareta Alexie, Mariana Mărculescu, Ana Cismaru, Dumitru Cotoi,
Ileana Didu, Sorin Pîrlea, Denisa Florică, Doina Chiriac,Ileana Mândruleanu, Roxana
Vasile, Luminiţa Vasile, Marilena Andreescu, Mircea Tereujeanu, Cătălin Cristea, Florentin
Nicolae, Gabriel Tica, Monica Matei, Marius Zgaiba, Ioan Purcaru, Butaru Zizi-Iuliana
9. Judeţul CARAŞ - SEVERIN: Adriana Dragomir, Lucian Dragomir
10. Judeţul GALAŢI : Marin Dolteanu, Constantin Ursu, Romeo Zamfir, Iuliana Duma,Vasile
Popa, Mihai Totolici, Radu Tătaru, Rodica Bălan, Dumitru Bălan
11. Judeţul HUNEDOARA: Ioan Jurchela, Dan Ştefan Marinescu, Ioan Şerdean, Daniel
Nicoară, Gheorghe Stoica
12. Judeţul MARAMUREŞ : Vasile Berinde
13. Judeţul OLT : Marius Perianu
14. Judeţul PRAHOVA : Mircea Ganga, Ion Nedelcu,
15. Judeţul SATU – MARE: Traian Tămâian
16. Judeţul SIBIU: Dumitru Acu, Simona Florea, Mugur Acu, Nicolae Secelean,Ana-Maria Acu,
Alexandru Lupa, Petrică Dicu, Dumitru Barac, Emil C. Popa, Alin Pop,

ISSN 1584-983X
CU LACRIMI ÎN OCHI…

Profesorul MIRCEA GANGA, format la „ Şcoala de Matematică ” a Prof.univ.dr. ION


CUCULESCU, din cadrul Facultăţii de Matematică Bucureşti, pe care a absolvit-o în 1976, s-a impus în
publicistica matematică românească prin lucrări dense, scrise special pentru elevii de liceu, într-un limbaj
accesibil acestora. Autor de probleme propuse şi articole în „Gazeta Matematică”, s-a preocupat intens de a
regândi si reformula într-un stil inconfundabil unele probleme de bază cu care se confruntă învăţământul
matematic românesc, dovadă vie şi grăitoare fiind colecţia de manuale şi culegeri de matematică, fiecare cu
un aport matematic distinct.
Pentru majoritatea liceelor din ţară, literatura matematică oferită de domnul profesor Mircea Ganga
a reprezentat un real sprijin în pregătirea elevilor atât pentru studiul la clasă, cât şi pentru pregătirea la un
nivel avansat, pentru olimpiade şi concursuri.
Pentru Brăila, anul 2005 a oferit deosebita onoare de a ne bucura de prezenţa domnului profesor ca
preşedinte al Concursului Interjudeţean „Petru Moroşan - Trident”, concurs care acum se află la cea de-a
şasea ediţie.
Articolul ar putea continua astfel ca şi altele scrise în memoria cuiva. Însă, nu dorim acest lucru. În
septembrie 2008, bucuria finalizării noului manual şi a culegerii de clasa a IX-a ne-a fost împărtăşită de
Mircea. Ne-am bucurat cu toţii de o reuşită de excepţie şi ne pregăteam pentru un nou număr al revistei şi
pentru concurs. Totul era ca de fiecare dată făcut cu răbdare, cu migală şi mult respect pentru matematică.
Deja onoarea şi valoarea erau de undeva, din negura timpului, linii ce defineau activitatea noastră. Am
înţeles de la Mircea că munca realizată la nivel profesionist aduce întotdeauna roade. Uneori uita că mai
trebuie şi odihnă şi alte lucruri ce fac viaţa mai frumoasă. Adevărul este că, de cele mai multe ori, când ai
un ideal, uiţi de toate şi mergi până la capăt. Unii râd de tine, alţii te blamează, dar totdeauna strugurii de
sus sunt acri fiindcă este greu să-i culegi. A lăuda nişte manuale, cărţi, culegeri şi alte scrieri realizate de
Mircea, o poate face orice persoană avizată din domeniul matematic, dar cel mai important este altceva:
faptul că totul este realizat din respect şi dragoste pentru matematică, pentru elevi şi pentru cei care doresc
să-i introducă pe aceştia pe drumul sinuos al matematicii.
Începusem să ne pregătim pentru concurs, cu aceeaşi dorinţă ca în fiecare an, cu respect pentru
elevii şi profesorii din ţară nerăbdători să revină la Brăila, dar şi cu speranţa de a realiza un concurs mai
bun, mai performant decât în anii precedenţi alături de colegii noştri. Ştiam că munca depusă la realizarea
ultimelor manuale îşi lăsase o amprentă asupra sănătăţii sale. Într-o dimineaţă, pe 17 octombrie 2008, sună
un telefon. A murit Mircea. Şi atât. Era mai mult decât de ajuns. A doua zi, l-am văzut pentru ultima dată.
Se odihnea după o muncă deosebită, din care noi putem trage învăţăminte şi pe care o folosim în fiecare zi.
„Aveţi ca temă din Ganga de la pagina…” este ceva din viaţa noastră de zi cu zi. Noi, cei care i-am fost
aproape, nu putem uita nimic. Defapt, trecerea timpului nu poate şterge nimic din suflet când ţii cu adevărat
la cineva.
„ Dacă se întâmplă ceva cu mine, roagă-i să continue mai departe…”, a fost una din ultimele
fraze înainte de….
Este primul articol al revistei, deşi a fost scris ultimul. Când am început revista, bucuria unui nou
început era firească. Acum, va apărea un nou număr al revistei, cu lacrimi în ochi. Vom merge mai
departe şi vom urma exemplul său moral şi profesional . Sperăm să reuşim. . . Mircea va
rămâne mereu preşedintele concursului nostru, iar timpul va demonstra cu adevărat ceea ce a însemnat el,
pentru noi şi pentru matematica românească în general. Îl vom aştepta mereu, să revină printre noi, fiindcă
în fiecare zi îl atingem, deschizând o nouă pagină.

„Ora a luat sfârşit, vă invit să ieşiţi în pauză.”

Redacţia Revistei.
FRACŢII CU SIMPLIFICĂRI FRAUDULOASE

de Roxandra Murea

“Obiectul matematicii este atât de serios, încât este util să nu pierdem ocazia pentru a-l face
distractiv .”
Blaise Pascal

(13
26 26 2
1. Dacă s-ar cere să simplificaţi fracţia aţi proceda astfel : = .
65 65 5
Se observă că, în cazul acestei fracţii, simplificarea revine la o simplă eliminare a cifrei 6, aflată
atât la numărător, cât şi la numitor.
În mod natural se poate pune întrebarea: care sunt fracţiile la care putem scrie
ax a
= , a, b, x ∈ {1, 2, 3, …, 9} ? (1)
xb b
Evident că vom evita cazul banal a = b = x. Din (1) avem ( 10 a + x ) b = ( 10 x + b ) a ,
9ab
de unde x= . Luând pe rând a egal cu 1, 2, …,9, găsim următoarele soluţii pentru (1) :
10a − b
1(6) 1 2(6) 2 1(9) 1 4(9) 4
= , = , = şi = . (2)
(6) 4 4 (6)5 5 (9)5 5 (9)8 8
Între paranteze s-a scris cifra (numărul) cu care s-a făcut simplificarea, convenţie ce se va păstra şi
în continuare.
Iată că, în fracţiile de la (2), deşi simplificarea se face eronat, nerespectând regulile obişnuite de
simplificare, totuşi rezultatele sunt corecte. Dacă acelaşi procedeu s-ar aplica în cazul simplificării altor
fracţii, atunci rezultatele ar fi greşite. Prin urmare, este de reţinut faptul că un raţionament greşit poate
conduce la rezultate bune. De aceea, în rezolvarea problemelor de matematică, trebuie să urmărim
corectitudinea nu numai pentru rezultatele finale, ci şi pentru raţionamentele prin care au fost obţinute
aceste rezultate.
ax
2. O problemă analoagă cu (1) este cea a determinării fracţiilor la care să putem scrie:
xb
xa a
= , a ≠ b, a, b, x ∈ { 1, 2, … ,9} . (3)
bx b
9ab
Din (3) rezultă x= ,
10b − a
Ceea ce ne arată că soluţiile pentru (3) se obţin din (2) prin schimbarea lui a cu b. Astfel se obţin
fracţiile cu simplificările frauduloase :
(6)4 4 (9)5 5 (6)5 5 (9)8 8
= , = , = , = . (4)
1(6) 1 1(9) 1 2(6) 2 4(9) 4

3. În mod firesc apare problema studierii simplificărilor de forma


ax a xa a
= sau = , cu a, b, x ∈ {1, 2, …, 9}, a ≠ b.
bx b xb b
Prin simple calcule se poate deduce că astfel de simplificări nu sunt posibile.
4. Acum problemele studiate mai sus pot fi privite într-un cadru puţin mai general. Şi anume, în ce condiţii
se poate face simplificarea ca un factor comun la o cifră de la numitor şi la o cifră de la numărător.
Astfel, dacă a = a1 x , d = d1 x , (a1 , d1 ) = 1 (s-a notat prin (u, v) cel mai mare divizor comun
pentru numerele naturale u şi v), atunci putem scrie
ab a1b
= , a, b, c, d ∈ {1, 2, ..., 9}. (5)
cd cd1
Din (4) avem (10a + b)(10c + d1 ) = (10a1 + b)(10c + d ), de unde găsim
( x − 1)(10 ca1 − bd1 ) = 0.
De aici, ţinând seama că 10 ca1 - bd1 ≠ 0 , rezultă x = 1 , ceea ce ne arată că (4) este posibilă
numai dacă a = d , adică suntem în cazul de la 2). În mod analog se arată că o simplificare de tipul
ab ab1
= , b = b1 x , c = c1 x , (b1 , c1 ) = 1 , a, b, c, d ∈ {1, 2, …, 9} (6)
cd c1d
este posibilă numai dacă b = c, adică în cazul de la 1).
Acum considerăm probleme analoage cu (5) şi (6) numai ca simplificarea să se facă pe verticală.
ab
Astfel, ne punem problema precizării condiţiilor în care fracţia poate fi simplificată cu x, unde
cd
a = a1 x , c = c1 x , (a1 , c1 ) = 1 , a ≠ c , adică să putem scrie egalitatea
ab a1b
= , a, b, c, d ∈ {1, 2, 3, ...,9}. (7)
cd c1d
a b1
Din (7) rezultă ( x − 1)( ad1 − cb1 ) = 0 , de unde, ţinând seama că x ≠ 1, găsim = , adică
c d1
a b
= . (8)
c d
La aceeaşi condiţie (8) se ajunge şi dacă s-ar face o simplificare de tipul
ab ab1
= , b = b1 x , d = d1 x , (b1 , d1 ) = 1 , b ≠ d. (9)
cd cd1
Iată câteva exemple de astfel de fracţii şi simplificările lor frauduloase:
1(2) 11 1 (3)2 1(2) 11 1
= = , = = = ,
3(6) 33 3 (9)6 3(6) 33 3
(2)3 1(3) 11 1 (6) 4 2(4) 22 2
= = = , = = = ,
(6)9 3(9) 33 3 (9)6 3(6) 33 3
(4)6 2(6) 22 2 (6)3 33 3
= = = , = = .
(6)9 3(9) 33 3 (8)4 44 4

5. Următorul salt în lumea fracţiilor care admit simplificări frauduloase ar fi mărirea numărului de cifre de
la numărător şi de la numitor.
În [1] profesorul Solomon Marcus ne propune pentru contemplare următorul exemplu în care se pot
face trei simplificări frauduloase succesive cu 6 :
2666 266 26 2
= = = .
6665 665 65 5
În plus, se sugerează cercetarea problemei generale care se ascunde aici.
Această problemă s-ar putea formula astfel: să se determine numerele axn xn − 1...x1 , xn xn − 1 ...x1b
scrise în baza 10, a ≠ b , astfel încât să putem scrie
axn xn− 1...x1 axn− 1 xn− 2 ...x1 ax2 x1 ax1 a
= = ... = = = . (10)
xn xn− 1...x1b xn − 1 xn− 2 ...x1b x2 x1b x1b b
ax1 a
Pentru n = 1 avem = , adică suntem în situaţia de la 1) cu soluţiile date de (2).
x1b b
Considerăm cazula = 1, b = 4, x1 = 6 . Atunci, din ax2 x1 / x2 x1b = a / b avem
4(100 + 10 x2 + 6) = = (100 x2 + 10 + 4) , de unde x2 = 6 . Acum din ax3 x2 x1 / x3 x2 x1b = a / b
rezultă x3 = 6 ş.a.m.d. În final găsim x1 = x2 … = xn = 6 (o demonstraţie riguroasă se face prin inducţie
matematică).
Astfel am obţinut :
1666 ...6 666
    ...64 1666...6 666...64
/     =     /     = … = 166 / 664 = 16 / 64 = 1 / 4. (11)
n ori n ori ( n − 1) ori ( n − 1) ori

1666
   ...6 66...64 = 1
O demonstraţie directă a faptului că / 123 are loc pentru orice n natural , n ≥ 1,
n ori
n ori
4
se poate face astfel:

  ...64 / = 10n (+ 6 ·111…1) / (6 ·111…1·10 + 4) 10n = ( + 6 · 10 − 1 ·10 + 4) =


n
1666
   ... 6 666
n ori n ori
9
3 ⋅ 10 + 2 ⋅ 10 − 2 4(5 ⋅ 10 − 2)
n n n
1
= / = (5· 10n -2) / 4 (5· 10n -2) = .
3 3 4
În mod analog, pornind de la celelalte fracţii obţinute la (2), se găsesc următoarele egalităţi :
2666
   ...6 666 ...65 266...6 66...65 26 2
/     =    /    = ...= 266/665 = = , (12)
n ori n ori ( n − 1) ori ( n − 1) ori 65 5
1999
   ...9 999 ...95 1999...9 999...95 199 19 1
/     =     /     =...= = = , (13)
n ori n ori ( n − 1) ori ( n − 1) ori 995 95 5
4999
    ...9 999 ...98 499...9 999...98 49 1
/     =    /     = ... = = . (14)
n ori n ori ( n − 1) ori ( n − 1) ori 98 2
De la 2) rezultă că egalităţile (11)-(14) rămân valabile şi dacă se inversează fracţiile.

Profesor,
Liceul Teoretic „ Panait Cerna ”, Brăila

BIBLIOGRAFIE

1. Solomon Marcus, Şocul matematicii, Editura Albatros, 1987.


2. H. R. Radian, T. J. Radian, Recreaţii matematice, Editura Albatros, Lyceum, 1973
3. Revista Astra Mate

INEGALITĂŢI ÎNTRE ELEMENTELE UNUI TRIUNGHI


Vom prezenta un mod de abordare la clasa a VI - a a capitolului „ INEGALITĂŢI ÎNTRE
ELEMENTELE UNUI TRIUNGHI ” sub aspect teoretic urmat de câteva aplicaţii.
Considerăm ∆ ABC şi notăm cu a, b, c lungimile laturilor BC , AC respectiv AB .

TEOREMĂ

Într-un triunghi cu două laturi necongruente, laturii cu lungimea mai mare i se opune unghiul mai
mare.
Ipoteză : ∆ ABC , BC > AB
Concluzie : m ( S BAC ) > m ( S ACB )
Demonstraţie :
Fie D ∈ ( BC ) ( BD ) ≡ ( AB ) . Rezultă
astfel încât

S ADB ≡ S BAD , dar m ( S ADB ) > m ( S ACB ) (unghi exterior


∆ ADC ) rezultă că m ( S BAD ) > m ( S ACB ) , dar
m ( S BAC ) > m ( S BAD ) ⇒ m ( S BAC ) > m ( S ACB )

Aplicaţie

Să se ordoneze descrescător unghiurile triunghiului din fig.1.

TEOREMA RECIPROCĂ

Într-un triunghi cu două unghiuri necongruente, unghiului mai mare i se opune latura mai mare
( demonstraţie prin reducere la absurd)
Ipoteză :∆ ABC , m ( S BAC ) > m ( S ACB )
Concluzie : BC > AB
Demonstraţie :
Presupunem că BC ≤ AB , rezultă, conform teoremei directe că m ( S BAC ) ≤ m ( S ACB ) , ceea ce
contrazice ipoteza. Deci BC > AB .
Aplicaţie

Să se ordoneze descrescător laturile triunghiului ABC din figura 2.


Consecinţe :

1) În orice
triunghi dreptunghic ipotenuza este mai mare decât fiecare catetă.
2) Dintr-un punct exterior unei drepte , se pot construi o singură perpendiculară şi mai multe oblice .
Lungimea perpendicularei este mai mică decât lungimea fiecărei oblice. Dintre două
oblice mai mare este cea care are piciorul mai depărtat de piciorul perpendicularei.
3) Notăm cu d o dreaptă din plan şi fie A un punct A ∉ d . Ducem AM ⊥ d şi
AB , AC două oblice faţă de AM Conform consecinţei 2), avem că
AM < AB < AC .

TEOREMĂ

Suma lungimilor a două laturi ale unui triunghi este mai mare decât lungimea celei de a treia laturi.

Ipoteză : ∆ ABC de laturi ( AB ) , ( BC ) , ( AC )


Concluzie : AB + BC > AC
Demonstraţie : Fie B ∈ ( AM ) astfel încât CB = BM . Rezultă că S BCM ≡ S BMC , dar

m ( S ACM ) > m ( S BCM ) , de unde obţinem că m ( S ACM ) > m ( S AMC ) atunci conform
teoremei reciproce, AM > AC , deci AB + BM > AC , adică AB + BC > AC .
Observaţie: Acest rezultat poate fi transpus pentru fiecare latură a triunghiului şi avem următoarele
inegalităţi între laturile unui triunghi ABC :

AB − BC < AC < AB + BC a− c < b < a + c


AC − BC < AB < AC + BC sau a− b < c < a + b
AB − AC < BC < AB + AC b− c < a < b + c

Aplicaţii

1. Să se stabilească, în fiecare caz, dacă cu segmentele de lungimi a, b, c se poate construi un triunghi.


1) a = 7 cm, b = 18 cm, c = 13 cm

2) a = 2 cm, b = 6 cm, c = 10 cm

3) a = 10 cm, b = 15 cm, c = 25 cm.

2. Demonstraţi că într-un triunghi ABC fiecare latură este mai mică decât semiperimetrul său.

Soluţie:
a + b+ c
a < b + c ⇒ 2a < a + b + c ⇒ a < ⇒ a < p.
2
AB + AC
3. Demonstraţi că într-un triunghi AB , mediana AM < . Soluţie:
2
Fie AM = MD , atunci ∆ AMB ≡ ∆ CMD ( L.U .L.) , de unde rezultă
că AB = CD , AM = MD .
AB + AC
În triunghiul ACD avem AD< AC+CD sau AM < .
2
4. Fie A, B două puncte diferite care nu aparţin dreptei d, situate în acelaşi semiplan. Să se determine
astfel încât suma AM + MB să fie minimă.
Object 209

Soluţie:
Fie B ' simetricul lui B faţă de dreapta d, rezultă că
MB = MB ' . În triunghiul AMB ' avem AM + MB '> AB ' , deci
AM + MB > AB ' . Fie P∈ AB '∩ d atunci AM + MB este minimă
minimul dacă P = M .

5. Dacă un triunghi are două bisectoare congruente atunci triunghiul este isoscel.

Soluţie:

În triunghiul ABC , ( BE ) şi
(CF ) sunt cele două bisectoare congruente.
Presupunem că m ( S ABC ) > m ( S ACB ) . Construim BO P AC şi CO P BE ,
∆ BEC ≡ ∆ CBO ( L.U .L.) rezultă că BE = OC , deci CF = OC , iar m ( S EBC ) =
= m ( S BCO ) deci m ( S FCB ) ≤ m ( S EBC ) ⇒ m ( S FCB ) ≤ m ( S BCO ) rezultă

BF ≤ OB şi prin urmare în ∆ BFO avem m ( S BFO ) ≥ m ( S FOB ) . Deoarece


m ( S OFC ) = = m ( S FOC ) , ( CF = CO ) rezultă că m ( S BFC ) ≥ m ( S BOC ) = m ( S BEC ) .
Considerând aceste unghiuri în V BFM obţinem m ( S FBM ) ≤ m ( S MCE ) şi atunci obţinem că
m ( S ABC ) ≤ m ( S ACB ) , dar m ( S ABC ) > m ( S ACB ) , deci m ( S ABC ) = m ( S ACB ) adică
triunghiul ABC este isoscel
6. Problema lui Pompeiu

Fie P un punct în interiorul triunghiului echilateral ABC . Să se demonstreze că


( PA) , ( PB ) , ( PC ) pot forma un triunghi.( varianta restrânsă)
Soluţie:
AP ∩ BC = { Q} . Unul din unghiurile S AQB sau S AQC va fi obtuz sau drept.
Presupunem că m ( S AQC ) > 90° . Atunci m ( S AQC ) > m ( S ACQ ) , de unde rezultă, în
∆ AQC că AP < AQ < AC = BC < PB + PC deci ( PA), ( PB ), ( PC ) pot forma un triunghi.

Profesor : Neculai Carnaru,


Şcoala “Ecaterina Teodoroiu” Brăila
Profesor : Iugulescu Nicolae,
Şcoala “Mihai Viteazul ” Brăila
Profesor : Turcu Narcis Gabriel,
Liceul Teoretic “Nicolae Iorga” Brăila

BIBLIOGRAFIE

1. Programa şcolară de matematică, O.M.E.N. nr. 4237/23.08.1999, nr. 3458 din 9.03.2004.
2. Probleme de geometrie elementară M. Pimsner şi S. Popa E.D.P. Bucureşti 1979.
3. Inegalităţi elementare ... şi mai puţin elementare de Mircea Becheanu şi Bogdan Enescu,
Editura GIL 2002.

Inegalitatea lui I.E.BURKELL

de Alexe Gheorghe

Fie f : [ a, b ] → ¡ , f convexă atunci ( ∀ ) x, y, z ∈ [ a, b ] şi ( ∀ ) p, q, r ∈ ¡ + avem:

pf ( x ) + qf ( y ) + rf ( z )
 px + qy + rz  p+ q  px + qy  q+ r  qy + rz  p+ r  px + rz 
+ f ≥ f + f + f 
p+ q+ r  p+ q+ r  p+ q+ r  p+ q  p+ q+ r  q+ r  p+ q+ r  p+ r 
Pentru p = q = r ∈ ¡ + avem inegalitatea lui Tiberiu Popoviciu şi anume
1) Dacă f convexă [ a, b ] atunci

 x+ y+ z    x+ y  y+ z  x+ z
f ( x) + f ( y) + f ( z ) + 3 f   ≥ 2 f  + f + f   , ∀ x, y , z ∈ [ a, b ] .
 3    2   2   2 
2) Dacă f concavă [ a, b ] atunci

 x+ y+ z    x+ y  y+ z  x+ z
f ( x) + f ( y) + f ( z ) + 3 f   ≤ 2 f  + f + f   , ∀ x, y , z ∈ [ a, b ] .
 3    2   2   2 
Aplicaţii:

C 3237 G.M.11/2007

Fie V ABC ascuţitunghic, atunci avem :


A B C 5 r
sin + sin + sin ≥ +
2 2 2 4 2R
T. Lupu , Constanţa
Soluţie:

Fie
 π   π 
f :  0,  → ¡ , f ( x ) = cos x, f ' ( x ) = sin x, f '' ( x ) = − cos x < 0, ∀ x ∈  0,  ⇒ f concavă
 2  2
 π 
pe  0,  .
 2
 x+ y+ z   x+ y  y+ z  x+ z  π 
Atunci f ( x ) + f ( y ) + f ( z ) + 3 f   ≤ 2 f  + f + f   , ∀ x, y, z ∈  0,  .
 3    2   2   2   2
 π   π   π 
Fie x = A ∈  0,  , y = B ∈  0,  , z = C ∈  0,  ⇒
 2  2  2
 A+ B + C 
cos A + cos B + cos C + 3cos  ≤
 3 
 A + B B + C C + A  3  A B C
≤ 2  cos + cos + cos ⇒ cos A + cos B + cos C + ≤ 2  sin + sin + sin  . Dar
 2 2 2  2  2 2 2
r  A B C r 3 A B C 5 r
cos A + cos B + cos C = 1 + ⇒ 2  sin + sin + sin  ≥ 1 + + ⇒ sin + sin + sin ≥ + .
R  2 2 2 R 2 2 2 2 4 2R

Supliment R.M.T. 29 aprilie-4mai 2008

Arătaţi că în orice triunghi ascuţitunghic ABC este adevărată inegalitatea:


a b c
+ + ≥ 2( a + b + c) .
cos A cos B cos C
Mircea Becheanu, Bucureşti
Soluţie:

 π 
Fie A, B, C ∈  0,  . Atunci tgA, tgB, tgC ∈ ( 0, ∞ ) . Demonstrăm că :
 2
a b c
+ + = 2 R ( tgA + tgB + tgC ) ≥ 4 p
cos A cos B cos C
 π 
Fie f ( x ) = tgx, f :  0,  → ( 0, ∞ ) ⇒
 2
1 sin 2 x  π 
f '( x) = 2
, f '' ( x ) = 4
> 0, ∀ x ∈  0,  ⇒
cos x cos x  2
 π 
⇒ f convexă pe  0,  . Atunci :
 2
 x+ y+ z    x+ y  y+ z  z+ x  π 
f ( x) + f ( y) + f ( z ) + 3 f   ≥ 2 f  + f + f   , ∀ x, y , z ∈  0, 
 3    2   2   2   2
.
 π   π   π   A B C
Fie x = A ∈  0,  , y = B ∈  0,  , z = C ∈  0,  ⇒ tgA + tgB + tgC + 3 3 ≥ 2  ctg + ctg + ctg  ,
 2  2  2  2 2 2
A B C p
dar ctg + ctg + ctg = .
2 2 2 r
 A B C
Rezultă că tgA + tgB + tgC + 3 3 ≥ 2  ctg + ctg + ctg  , dar
 2 2 2
A B C p
ctg + ctg + ctg = ⇒
2 2 2 r
2p a b c  2p 
⇒ tgA + tgB + tgC ≥ − 3 3 ⋅ 2R ⇒ + + = 2 R ( tgA + tgB + tgC ) ≥ 2 R  − 3 3
r cos A cos B cos C  r 
 2p   2p 
Vom demonstra că 2 R  − 3 3 ≥ 4p :2 ⇒ R − 3 3 ≥ 2p
 r   r 
⇒ 2 pR ≥ 3 3Rr + 2 pr
Se cunosc inegalităţile: R ≥ 2r , p ≥ 3 3r . Atunci Rp ≥ 2rp şi
Rp ≥ 3 3Rr ⇒ 2 Rp ≥ 2rp + 3 3Rr
a b c  2p  a b c
⇒ + + ≥ 2R  − 3 3 ≥ 4p ⇒ + + ≥ 4p .
cos A cos B cos C  r  cos A cos B cos C
Am obţinut o confirmare a inegalităţii date folosind inegalitatea lui T. Popoviciu.

Soluţia 2

tgA + tgB + tgC = tgAtgBtgC , tgA > 0, tgB > 0, tgC > 0 . Din inegalitatea mediilor avem că :
3
( tgAtgBtgC )
3
tgA + tgB + tgC ≥ 3 3 tgAtgBtgC ⇒ ≥ 27tgAtgBtgC ⇒ tgAtgBtgC ≥ 3 3
sau
1 3 3
sin A sin B sin C ≥ 3 3 cos A cos B cos C sau ≥
cos A cos B cos C sin A sin B sin C
abc ⋅ 3 3
a b c abc
( )
3
+ + ≥ 33 ≥ 33 ≥ 3 3 3 ⋅ 8R 3 = 3 3 ⋅ 2 R ≥ 4 p.
cos A cos B cos C cos A cos B cos C sin A sin B sin C
3 3R
Vom demonstra că 6 3R ≥ 4 p : 2 ⇒ p ≤ , care este cunoscută. Rezultă că
2
a b c
+ + ≥ 4 p , ceea ce trebuia demonstrat.
cos A cos B cos C
III. 25973, GM3/2008, pag 160

În triunghiul ABC are loc relaţia :


A B C  A B C
sin + sin + sin ≤ 2  sin 2 + sin 2 + sin 2  ,.
2 2 2  2 2 2
M.Olteanu, Râmnicu Vâlcea

Demonstraţie:
A B C 3
Demonstrăm că sin + sin + sin ≤ în orice V ABC .
2 2 2 2
x 1 x 1 x
1) Fie f ( x ) = sin , x ∈ ( 0, π ) , f ' ( x ) = cos , f '' ( x ) = − sin < 0, ∀ x ∈ ( 0, π ) ⇒ f concavă pe
2 2 2 4 2
( 0, π ) .
Din inegalitatea lui Jensen avem:
 x + x + x  f ( x1 ) + f ( x2 ) + f ( x3 )
f 1 2 3 ≥ , x1 = A ∈ ( 0, π ) ,
 3  3
x2 = B ∈ ( 0, π ) , x3 = C ∈ ( 0, π ) ⇒
A B C
sin + sin + sin A B C 3
 A+ B + C  2 2 2 ⇒ sin + sin + sin ≤ , ∀ V ABC .
⇒ sin  ≥ 2 2 2 2
 6  3

x  π   π 
2) Fie f ( x ) = 2sin 2 , x ∈  0,  , f :  0,  → ¡ , .
2  2  2
1 x x  π 
f ' ( x ) = 4 ⋅ sin cos = sin x ⇒ f '' ( x ) = cos x > 0, ∀ x ∈  0,  ⇒ f convexă pe
2 2 2  2
 π  A B C  π 
 0,  , A, B, C ∈ ( 0, π ) ⇒ , , ∈  0,  ⇒ după inegalitatea lui T. Popoviciu avem:
 2 2 2 2  2
 x+ y+ z    x+ y  y+ z  z+ x
f ( x) + f ( y) + f ( z ) + 3 f   ≥ 2 f  + f + f  ,
 3    2   2   2 
 π 
∀ x, y , z ∈  0,  .
 2

 π 
a) Vom demonstra inegalitatea în cazul în care triunghiul este ascuţitunghic, A, B, C ∈  0, 
 2
 A+ B + C    A+ B   B+ C   C + A
⇒ f ( A) + f ( B ) + f ( C ) + 3 f   ≥ 2 f  + f + f  ⇒
 3    2   2   2 

A B C  A+ B + C   2  A+ B  2 B+ C 
2sin 2 + 2sin 2 + 2sin 2 + 3 ⋅ 2sin 2   ≥ 2  2sin   + 2sin  +
2 2 2  6    4   4 
 C + A 2 A B C 1  A+ B B+ C C + A
+ 2sin 2   ⇒ 2sin + 2sin 2 + 2sin 2 + 6 ⋅ ≥ 2  1 − cos + 1 − cos + 1 − cos ≥
 4  2 2 2 4  2 2 2 
  A B C
≥ 2  3 −  sin + sin + sin  
  2 2 2 
A B C 3  A B C
⇒ 2sin 2 + 2sin 2 + 2sin 2 ≥ 6 − − 2  sin + sin + sin  ⇒
2 2 2 2  2 2 2
 A B C 9  A B C
2  sin 2 + sin 2 + sin 2  ≥ − 2  sin + sin + sin  . Vom demonstra că :
 2 2 2 2  2 2 2
9  A B C A B C
− 2  sin + sin + sin  ≥ sin + sin + sin ⇒
2  2 2 2 2 2 2
 A B C 9 A B C 3
⇒ 3  sin + sin + sin  ≤ ⇒ sin + sin + sin ≤ , adevărată în orice triunghi ABC.
 2 2 2 2 2 2 2 2
Deci,
 A B C 9  A B C A B C
2  sin 2 + sin 2 + sin 2  ≥ − 2  sin + sin + sin  ≥ sin + sin + sin ⇒
 2 2 2 2  2 2 2 2 2 2
 A B C A B C  π 
⇒ 2  sin 2 + sin 2 + sin 2  ≥ sin + sin + sin , unde A, B, C ∈  0,  .
 2 2 2 2 2 2  2
Am obţinut o “rafinare” inegalităţii date. Asemănător se poate deduce cazul când triunghiul este
dreptunghic sau obtuzunghic.

b) Dăm o soluţie în triunghiul oarecare. Am demonstrat că în orice triunghi ABC


A B C 3
+ sin + sin ≤ .
sin
2 2 2 2
3  2A B C A B C 2R − r
Demonstrăm că ≤ 2  sin + sin 2 + sin 2  . Dar, sin 2 + sin 2 + sin 2 = .
2  2 2 2 2 2 2 2R
3 2 ( 2R − r )
Deci, vom demonstra că ≤ , adică 3R ≤ 4 R − 2r ⇔ R ≥ 2r (Euler) adevărată.
2 2R
A B C 3  A B C
Deci, sin + sin + sin ≤ ≤ 2  sin 2 + sin 2 + sin 2  , ceea ce este echivalent cu
2 2 2 2  2 2 2
A B C  A B C
sin + sin + sin ≤ 2  sin 2 + sin 2 + sin 2  în orice triunghi ABC.
2 2 2  2 2 2

Profesor,
Colegiul Naţional “Gheorghe M. Murgoci”,
Brăila

BIBLIOGRAFIE

1. Gazeta Matematică
2. Revista de Matematică din Timişoara

UTILIZAREA UNOR IDENTITĂŢI COMBINATORICE ÎN REZOLVAREA UNOR


IDENTITĂŢI TRIGONOMETRICE

de Ani Drăghici şi Mădălina Călinescu

În cele ce urmează vom prezenta o metodă de rezolvare a două probleme :

1. Demonstraţi că:
π 2π ( n − 1) π n
+ ... + ( − 1) cos n
n− 1
1 − cos n + cos n = n− 1 .
n n n 2
(problema 363 din culegerea „Mathematical Quickies” autor Ch. Trigg).
2. Să se arate că pentru n ≥ 2, n ∈ ¥ avem:
π 2π ( n − 1) π n
1 + cos 2 n + cos 2 n
n n
+ ... + cos 2 n
n
= n 2 + C2nn
4
( )
(problema 22966 din Gazeta Matematică nr.3 din 1994, autor Vasile Bivolaru, Timişoara ).
Cele două rezultate se vor demonstra utilizând următoarele propoziţii:

Propoziţia 1

2kπ 2 kπ
Fie m ∈ ¥ ∗ şi ε k = cos + i sin , k = 1, m , rădăcinile de ordinul m ale unităţii. Dacă
m m
n ∈ ¥ , atunci:
 m dacă m n
ε 1n + ε 2n + ... + ε mn = 
/m
 0 dacă n ⋮
Demonstraţie:

Dacă m n atunci ε kn = 1 şi egalitatea este demonstrată. Dacă n ⋮


/ m , atunci există numerele
naturale N şi r , 0 < r < m astfel încât n = mN + r şi
r k
m m
 m
2 kπ 2 kπ  m
 2rπ 2rπ  m
ε rm − 1
∑k = 1 ε = ∑k = 1 ε = ∑k = 1  cos m + i sin m  =
n
k
r
k ∑k = 1  cos m + i sin m  = ∑
k= 1
ε rk = ε r
εr−1
= 0

ceea ce încheie demonstraţia propoziţiei 1.

Propoziţia 2

Dacă m, n ∈ ¥ ∗ , m ≤ n atunci:
 n
 m m 2n m
kπ nkπ
C +C +C
0
n
m
n
2m
n + ... + C 
n =
m

k=1
cos n
m
cos
m
(∗)
Demonstraţie:
n
Fie S = ( 1 + ε 1 ) + ( 1 + ε 2 ) + ... + ( 1 + ε m ) =
n n n
∑ ( )
Cnk ε 1k + ε 2k + ... + ε mk unde ε k , k = 1, m
k= 0
sunt rădăcinile de ordinul m ale unităţii. Conform propoziţiei 1 avem

 m dacă k m
m

s= 1
ε sk =  ∑ /k
 0 dacă m ⋮
 0  m m 
 n

şi deci S = m  Cn + Cn + Cn + ... + Cn   .


m 2m
 
 
Pe de altă parte
n
m
 2 kπ 2k π  m
 nkπ nkπ  n kπ
S = ( 1 + ε 1 ) + ( 1 + ε 2 ) + ... + ( 1 + ε n ) = ∑k = 1  ∑
n n n
1 + cos + i sin  = 2 n
 cos + i sin  cos .
m m  k=1  m m  m
S
Cum ∈ ¡ obţinem imediat identitatea din enunţ. În plus,
m

m
nkπ
k=1
∑m
cos n
sin
m
= 0

Trecând acum la rezolvarea problemelor 1 şi 2 şi făcând n = m în ( ∗ ) obţinem:


n
kπ n
kπ n
( )
n Cn0 + Cnn = 2n ∑ cos n
n
cos kπ sau ∑ ( − 1)
k
cos n = n− 1 ,
n 2
k= 1 k=1
adică concluzia problemei 1.
Dacă în ( ∗ ) înlocuim pe n cu 2n şi pe m cu n obţinem:
n
kπ n
kπ n
( )
n C20n + C2nn + C22nn = 22 n ∑ cos 2 n
n
cos 2kπ sau ∑ cos 2 n ( )
= n 2 + C2nn ,
n 4
k= 1 k=1
adică concluzia problemei 2.

BIBLIOGRAFIE

1. Ch.Trigg - Mathematical Quickies - 1967


2. Gazeta Matematică.

Profesori, Craiova

ASUPRA UNEI PROBLEME DIN GAZETA MATEMETICĂ

de prof. Mihaela Giurcă

C.O.4961(Gazeta Matematică nr.7-8/2008)

ABC avem:
Să se arate că în orice triunghi
a + b + c
2 2 2
1 1 1 a2 + b2 + c2
≥ + + ≥ .
2rp sin A sin B sin C Rp

Cristina – Paula Nica şi Nicolae Nica, Drăgăneşti-Olt

Soluţie dată de prof. Mihaela Giurca

Utilizăm următoarele formule :


a b c abc S
= = = 2R R = , şi r = ,
sin A sin B sin C 4S p
unde notaţiile sunt cele consacrate.
Din egalitatea :
(a + b + c) 2 = a 2 + b 2 + c 2 + 2(ab + ac + bc ) obţinem :
a 2 + b 2 + c 2 = (a + b + c) 2(ab + ac + bc ) -= 4 p - 2(ab + ac + bc)
2 2
(1)
Pentru a calcula expresia ab + ac + bc se va proceda în modul următor:
prelucrăm expresia
S 2 p ( p − a)( p − b)( p − c)
= = p 3 − p 2 (a + b + c) + p(ab + ac + bc) − abc (2),
p p
dar produsul abc = 4rRp şi a + b + c = 2 p . Înlocuim în egalitatea (2) şi avem :
r 2 p2 S 2
r2 p = = = p3 − 2 p 3 + p(ab + ac + bc) − 4rRp = − p 3 + p(ab + ac + bc) − 4rRp ⇒
p p
r 2 = − p 2 + ab + ac + bc − 4rR ⇒ ab + ac + bc = p 2 + r 2 + 4rR
Revenim în egalitatea (1) şi avem :
a 2 + b 2 + c 2 2 p − 2r − 8Rr =.
2 2

Calculăm suma:
2 2
1 1 1  1 1 1  ab + ac + bc  p + r + 4 Rr
+ + = 2R  + +  = 2R   = .
sin A sin B sin C  a b c  abc  2rp
Cu acestea, inegalitatea pe care trebuie să o demonstrăm, devine:
2 p 2 − 2r 2 − 8 Rr p 2 + r 2 + 4 Rr 2 p 2 − 2r 2 − 8Rr .
≥ ≥
2rp 2rp Rp
Demonstraţia primei inegalităţi:

Prin eliminarea numitorilor şi separarea termenilor, obţinem:


p 2 ≥ 3r (4 R + r ) , inegalitate care este adevărată deoarece a, b, c sunt rădăcinile ecuaţiei:
x 3 − 2 px 2 + ( p 2 + r 2 + 4rR) x − 4 Rrp = 0
Dar ecuaţia anterioară are trei rădăcini reale ceea ce implică faptul că derivata funcţiei
f ( x ) = x3 − 2 px 2 + ( p 2 + r 2 + 4rR) x − 4 Rrp
are două rădăcini reale, ceea ce este echivalent cu faptul că discriminantul ecuaţiei de gradul al II-lea este
pozitiv. După efectuarea calculelor se obţine: p 2 ≥ 3r (4 R + r ) .

Demonstrarea inegalităţii a II-a:

După eliminarea numitorilor şi separarea termenilor, obţinem:


4 R 2 r + r 2 R + 16 Rr 2 + 4r 3 ≥ p 2 (4r − R ) .
Apar două situaţii:
17. dacă 4r - R ≤ 0 , inegalitatea este evidentă;
R
18. dacă 4r - R ≥ 0 avem că : 2 ≤ ≤ 4.
r
În cea de a doua situaţie avem că:
(4 R + r ) 2 27 R 2
27 r 2 ≤ 3(4 Rr + r 2 ) ≤ p 2 ≤ ≤ .
3 4
27 R 2 4 R 2 r + r 2 R + 16 Rr 2 + 4r 3
Atunci vom demonstra că : p 2 ≤ ≤ .
4 4r − R
După eliminarea numitorilor se ajunge la :
16 R 2 r + 4 Rr 2 + 64 Rr 2 + 16r 3 − 108 R 2 r + 27 R3 ≥ 0
R
Utilizând substituţia = x , se ajunge la inegalitatea :
r
27 x 3 − 92 x 2 + 68 x + 16 ≥ 0, ∀ x ∈ [ 2,4] .
Printr-o grupare convenabilă a termenilor inegalitatea anterioară este adevărată :
( x − 2)(27 x 2 − 38 x − 8) ≥ 0 .
Deci inegalităţile respective sunt demonstrate.

Prof.
Colegiul Economic ”Ion Ghica ”, Brăila

PROBLEME DATE LA OLIMPIADA JUDEŢEANĂ CU SOLUŢII INGENIOASE

de Dan Negulescu şi Radu Vasile

1. Fie ( xn ) n≥ 1 , ( yn ) n≥ 1 două şiruri de numere reale strict pozitive,astfel încât pentru ∀ n ∈ ¥,


xn + yn xn2 + yn2 .
xn + 1 ≥ , yn + 1 ≥
2 2
a) Să se arate că şirurile ( xn + yn ) n≥ 1 , şi ( xn yn ) n≥ 1 au limită.

b) Să se arate că şirurile ( xn ) n≥ 1 , ( yn ) n≥ 1 au limită şi limitele lor sunt egale.

Soluţie: ( prof. Dan Negulescu )

xn + yn
a) Obţinem imediat că şi yn + 1 ≥ , ∀ n ∈ ¥ şi deci xn + 1 + yn + 1 ≥ xn + yn , ∀ n ≥ 1 adică
2
sn = xn + yn
este monoton crescător, deci are limită.
De asemenea obţinem xn + 1 ≥ xn yn şi yn + 1 ≥ xn yn ⇒ xn + 1 yn + 1 ≥ xn yn , ∀ n ≥ 1 şi deci
pn = xn yn este monoton crescător, deci are limită.
2
 x + yn  x + yn
≥  n
b) Deoarece xn + 1 yn + 1  ≥ xn yn , ∀ n ≥ 1 obţinem că ∃ lim xn yn = lim n = L.
 2  n→ ∞ n→ ∞ 2
i) Dacă L = ∞ atunci xn → ∞ şi yn → ∞
L < ∞ deoarece ( xn + yn ) n≥ 1 este convergent şi xn − yn = ( xn + yn ) − 4 xn yn ⇒
2
ii) Dacă

⇒ 4 L2 − 4 L2 = 0 obţinem că ( xn ) n≥ 1 , ( yn ) n≥ 1 au limită şi limitele lor sunt egale.

2. Fie A, B ∈ M n ( ¡ ) cu proprietatea B 2 = I n şi A2 = AB + I n . Să se demonstreze că


n
 1+ 5 
det A ≤   .
 2 
Soluţie : ( prof. Radu Vasile )

A2 = AB + I n ⇒ A3 = A2 B + A = ( AB + I n ) B + A = AB 2 + A + B = 2 A + B ⇒ A3 = 2 A + B ⇒
( AB ) ( AB )
2 2
⇒ B = A3 − 2 A ⇒ AB = BA ⇒ = A 2 B 2 = A2 ⇒ = AB + I n
Notăm AB = C . DinB = I n ⇒ det B = 1 ⇒ det C = det A . Matricea C verifică relaţia
2

C 2 = C + I n ⇒ C 2 − C − I n = On . Fie λ o valoare proprie pentru C . Atunci există un vector nenul v


astfel încât
( ) (
Cv = λ v ⇒ C 2 v = C ( Cv ) = C ( λ v ) = λ 2 v ⇒ C 2 − C − I n v = λ 2 − λ − 1 v ⇒ λ 2 − λ − 1 v = 0, ) ( )
v ≠ 0 ⇒ λ − λ − 1 = 0 . Deci orice valoare proprie a lui C este soluţie a ecuaţiei λ − λ − 1 = 0 ⇒
2 2

1± 5
λ = .
2
 1 − 5 1 + 5 
Fie λ 1 , λ 2 ,..., λ n valorile proprii ale lui C , λ i ∈  ,  . Se ştie că
 2 2 
det C = λ 1λ 2 ...λ n ,
n n n
 1+ 5   1+ 5   1+ 5 
deci det C = λ 1 λ 2 ... λ n ≤   ⇒ det A ≤   ⇒ det A ≤   .
 2 
   2   2 

Profesori,
Colegiul Naţional “Gheorghe M. Murgoci”,
Brăila

BIBLIOGRAFIE

3. Gazeta Matematică
4. Revista de Matematică din Timişoara
TEOREMA DE EXISTENŢĂ A PRIMITIVELOR UNEI FUNCŢII CONTINUE

de Mădălina Teodorescu

Pentru orice funcţie continuă f : [ a , b] → ¡ , funcţia F : [ a, b] → ¡ definită prin


x
F ( x) = ∫ f (t )dt , ( ∀ ) x ∈ [ a, b] este o primitivă a lui f care se anulează în punctul a.
a

În manualele şcolare demonstraţia se bazează pe teorema de medie şi este făcută


cu ajutorul noţiunii de integrală.
Prezentăm în continuare o altă demonstraţie a teoremei care se bazează pe faptul
că o funcţie continuă pe un interval compact este limita uniformă a unui şir de
polinoame.
În acest scop vom prezenta mai întâi câteva definiţii şi criterii de convergenţă la
şiruri de funcţii.

DEFINIŢIE
Fie ( f n ) n un şir de funcţii definite pe o mulţime A şi f o funcţie definită tot pe
mulţimea A. Spunem că şirul ( f n ) n este simplu convergent pe mulţimea A către funcţia f
dacă (∀ ) x∈ A şi (∀ )ε > 0 , există nε ,x ∈ ¥ astfel încât ( ∀ ) n > nε ,x să avem
fn ( x) − f ( x) < ε .

DEFINIŢIE
Fie ( f n ) n un şir de funcţii definite pe mulţimea A. Spunem că şirul ( f n ) n este
uniform convergent pe mulţimea A către funcţia f definită pe A dacă ( ∀ ) ε > 0 , există
nε ∈ ¥ , astfel încât ( ∀ ) n > nε şi ( ∀ ) x ∈ A , avem f n ( x ) − f ( x ) < ε .

TEOREMA I (CAUCHY)
Fie ( f n ) n un şir de funcţii definite pe o mulţime A. Şirul ( f n ) n este uniform
convergent către o funcţie f definită pe A dacă şi numai dacă ( ∀ ) ε > 0 există nε ∈ ¥ astfel
încât , ( ∀ ) m, n ≥ nε şi ( ∀ ) x ∈ A avem f n ( x ) − f m ( x ) < ε .

CONSECINŢĂ
Fie ( f n ) n un şir de funcţii definite pe mulţimea A. Şirul ( f n ) n este uniform
convergent către o funcţie f definită pe A dacă şi numai dacă ( ∀ ) ε > 0 există nε ∈ ¥ astfel
încât, ( ∀ ) n ≥ nε şi ( ∀ ) p ∈ ¥ , ( ∀ ) x ∈ A avem f n + p ( x ) − f n ( x ) < ε .
Dăm în continuare demonstraţia a două teoreme ce stabilesc legătura dintre
derivabilitate şi convergenţa uniformă. Pe baza acestora vom arăta că, dacă ( g n ) n este un
şir de funcţii uniform convergent pe intervalul I către o funcţie g şi dacă funcţiile g n au
primitive pe I, atunci şi limita g are primitive pe I.

TEOREMA 2.
TEOREMA DE DERIVARE TERMEN CU TERMEN A ŞIRURILOR DE
FUNCŢII
Fie ( f n ) n un şir de funcţii derivabile pe un interval I. Dacă:
1) şirul ( f n ) n este uniform convergent pe I către o funcţie f definită pe I.
2) şirul derivatelor ( f 'n ) este uniform convergent pe I către o funcţie g,
atunci funcţia f este derivabilă pe I şi f ' = g .

DEMONSTRAŢIE:
Fie a ∈ I un punct oarecare; arătăm că f derivabilă în a şi f ' ( a ) = g ( a )
(
f 'n ) şir uniform convergent, rezultă conform criteriului
Fie ε > 0 . Deoarece
ε
Cauchy că există nε ∈ ¥ astfel încât dacă m, n ≥ nε să avem f n ( x ) − f m ( x ) < ,
' '

3
(∀ ) x∈ I .
Alegem n0 > nε . Cum f m'  u→ g obţinem f n'0 − f m'  u→ f n'0 − g iar din
ε
inegalitatea precedentă deducem că f n0 ( x ) − g ( x ) ≤ , ( ∀ ) x ∈ I .
'

3
Deoarece funcţia f n0 este derivabilă în a, pentru ε ales, există o vecinătate U a lui
f n ( x ) − f n0 ( a ) ε
a, astfel încât 0 − f n'0 ( a ) < , ( ∀ ) x ∈ U ∩ I .
x− a 3
Putem scrie
f n ( x ) − f n (a ) f m ( x ) − f m ( a ) ( f n ( x ) − f m ( x ) ) − ( f n ( a ) − f m (a ) ) ε
− = = f n' (c ) − f m' ( c) <
x− a x− a x− a 3
dacă m, n ≥ nε , ( ∀ ) x ∈ I unde c este un punct cuprins între x şi a, provenit din aplicarea
teoremei lui Lagrange funcţiei f n − f m .
f m  u→ f , avem f n − f m  u→ f n − f ; din inegalitatea precedentă
Cum 0 0

deducem pentru n = n0 şi m → ∞ că
f n0 ( x ) − f n0 ( a ) f ( x ) − f ( a ) ε
− ≤ ,( ∀ ) x∈ I .
x− a x− a 3
Pentru ( ∀ ) x ∈ U ∩ I deducem atunci
f ( x ) − f (a ) f ( x ) − f ( a ) f n0 ( x ) − f n0 ( a ) f n ( x ) − f n0 ( a )
− g (a ) ≤ − + 0 − f n'0 (a ) +
x− a x− a x− a x− a
ε ε ε
+ f n'0 ( a ) − g ( a ) < + + = ε.
3 3 3
Aşadar, pentru ( ∀ ) ε > 0 , putem găsi o vecinătate U a lui a, astfel încât să avem
f ( x ) − f (a )
− g ( a ) < ε , ( ∀ ) x ∈ U ∩ I . Aceasta înseamnă că f este derivabilă în
x− a
x = a şi că f ' ( a ) = g ( a ) . Cum a a fost ales arbitrar, deducem că f este derivabilă pe I şi
că f ' = g , ceea ce încheie demonstraţia.

OBSERVAŢII
1. Deoarece f = lim f n şi g = lim f n' , egalitatea f '= g se poate scrie
n→ ∞ n→ ∞

( )
'
lim f n = lim f n' de unde şi numele teoremei.
n→ ∞ n→ ∞

2. Convergenţa uniformă a şirului ( f n ) n nu atrage convergenţa uniformă a şirului


derivatelor. Justificăm afirmaţia făcută prin următorul exemplu:
cos nx
Fie f n : [0, π ] → ¡ , definită prin f n ( x ) = , f ( x ) = 0 . Avem f n  → f .
u

n
cos nx 1
Într-adevăr, f n ( x ) − f ( x ) = ≤  n→ ∞→ 0 .
n n
Funcţiile ( f n ) sunt derivabile pe I, f n ( x ) = − sin nx. Şirul derivatelor ( f n ' ) nu
'

este însă convergent pe I.

TEOREMA 3
Fie I un interval mărginit şi ( f n ) un şir de funcţii derivabile pe I. Dacă
1) şirul ( f n ) este convergent într-un punct x0 ∈ I
2) şirul derivatelor ( f n ' ) este uniform convergent pe I către o funcţie g, atunci
i) şirul ( f n ) este uniform convergent pe I către o funcţie f
ii) limita f este derivabilă pe I şi f ' = g .

DEMONSTRAŢIE:
Avem de demonstrat doar punctul i) deoarece ii) rezultă din teorema 5.
Vom nota cu l lungimea intervalului I. Presupunem l > 0 . Fie ε > 0 . Din
ipotezele 1) şi 2) deducem că există n ε ∈ ¥ astfel încât pentru ( ∀ ) n ≥ nε şi ( ∀ ) m ≥ nε şă
avem, în acelaşi timp, verificate următoarele inegalităţi:
ε ε
f n ( x0 ) − f m ( x0 ) < şi f n ( x ) − f m ( x ) < , (∀ ) x∈ I
' '

2 2l
Atunci,
f n ( x ) − f m ( x ) ≤ f n ( x ) − f m ( x ) − ( f n ( x0 ) − f m ( x0 ) ) + f n ( x0 ) − f m ( x0 )
Dacă notăm h = f n − f m , h va fi o funcţie derivabilă pe intervalul închis cu
extremităţile x şi  x0 , deci i se poate aplica teorema lui Lagrange. Atunci există un punct c
între x şi  x0 , astfel încât h ( x ) − h ( x0 ) = h ' ( c ) ( x − x0 ) de unde
f n ( x ) − f m ( x ) − ( f n ( x0 ) − f m ( x0 ) ) = h( x ) − h ( x0 ) = x − x0 ⋅ h '( c ) =
ε ε
= , ( ∀ ) m, n ≥ nε , ( ∀ ) x ∈ I
= x − x0 ⋅ f n' (c ) − f m' ( c ) ≤ l ⋅
2l 2
ε ε
În concluzie, f n ( x ) − f m ( x ) ≤ + = ε , ( ∀ ) m, n ≥ nε , ( ∀ ) x ∈ I .
2 2
Conform criteriului de convergenţă uniformă a lui Cauchy, rezultă că şirul ( f n )
converge uniform pe I către o funcţie f, ceea ce încheie demonstraţia.
Ne interesează cum se comportă proprietatea de a avea primitive în raport cu
operaţia de trecere la limită.

TEOREMA 4
Limita g a unui şir uniform convergent ( g n ) de funcţii care admit primitive pe
intervalul I este o funcţie care admite primitive pe I.
DEMONSTRAŢIE
Presupunem pentru început că intervalul I este mărginit şi fie x0 ∈ I . Pentru
fiecare n, fie f n o primitivă a funcţiei g n . Deci f n derivabilă pe I şi g n = f n . Putem alege
'

primitive f n astfel încât f n ( x0 ) = 0 . Suntem acum în condiţiile teoremei 3 şi rezultă că


şirul ( f n ) este uniform convergent pe I către o funcţie derivabilă f şi f ' = g , deci f este o
primitivă a funcţiei g pe intervalul I.
Să presupunem acum că intervalul I este nemărginit. Putem construi un şir
crescător ( I n ) de intervale mărginite, a căror reuniune să fie egală cu I:

I1 ⊂ I 2 ⊂ ... ⊂ I n ⊂ ... ⊂ I şi UI
n= 1
n = I

Fie x0 ∈ I1 , deci x0 ∈ I n , pentru orice n. Conform primei părţi a demonstraţiei, pe


fiecare interval mărginit I n , funcţia g este derivata unei funcţii Fn definită pe I n . Putem
alege funcţiile Fn astfel ca Fn ( x0 ) = 0 . De remarcat faptul că, dacă x ∈ I n şi x ∈ I m ,
atunci Fn ( x) = Fm ( x) deoarece funcţiile Fn şi Fm au pe intervalul I n ⊂ I m aceeaşi
derivată g. Deci diferenţa lor pe I n este o constantă; dar Fn ( x0 ) = Fm ( x0 ) = 0 , deci
Fn ( x) = Fm ( x), ( ∀ ) x ∈ I n .
Definim acum funcţia F pe întregul interval I astfel: dacă un punct x ∈ I se află
într-un interval I n , atunci luăm F ( x) = Fn ( x). Conform observaţiei de mai sus, numărul
F ( x) este independent de alegerea particulară a intervalului I n care conţine pe x. Funcţia
F este derivabilă pe I şi F ' = g . Pentru a demonstra această afirmaţie vom considera un
punct arbitrar ales a ∈ I . Atunci există n ∈ ¥ astfel încât a ∈ I n , deci F (a ) = Fn (a ). Dar
Fn derivabilă în a şi Fn' (a ) = g (a ) . Rezultă că F este derivabilă în a şi
F '(a ) = Fn' (a ) = g (a ). Cum a a fost ales arbitrar în intervalul I, rezultă că funcţia F este
derivabilă pe I şi F ' = g , deci g admite primitive pe I.
În teoremele care urmează vom demonstra că funcţiile continue pot fi aproximate uniform cu
funcţii aparţinând unei mulţimi mai restrânse de funcţii continue.
Următoarea teoremă o vom enunţa, fără a-i face demonstraţia.

TEOREMA 5 – TEOREMA LUI WEIERSTRASS – STONE


Fie intervalul compact I = [ a, b ] şi A o algebră de funcţii continue definite pe intervalul I.
Dacă:
1. funcţia identică 1I : I → I ,1I ( x ) = x aparţine lui A
2. pentru orice puncte x '
'
( ) ''
( )
≠ x '' există o funcţie f ∈ A astfel încât f x ≠ f x atunci orice
funcţie continuă pe I este limita uniformă a unui şir de funcţii din A.

TEOREMA 6 – TEOREMA LUI WEIERSTRASS –


Orice funcţie continuă pe un interval compact I = [ a, b ] este limita uniformă pe I a unui şir de
polinoame.

DEMONSTRAŢIE
Notăm cu P algebra polinoamelor, funcţia identică egală cu 1 pe I, f ( x ) ≡ 1 este un polinom,
deci aparţine algebrei P . Dacă ( )
' ''
( )
x ' , x '' ∈ I cu x ' ≠ x '' , pentru funcţia P şi f x ≠ f x . Am arătat
astfel că sunt îndeplinite condiţiile 1) şi 2) din teorema 5, deci orice funcţie continuă pe I = [ a, b ] este
limita uniformă a unui şir de funcţii din P , deci a unui şir de polinoame.
Putem reveni în acest moment asupra demonstrării teoremei de existenţă a primitivelor unei
funcţii continue.
Fie f : [ a, b ] → ¡ o funcţie continuă pe [ a, b ] . Conform teoremei 6, există un şir de

polinoame ( Pn ) n≥ 0 cu proprietatea Pn  u→ f.


Fie ( Qn ) n≥ 0 şirul de polinoame cu proprietatea că Qn = Pn , ∀ n ≥ 0 pe [ a, b ] şi Qn ( a ) = 0
'

. Conform teoremei 3, şirul ( Qn ) n≥ 0 este uniform convergent pe [ a, b ] către o funcţie F : [ a, b ] → ¡

şi F derivabilă pe intervalul [ a, b ] cu F ' = f . În concluzie, funcţia f admite primitive pe [ a, b ] , o


primitivă a sa fiind F.

Prof.
Liceul Pedagogic „D. P. Perpessicius ”Brăila

PROBLEME DE BACALAUREAT CU… PROBLEME

de Botea Viorel

m 
1) Fie mulţimea ¤ 0 =  , m, n ∈ ¢ , m, n impare  şi G = ¤ 0 × ¢ . Pe G se defineşte legea de compoziţie:
 n 
( g1 , k1 ) ∗ ( g 2 , k2 ) = ( g1 g 2 , k1 + k2 ), ( ∀ ) g1 , g 2 ∈ ¤ 0 , ( ∀ ) k1 , k2 ∈ ¢ .
a) Demonstraţi că (G , ∗ ) este grup abelian.
b) Calculaţi (1,1) ∗ (1,2) ∗ ... ∗ (1,2008).
c) Demonstraţi că funcţia f : G → ¤ ∗ , f (q, k ) = q ⋅ 2k este un izomorfism între grupurile (G, ∗ ) şi (¤ ∗ ,⋅ ).
Soluţie :
m1 m
a) ( g1 , k1 ) * ( g2 , k2 ) = ( g1 ⋅ g2 , k1 + k2 ) ∈ G deoarece g1 , g2 ∈ ¤ 0 pentru că g1 = , g2 = 2 ,
n1 n2
m1 , n1 , m2 , n2 impare ⇒ m1m2 şi n1n2 impare; k1 + k2 ∈ ¢ .
( g1 , k1 ) ∗ ( g 2 , k2 ) = ( g2 , k2 ) ∗ ( g1 , k1 ), ∀ ( g1 , k1 ),( g 2 , k2 ) ∈ G.
[( g1 , k1 ) ∗ ( g 2 , k2 )] ∗ ( g3 , k3 ) = ( g1 g 2 g3 , k1 + k2 + k3 ) = ( g1 , k1 ) ∗ [( g 2 , k2 ) ∗ ( g3 , k3 )].
Demonstrăm că ∃ ( e1 , e2 ) ∈ G astfel încât ( ∀ ) ( g1 , k1 ) ∈ G avem ( g1 , k1 ) ∗ ( e1 , e2 ) = ( g1 , k1 ) ⇒
⇒ g1e1 = g1 şi k1 + e2 = k1 ⇒ e1 = 1 şi e2 = 0 ⇒ (1,0) element neutru.
1
Demonstrăm că ( ∀ ) ( g1 , k1 ), ∃ ( g ', k ') astfel încât ( g1 , k1 ) ∗ ( g ', k ') = (1,0) ⇒ g 'şi=
g1
k ' = -k1.
 2008 ⋅ 2009 
b) (1,1) ∗ (1, 2) ∗ ... ∗ (1, 2008) = (1,1 + ... + 2008) =  1, 
 2 
k1 + k2
c) f (( g1 , k1 ) ∗ ( g2 , k2 )) = f (( g1 g2 , k1 + k2 )) = g1 g 2 ⋅ 2 = g1 ⋅ 2 ⋅ g 2 ⋅ 2k2 = f ( g1 , k1 ) ⋅
k1

f ( g2 , k2 ) ⇒ f morfism de grupuri.
m1 m
Fie f ( g1 , k1 ) = f ( g2 , k2 ) ⇒ g1 ⋅ 2k1 = g 2 ⋅ 2k2 , g1 = , g 2 = 2 , cu m1 , n1 , m2 , n2
n1 n2
m1 m2
impare ⇒ m1 ⋅ 2k1 ⋅ n2 = m2 n1 ⋅ 2k2 ⇒ m1n2 = m2 n1 şi k1 = k1 ⇒ = ⇒ g1 = g 2 şi k1 = k2 ⇒
n1 n2
⇒ f injectivă.
Fie y ∈ ¤ ∗ . Demonstrăm că ∃ g ∈ ¤ 0 şi ∃ k ∈ ¢ cu f ( g , k ) = y ⇔ g ⋅ 2k = y.
a a
Fie y =
, a , b ∈ ¢ , ( a , b) = 1 ⇒ g ⋅ 2 k = .
b b
a a
1. Dacă a, b impare ⇒ g = ⇒ g = şi k = 0.
b b
a' a'
2. Dacă a = 2l ⋅ a ' , a ' impar , l ∈ ¥ şi b impar ⇒ g ⋅ 2k = 2l ⋅ ⇒ g= şi k = l.
b b
a a
3. Dacă a = impar şi b = 2t ⋅ b' , b' impar şi t ∈ ¥ ⇒ g ⋅ 2 k = t ' ⇒ g = ' şi k = t.
2 ⋅b b
2. Fie a, b, c ∈ ¡ şi polinomul f = 2 x 4 + 2(a − 1) x 3 + (a 2 + 3) x 2 + bx + c.
a) Să se afle a, b, c dacă a = b = c iar restul împărţirii lui f la x + 1 este 10.
b) Ştiind că x1 , x2 , x3 , x4 ∈ £ sunt rădăcinile lui f , să se calculeze x12 + x22 + x32 + x42 .
c) Aflaţi a, b, c ∈ ¡ şi rădăcinile polinomului f în cazul în care acesta are toate rădăcinile reale.

Soluţie :

a) f = 2 x 4 + 2(a − 1) x 3 + (a 2 + 3) x 2 + ax + a ⇒ f (− 1) = 10 ⇔ 2 − 2(a − 1) + a 2 + 3 − a + a = 10 ⇔
⇔ a 2 - 2a + 7 = 10 ⇔ a 2 - 2a − 3 = 0 ⇔ a = b = c ∈ { -1,3}
a2 + 3
x12 + x22 + x32 + x42 = S12 − 2 S 2 = ( − a + 1) − 2
2
b) = a 2 − 2a + 1 − a 2 − 3 = − 2 a − 2
2
c)

∑ (x − x ) = ( x1 − x2 ) + ( x1 − x3 ) + ( x1 − x4 ) + ( x2 − x3 ) + ( x2 − x4 ) + ( x3 − x4 ) =
2 2 2 2 2 2 2
i j
1≤ i ≤ j ≤ 4

a2 + 3
= 3 ( x12 + x22 + x32 + x42 ) − S 2 = 3 ( − 2a − 2 ) − 2 = 6a − 6 − a 2 − 3 = − a 2 − 6a − 9 = − ( a + 3 ) ≤ 0 ⇒
2

2
S 1− a 4 c b
⇒ a = − 3 şi x1 = x2 = x3 = x4 = 1 = = = 1⇒ = S 4 = 1 ⇒ c = 2 ⇒ − = S3 = 4 ⇒ b = − 8.
4 4 4 2 2
3) Se consideră funcţia f : ¡ → ¡ , f ( x ) = x + x + 1 .
3

1
a) Demonstraţi că ∀ n ∈ ¥ , ecuaţia f ( x) = 3 + are o unică soluţie xn ∈ ¡ .
n+ 1
b) Demonstraţi că lim xn = 1 , unde xn este precizat la a).
n→ ∞

c) Aflaţi lim n( xn − 1) , unde xn este precizat la a).


n→ ∞
Soluţie :
1
a) Fie g ( x) = f ( x) − 3 −
, g : ¡ → ¡ , g continuă;
n+ 1
lim g ( x ) = ∞ > 0; lim g ( x ) = − ∞ ; lim g ( x ) ⋅ lim g ( x ) < 0 ; g strict crescătoare pe ¡ atunci
x→ ∞ x→ ∞ x→ ∞ x→ ∞

∃ xn ∈ ¡ rădăcină pentru g.
3
1
< 0 ; g  1 −  =  1 −  + 1 − − 2 −
1 1 1 1 1 3 3 1 4
b) g ( 1) = − = − 3+ 2− − − =
n+ 1  n  n n n+ 1 n n n n n+ 1
− 5n3 − n 2 + 3n − 1  1 
= < 0 ⇒ xn ∈  1 − ,1 ⇒ xn  n→ ∞→ 1 .
n ( n + 1)
3
 n 
1 1
c) xn + xn + 1 − 3 −
3
= 0 ⇔ ( xn3 − 1) + ( xn − 1) = ;
n+ 1 n+ 1
1 1 n 1 1
( xn − 1) ( xn2 + xn + 2 ) = ⇒ xn − 1 = ⇒ n ( xn − 1) = ⋅ 2 → ⇒
n+ 1 ( n + 1) ( xn + xn + 2 )
2
n + 1 xn + xn + 2 4
1
⇒ lim n ( xn − 1)  n→ ∞→ .
n→ ∞ 4
4) Se consideră funcţiile f n : (0, ∞ ) → R f ( x ) = x + ln x, n ∈ ¥
n *
a) Arătaţi că ecuaţia f n ( x) = 2 are soluţia unică xn .
n

b) Arătaţi că lim xn = 2, unde ( xn ) n∈ ¥ * este de la a ).


n→ ∞
Soluţie :
n− 1 1
a) Fie h( x) = x n + ln x − 2n , x ∈ (0, ∞ ), h continuă; h '( x) = nx + > 0, ∀ x > 0 ⇒ h strict
x
crescătoare;
lim h( x) = − ∞ < 0 h( x) = + ∞ ⇒ ∃ ! xn ∈ (0, ∞ ) rădăcină pentru h .
x→ 0 şi x > 0 lim
x→ ∞
x> 0

b) h(2) = ln 2 > 0;
 1  
n n− 1 n− 2
 1  1  1 1  1  1
h  2 −  =  2 −  + ln  2 −  − 2n =  −    2 −  +  2 −  ⋅ 2 + ... + 2n − 1  + ln  2 −  .
 n  n  n  n    n  n   n
Demonstrăm că:
1 1  
n− 1
 1
ln  2 −  <   2 −  + ... + 2n − 1  ;
 n  n   n 
Avem
1   1 
n− 1 n− 1 n− 1
1 1  1 1  1
⋅   2 −  + ⋯ + 2n − 1  > ⋅  2 −  ⋅ n =  2 −  > 2 − > ln  2 −  deoarece x > ln x, ( ∀ ) x > 1 ⇔
n   n  n  n  n n  n
 1  1  1 
⇔ e x > x ⇒ h  2 −  < 0 ⇒ h(2) ⋅ h  2 −  < 0 ⇒ xn ∈  2 − , 2  ⇒ lim xn = 2 .
 n  n  n  n→ ∞

π
π
5 ) Se consideră şirul de numere reale ( Ι n )n∈ ¥ , definit deşiΙ 0 = Ι n = ∫ 2 , n x dx. n ∈ ¥ *
cos
2 0

a) Calculaţi Ι 1 .
b) Demonstraţi că ( Ι n )n∈ ¥ este descrescător.
π
c) Demonstraţi că nΙ n ⋅ Ι n + 1 = , ∀ n ∈ ¥ *.
2
Soluţie:
π π
a) Ι 1 = ∫ 2
0
cosn x dx = sin x / 02 = 1.
π
b) Ι n + 1 − Ι n = ∫ 0
2
cosn x ( − 1 + cos x )dx ≤ 0, ∀ n ∈ ¥ .
π π π
c) Ι n = ∫ 0
2
cosn x dx = cosn − 1 x ⋅ sin x / 02 − ∫ 0
2
(n − 1) cosn − 2 x.
π
sin x dx = ( n − 1) ∫ cosn − 2 x (1 − cos2 x )dx = ( n − 1)Ι n − 2 − ( n − 1)Ι n ⇒ nΙ n = ( n − 1) Ι n − 2
2 2
0

2k
Pentru n impar ⇒ Ι 2 k + 1 = ⋅ Ι 2k − 1 , ∀ k ∈ ¥ *
2k + 1
2k − 2
Ι 2k − 1 = ⋅ Ι 2k − 3
2k − 1
...............................
2
Ι 3 = ⋅ Ι 1, Ι 1 = 1
1
________________
(2k )!!
Ι 2k + 1 = ⋅Ι1,
(2k + 1)!!
1 π
⇒ Ι 2k ⋅ Ι 2k + 1 = ⋅
2k + 1 2
π
Pentru n impar (2k + 1) ⇒ n ⋅ Ι n ⋅ Ι n − 1 =
2
(2k + 1)!! π
Pentru n par (2k + 2) ⇒ Ι 2 k + 2 ⋅ Ι 2 k + 1 = ⋅
(2k + 2)!! 2
(2k )!! 1 π π
= ⋅ ⇒ n ⋅ Ι n ⋅ Ι n− 1 = .
(2k + 1)!! 2k + 2 2 2

Profesor,
Colegiul Naţional ” Nicolae Bălcescu”
Brăila
CONCURSUL INTERJUDEŢEAN DE MATEMATICĂ
“ PETRU MOROŞAN-TRIDENT ”
Ediţia a V-a , Secţiunea A (M 1 ),
Brăila, 23 - 25. 11. 2007

Clasa a V a

1. Împărţind un număr natural „ a ” la 2007 şi 4014 se obţin resturile 20 şi 23. Ştiind că diferenţa câturilor
obţinute este 55, aflaţi ultima cifră a lui „ a ”.

(∗ ∗ ∗ )
2. a) Arătaţi că 1 + 3 + 5 + ... + 2007 este pătrat perfect.
b) Suma a 40 de numere impare, naturale, distincte este 1602. Arătaţi că cel puţin unul este mai mare
decât 80.

Prof. Daniela Tilincă, Brăila

3. Să se scrie numărul 20062005 ca:


a) suma a cinci pătrate perfecte, distincte, nenule.
b) suma a şapte pătrate perfecte, distincte, nenule.

Prof. Nicolae Ivăşchescu, Craiova

Clasa a VI a

1. Dacă x, y , z sunt numere cu proprietatea că 2 x − 3 y − 10 z = 0 , demonstraţi că y ( x + z )( x + y )


este un număr divizibil cu 30.

Prof. Nazeli Boicescu, Brăila

2. Arătaţi că ecuaţia 4 x − 5 y = 1 nu are soluţii în mulţimea numerelor naturale.


Prof. Ani Drăghici, Doina Chiriac, Craiova

·AOB şi BOC
3. Fie unghiurile suplementare · cu m(·AOB ) > m(·BOC ) . Două puncte M şi P sunt în

( ) ( )
·
interiorul ·AOB astfel încât m MOP ·
= m BOC şi ( OX , ( OY sunt bisectoarele MOA
· şi

· .Demonstraţi că m ( ·XOY ) ≤ 90 . Când avem egalitate?


0
respectiv POB

Prof. Nicolae Stănică, Brăila

Clasa a VII a

1. Fie a1 , a2 ,..., a7 ∈ ¥ ∗ având media aritmetică egală cu 40 şi


d
( a1 , a2 ,..., a7 ) ( 1,1(1); 2,2(2);...7,7(7) ) şi c1 , c2 ,..., c7 ∈ ¥ * , cu media aritmetică egală cu 105
:
d  1,1(1) 1,1(1) 1,1(1) 1,1(1) 
şi ( c1 , c2 ,..., c7 )  ; ;..., , .
:  1,1(1)⋅ 2,2(2) 2,2(2)⋅ 3,3(3) 6,6(6)⋅ 7,7(7) 7,7(7)⋅ 8,8( 8 ) 
Determinaţi a1 , a2 ,..., a7 , c1 , c2 ,..., c7 .

Prof. Daniela Covaci, Brăila

2. Fie A, B, C , D astfel încât m ( S CAD ) = m ( S CBD ) = 90o . Să se arate că:


a) AB ≤ CD ;
b) [ AB ] ≡ [ CD ] dacă şi numai dacă cele patru puncte sunt vârfurile unui dreptunghi.

Prof. Dan Negulescu, Brăila

3. Se dă pătratul ABCD şi fie E , F mijloacele laturilor [ AB ] , respectiv [ BC ] . Să se arate că:


a) CE ⊥ DF ;
b) AD = AM , unde { M } = CE ∩ DF .
Prof. V.Chiriac, Bacău

Clasa a VIII a

x( y + z) y ( x + z) z ( x + y) 5
1. Demonstraţi că + + ≤ ( x + y + z ) , ∀ x, y , z ≥ 0 . În ce caz
3 3 3 6
avem egalitate?
(∗ ∗ ∗ )
2. Se dă cubul [ ABCDA ' B ' C ' D '] . Fie M mijlocul segmentului [ D ' C '] şi N mijlocul segmentului

[ BC ] .
a) Aflaţi muchia cubului, ştiind că aria V AMN este egală cu 32 29 cm.
b) Calculaţi sinusul unghiului dintre dreptele A ' C ' şi AN .
c) Arătaţi că A ' , C şi centrul de greutate al VDBC ' sunt coliniare.

Prof. Daniela Tilincă, Brăila

a , numărul 2007 2 + a 2
3. Să se arate că pentru orice număr natural este număr iraţional.
6
Prof. Dan Negulescu, Brăila

Clasa a IX a

1. Se consideră pătratul de latură 1. Se divide acest pătrat în nouă pătrate congruente şi se elimină pătratul
din mijloc. Se aplică acelaşi procedeu pentru fiecare din pătratele rămase. Dacă notăm cu S n , Pn aria şi
respectiv perimetrul figurii obţinute după n paşi, să se determine câte o relaţie de recurenţă pentru S n şi
Pn . Exprimaţi pe S n şi Pn în funcţie de n .

Prof. Mircea Ganga, Ploieşti


a n + bn + cn + ( a + b + c ) = ( a + b ) + ( b + c ) + ( c + a ) ,
n n n n
2. Să se determine n ∈ ¥ pentru care

oricare ar fi a, b, c ∈ ( 0, ∞ ) .
Prof. Dan Negulescu, Brăila

3. Fie x1 , x2 ,..., xn ∈ [ 0, ∞ ) , n ≥ 2, n ∈ ¥ , x1 + x2 + ...xn = n,


E ( x1 , x2 ,..., xn ) = [ x1 ] + { x2 } + [ x2 ] + { x3} + ... + [ xn− 1 ] + { xn } + [ xn ] + { x1} ,

unde [ x ] , { x} sunt notaţii obişnuite. Se cere:


a) max { E ( x1 , x2 ,..., xn ) / xi ∈ [ 0, ∞ ) ,1 ≤ i ≤ n}
b) Să se determine x1 , x2 ,..., xn pentru care este atinsă valoarea maximă.

Prof. Gheorghe Alexe, Brăila

Clasa a X a

1. Fie x, y , z > 1, xyz = 2 . Să se arate că log xy ( x + y ) + log yz ( y + z ) + log zx ( z + x ) ≥ 6 .

Prof. Mircea Ganga, Ploieşti

3
2. Să se arate că într-un triunghi echilateral de latură 1 nu se pot înscrie două paralelograme de arie ,
8
3
fără să aibă cel puţin un punct comun. Dar dacă aria fiecăruia este ?
18
Prof. Mircea Ganga, Ploieşti

3. Fie V ABC oarecare ascuţitunghic. Demonstraţi că avem relaţia:


 cos A cos B cos C  A B C
 2
+ 2
+ 2  ⋅ 4sin ⋅ sin ⋅ sin ≥ 1 .
 sin A sin B sin C  2 2 2

Prof. Viorel Botea, Brăila

Clasa a XI a

an
1. Fie ( an ) n≥ 1 un şir strict crescător de numere reale strict pozitive cu proprietatea că lim = a∈ ¡ *.
n→ ∞ n
1 + 2a2 + ... + n an
a) Să se calculeze lim .
( n + 1)
an
n→ ∞

a1 + a2 2 + ... + an n
b) Dacă, în plus, ( an ) n≥ 1 este şi progresie aritmetică, calculaţi lim .
n→ ∞ an + 1n

Prof. Ion Nedelcu, Ploieşti


1 1 1 
2. a) Calculaţilim  + 2 + ... + k  ; n ∈ ¥ * , n ≥ 2 fixat.
k→ ∞
n n n 
b) Fie x ∈ ( 0,1) \ ¤ fixat. Demonstraţi că există şi sunt unice numerele naturale n = n ( x ) ≥ 2 şi
1 1 1 1 1 1 1
k = k ( x ) ≥ 1 astfel încât + 2 + ... + k < x < + 2 + ... + k + k + 1 .
n n n n n n n
Prof. Radu Vasile, Brăila

3. Fie A ∈ M 3 ( £ ) , cu proprietăţile: Tr ( A2 ) = 0 şi ∃ X ∈ M 3 ( £ ) , astfel încât AX − XA = A .


a) Demonstraţi că An X − XAn = n ⋅ An , ∀ n ≥ 1 .
b) Calculaţi An , n ≥ 3 .

Prof. Carmen Botea, Brăila

Clasa a XII a

  a b *   a a *
1. Fie M =  A ( a, b ) =   ; a, b ∈ ¡ +  şi G ⊂ M , G =    ; a∈ ¡ +  .
  a b    a a 
a) Să se arate că ( M , ⋅ ) nu este grup.

b) Demonstraţi că ( G, ⋅ ) este grup izomorf cu ( ¡ * ,⋅ ) .


+

c) Să se arate că există o lege de compoziţie " Τ " astfel încât ( M , Τ ) este grup.

Prof. Ion Nedelcu, Ploieşti


Prof. Dan Negulescu, Brăila

e nx
2. Să se stabilească o relaţie de recurenţă pentru integralele In = ∫ dx , x ∈ ¡ , n ∈ ¥ * şi să
(e + 1)
x n

se calculeze I1 , I 2 , I 3 .
(∗ ∗ ∗ )

3. Fie f : ¡ → ¡ o funcţie continuă care admite o primitivă F şi există xlim F ( x ) = lim F ( x ) .


→ −∞ x→ + ∞

Arătaţi că există x0 ∈ ¡ astfel încât f ( x0 ) = 0 .

Prof. Florin Rotaru, Focşani


SOLUŢII:

Clasa a V a

1. Din teorema împărţirii cu rest obţinem a = 2007c1 + 20 respectiv a = 4014c2 + 23 = 2007 ⋅ 2c2 + 23
ceea ce este cotradictoriu. Deci nu există a ∈ ¥ care să îndeplinească condiţiile problemei şi nu avem cum
să aflăm ultima cifră.

2. a) 1 + 3 + 5 + ... + 2007 = 10042 deci suma este pătrat perfect.


b) Presupunem că numerele sunt cel mult egale cu 79. Numerele fiind distincte, impare suma lor este
1 + 3 + 5 + ... + 79 = 402 = 1600 < 1602 deci cel puţin unul dintre numere trebuie mărit cu 2. Dacă le
mărim pe cele mai mici decât 79 două devin egale deci 79 + 2 = 81 , adică cel puţin unul dintre numere
este mai mare decât 80.

( ) ( ) ( )
2 2
3. a) 2006 2005 = 20062004 ⋅ 2006 = 20061002 ⋅ 402 + 152 + 92 + 82 + 62 = 20061002 ⋅ 40 +

( ⋅ 15 ) + ( 2006 ⋅ 9 ) + ( 2006 ⋅ 8 ) + ( 2006 ⋅ 6 ) .


2 2 2 2
+ 20061002 1002 1002 1002

= 2006 ⋅ 2006 = ( 2006 ) ⋅ ( 35 + 20 + 15 + 10 + 6 + 4 + 2 ) = ( 2006 ⋅ 35 ) +


2 2
b) 20062005 2004 1002 2 2 2 2 2 2 2 1002

( ⋅ 20 ) + ( 2006 ⋅ 15 ) + ( 2006 ⋅ 10 ) + ( 2006 ⋅ 6 ) + ( 2006 ⋅ 4 ) + ( 2006 ⋅ 2 ) .


2 2 2 2 2 2
+ 20061002 1002 1002 1002 1002 1002

Clasa a VI a

1. Fie x, y , z ∈ ¥ , 2 x − 3 y − 10 z = 0 ⇒ 3 y = 2 x − 10 z ⇒ 3 y = 2 ( x − 5 z ) . Cum

( 3, 2 ) = 1 ⇒ y ⋮2 ;
2 x + 2 z − 3 y − 12 z = 0 ⇒ 2 ( x + z ) = 3 ( y + 4 z ) ⇒ ( x + z ) ⋮3
2 x + 2 y = 5 y + 10 z ⇒ 2 ( x + y ) = 5 ( y + 2 z ) ⇒ ( x + y ) ⋮5 deoarece ( 2,5 ) = 1 . Deci
y ( x + y ) ( x + z ) ⋮30.

2. Dacă y= 0 4 x = 2 iar pentru


Object 816
⇒ 4 x = 6 şi nu avem soluţie x ∈ ¥ . La fel pentru x = 0 .
Object 818

Presupunem y ≥ 2 şi ultimele două cifre ale lui 5 y sunt 25 4 x = ...26 . Cum x ≥ 1 şi 26 nu Object 825

se divide cu 4 rezultă că ..26 = 5 + 1 nu se divide cu 4. Deci ecuaţia nu are soluţii în ¥ 2 .


y

3. Dacă ( OM este în interiorul R AOP avem


 R AOM   R BOP 
m ( R XOY ) = m ( R XOM ) + m ( R MOP ) + m ( R POY ) = m   + m ( R MOP ) + m  =
 2   2 
m ( R AOM ) + m ( R MOP ) + m ( R MOP ) + m ( R BOP ) 180O
= = = 90O.
2 2
Dacă ( OM este în interiorul R AOM avem

 R AOM  m ( R POB ) − m ( R MOP )


m ( R XOY ) = m ( R XOM ) + m ( R MOY ) = m  + =
 2  2
m ( R AOM ) − m ( R MOP ) + m ( R MOC ) 180O − m ( R MOP )
= = < 90O.
2 2
Clasa a VII a

a1 + a2 + ... + a7
1. = 40 ⇒ a1 + a2 ... + a7 = 280 ;
7
a1 a2 a7 a1 + a2 + ... + a7 280 a a a
= = ... = = = = 9 ⇒ 1 = 2 = ... = 7 = 9 ⇔
1,1( 1) 2, 2 ( 2 ) 7, 7 ( 7 ) 1,1( 1) + 2, 2 ( 2 ) + ... + 7, 7 ( 7 ) 5 ⋅ 7 ⋅ 8 1
1
2
2
7
7
9 9 9 9
a a a c + c + ... + c7
⇔ 1 = 2 = ... = 7 = 9 ⇔ a1 = 10, a2 = 20,...,a7 = 70. 1 2 = 105 ⇒ c1 + c2 + ... + c7 = 735.
10 20 70 7
9 9 9
c1 c2 c7 c + c + ... + c7
Atunci = = ... = = 1 2
1,1( 1) 1,1( 1) 1,1( 1) S şi obţinem că
1,1( 1) ⋅ 2, 2 ( 2 ) 2, 2 ( 2 ) ⋅ 3,3 ( 3) 7, 7 ( 7 ) ⋅ 8,8 ( 8 )

10 81  1 1  9 1  63
S= ⋅ ⋅ + ... + =  1−  = ⇒ c1 = 420, c2 = 140, c3 = 70, c4 = 42, c5 = 28,
9 100  1 ⋅ 2 7 ⋅ 8  10  8  80
c6 = 20, c7 = 15
.

2. a) Fie M mijlocul lui [ CD ] , atunci avem următoarele cazuri:


CD CD
1. A şi B se află de aceeaşi parte a dreptei CD şi atunci AB ≤ AM + BM = + = CD.
2 2
CD CD
2. A şi B se află de o parte şi de alta a dreptei CD şi atunci AB ≤ AM + BM = + = CD.
2 2
b) Dacă suntem în cazul 1. atunci AB < CD . În cazul 2. avem: ( AB ) ≡ ( CD ) ⇔ M , A, B coliniare,

şi cum ( MA) ≡ ( MB ) ≡ ( MC ) ≡ ( MD ) ⇔ ABCD este dreptunghi.

3. a)
 ( EB ) ≡ ( FC )
 · ·
≡ FDC 
VEBC ≡ VFCD  ( BC ) ≡ ( CD ) ⇒
µ µ
ECB
· ·
dar DFC + CDF = 90 
o
· · ·
 ⇒ DFC + ECB = 90 ⇒ m DMC = 90 ⇒
o o
( )
 B = C = 90
o

⇒ EC ⊥ DF . ( EC ∩ DF = { M } ) .
P LD P EC 
b)Fie (AL ) ≡ ( AE ) ⇒ ( LE ) ≡ ( DC ) ⇒ LECD paralelogram ⇒
⇒ LEMD trapez
¶ = 90o 
M 
dreptunghic. Fie AN ⊥ DM ⇒ ( AN ) linie mijlocie în LEDM ⇒ N mijlocul lui ( DM ) ⇒

V ADM isoscel ⇒ ( AD ) ≡ ( AM ) .
Clasa a VIII a
a+ b
1. Folosim inegalitatea mediilor: ab ≤ şi obţinem
2
x ( y + z ) 3 x + y + z y ( x + z ) 3 y + x + z z ( x + y ) 3z + x + y
≤ , ≤ , ≤
3 6 3 6 3 6
x( y + z) y ( x + z) z ( x + y) 5( x + y + z)
Adunând cele trei relaţii obţinem + + ≤ .
3 3 3 6

Egalitate are loc dacă: 3 x = y + z şi 3 y = x + z şi 3z = x + y . Atunci obţinem


3( x + y + z ) = 2 ( x + y + z ) ⇒ x + y + z = 0 ⇒ x = y = z = 0

2. a) Notăm cu x muchia cubului şi calculăm laturile V AMN .Obţinem AN = x 5 ; MN = x 6


2 2
3x 2
x 29
respectiv AM = şi A [ V AMN ] = ⇒ x = 16cm.
2 8
b) Unghiul dintre dreptele A ' C ' şi AN este unghiul dintre AC şi AN . Exprimând aria triunghiului
1
V ANC în două moduri, obţinem sin ( R CAN ) = .
10
c) [ CBDC '] este piramidă triunghiulară regulată ⇒ C ' O ⊥ ( C ' DB ) , unde O este centrul de

greutate al triunghiului VC ' DB . A ' BDC ' tetraedru regulat ⇒ A ' O ⊥ ( C ' DB ) ⇒ A, O, C '
coliniare.

2007 2 + a 2 6 ( 2007 2 + a 2 )
3. = ∈¤ ⇔ 6 ( 2007 2 + a 2 ) ∈ ¤
6 6

Deoarece 6 ( 2007 2
+ a 2 ) ∈ ¥ obţinem 6 ( 2007 2 + a 2 ) ∈ ¥ ⇔ 6 ( 20072 + a 2 ) = k 2 , k ∈ ¥ * ⇒
⇒ 2007 2 + a 2 = 6k12 , k1 ∈ ¥ * ⇒ 3 | a ⇒ 9 | 6k12 ⇒ 6692 + a12 = 6k22 ⇒ 2232 + a22 = 6k32 ⇒
( M 6 + 1) + ( M 6 ± 1) = M 6 ⇒ 6 2, absurd.
2 2
⇒ a2 impar şi 3 a2 ⇒ a2 = 6m ± 1 ⇒

Clasa a IX a

1. Avem imediat :
2
1 8 64  8  1 1
S1 = 1 − = , S2 = =   , P1 = 4 + 4 ⋅ , P2 = P1 + 4 ⋅ 2 ⋅ 8 .
9 9 81  9  3 3
Prin inducţie matematică obţinem că
n− 1
8 4  8
S n = ⋅ S n − 1 , Pn = Pn − 1 + ⋅   ,( ∀ ) n ∈ ¥ ∗ ,
9 3  3
4  
n n
 8 8
iar de aici tot prin inducţie matematică găsim că S n =   , Pn = 4 + ⋅    − 1.
 9 5   3 
2. Luăm a = b = c = 1 şi obţinem 3 + 3n = 3 ⋅ 2n cu n ∈ ¥ . 0 nu este soluţie iar 1 şi 2 sunt soluţii. .
Pentru n ≥ 3 demonstrăm că 1 + 3n − 1 > 2n prin inducţie matematică.
Pentru n = 3 revine la 10 > 8, adevărat. Dacă inegalitatea este adevărată pentru n ≥ 3 atunci
2 < 2 + 2 ⋅ 3n− 1 < 1 + 3 ⋅ 3n− 1 = 1 + 3n şi deci inegalitatea este demonstrată pentru n + 1 . Deci
n+ 1

singurele soluţii sunt 1 şi 2 pentru care se verifică imediat egalitatea din enunţ pentru orice a, b, c > 0.

3. a) Aplicăm inegalitatea Cauchy-Buniakovski-Schwartz şi obţinem :

( [ x1 ] + { x2 } + ... + 1⋅ [ xn ] + { x1} ) ≤ ( 12 + ... + 12 ) ( [ x1 ] + { x2 } + ... + [ xn ] + { x1} ) =


2
1⋅
= n ( x1 + x2 + ... + xn ) = n ⋅ 1 = n
Valoarea maximă a lui E este n şi se obţine , de exemplu pentru xk = 1.
b) Egalitatea în inegalitatea Cauchy are loc dacă şi numai dacă

[ x1 ] + { x2 } = [ x2 ] + { x3} = ... =
[ xn ] + { x1} = [ x1 ] + { x2 } + [ x2 ] + { x3} + ... + [ xn ] + { x1} =
x1 + x2 + ... + xn
1 1 1 1 + 1 + ... + 1 n
 [ x1 ] + { x2 } = 1  [ x1 ] = 1, { x2 } = 0
 
n  [ x2 ] + { x3 } = 1  [ x2 ] = 1, { x3 } = 0
= = 1⇔  ⇔  de unde obţinem x1 = x2 = ... = xn = 1.
n  ......................  ............................
 [ x ] + { x } = 1  [ x ] = 1, { x } = 0
 n 1  n 1

Clasa a X a

x+ y 4
1. Avem inegalitatea: ≥ pentru x, y > 0
xy x+ y
Logaritmând această inegalitate obţinem
log xy ( x + y ) − 1 ≥ log xy 4 − log xy ( x + y ) ⇔ 2 log xy ( x + y ) ≥ 1 + 2 log xy 2 .
Scriem inegalităţile analoage şi adunăm cele trei inegalităţi:
1 log 2 xyz log 2 x 3
2∑ log xy ( x + y ) ≥ 3 + 2∑ = 3 + 2∑ = 3 + 2∑ ≥ 9 + 2 ⋅ = 12
log 2 xy log 2 xy log 2 yz 2
a 3
deoarece pentru a, b, c > 0 avem ∑ ≥
b + c 2.
2. Avem, folosind formula cunoscută a ariei paralelogramului că
3 3 3
S AMNP = ( 1 − x ) ⋅ x ⋅
= − x2 + x , x ∈ ( 0,1) .
2 2 2
3 3 1
Considerăm funcţia de gradul al II-lea f ( x ) = − x 2 + x . Observăm că xmax = atunci
2 2 2
3 1
max ( S AMNP ) = . Centrul de greutate aparţine acestui paralelogram, pentru care x = , deci, două
8 2
3
paralelograme de arie , vor avea cel puţin punctul G în comun.
8
Pentru partea a doua, vom poziţiona paralelogramele faţă de o paralelă la latura triunghiului care
trece prin G. Dacă paralelogramul este situat deasupra acestei drepte, aria sa va fi
2  x 3 x2 3 x 3  2 1 3
S =  − x = − + ,x∈  0,  şi este maximă pentru xmax = şi S = .
3  2 2 2  3 3 max 18
Obţinem că paralele au în comun punctul G. Dacă paralelogramul este „ sub ” paralelă, atunci
3 1  1
S = ( 1− x) ⋅ x ⋅ şi max se obţine pentru x = , nu convine deoarece x ∈  0,  .
2 2  3
3. Avem:
A B C A B C r
r = 4 R sin sin sin ⇔ 4sin sin sin = şi
2 2 2 2 2 2 R

r cos A b 2 + c 2 − a 2 2 r b2 + c 2 − a 2
⋅ = ⋅ 4 R ⋅ = ⋅ 2 Rr .
R sin 2 A 2bca 2 R a 2bc

abc S
Deoarece = R şi r = obţinem
4S p
b2 + c2 − a 2 b 2 + c 2 − a 2 2r b 2 + c 2 − a 2 1  b2 c 2 
2
⋅ 2 Rr = ⋅ = =  + − a
a bc a 4S 2 pa 2p  a a 
şi deci trebuie să demonstrăm că
1  b2 c 2 c2 a2 a 2 b2 
 + − a + + − b + + − c  ≥ 1.
2p  a a b b c c 
b2 a 2
≥ a + b ⇔ b3 + a 3 ≥ ab ( a + b ) ⇔ ( a − b ) ≥ 0 şi inegalităţile analoage:
2
Avem +
a b
c2 a2 b2 c2
+ ≥ a + c, + ≥ b + c,
a c c b
1  b2 b2  a + b + c
+ ... + − c ≥ = 1. Egalitatea are loc pentru a = b = c.
2 p  a
pe care le adunăm şi rezultă
c  2p

Clasa a XI a

xn = 1 + 2a2 + ... + n an şi yn = ( n + 1) n , n ∈ ¥ * . Avem că ( yn ) n ≥ 1 este strict crescător şi


a
1. a) Fie

( n + 1)
an+ 1
x − x 1 1
lim n + 1 n = lim = lim =
n + 1 − yn ( n + 2 ) n+ 1 − ( n + 1) n n→ ∞  n + 2  n+ 1 − ( n + 1) a1 − an+ 1 ea
a a a
nemărginit şi n→ ∞ y n→ ∞

 
 n+ 1
an+ 1 an+ 1
 1  lim
( n + 1)
an − an+ 1
deoarece lim  1 +  = e n→ ∞ n + 1 = e a , iar an − an + 1 < 0 ⇒ lim = 0 . Deci există,
n→ ∞
 n + 1 n→ ∞

xn 1
conform criteriului Cesaro-Stolz, lim = .
n→ ∞ yn e a
b) Fie xn = a1 + a22 + ... + ann şi yn = ann+ 1 , n ∈ ¥ * , cu şirul ( yn ) n ≥ 1 strict crescător şi nemărginit, iar
xn + 1 − xn an+ 1 1 1
lim = lim n + 1 n + 1 n = lim +
= 1
n→ ∞ yn + 1 − yn n → ∞ an + 1 − an + 1 n → ∞  a  n 1
−1
e , deoarece lim = 0 şi
 − an + 1
n+ 2 n→ ∞ a
 n+ 1
 an + 1 
n+ 1 n+ 1 a ( n + 1)
 a −a   a  lim
lim  1 + n + 2 n + 1  = lim  1 +  = e n→ ∞ an+ 1
= e.
n→ ∞
 an + 1  n→ ∞
 an + 1 

k
 1
1−   k→ ∞
1 1 1 1 1 n− 1 1 .
+ ... + k = ⋅   → ⋅
2. a) + n
=
n n 2
n n 1− 1 n n n− 1
n
 1 1 
b) ( 0,1) = ∪  n , n − 1  ⇒ ∃ !n ∈ ¥ , n ≥ 2 astfel încât x ∈  ,
1 1 
n≥ 2    , ( x ∉ ¤ ) . Presupunem
n∈ ¥  n n − 1 
1 1 1 1
prin absurd că x≥ + 2 + ... + k + 1 , ∀ k ≥ 1 . Trecând la limită după k, rezultă că x ≥ (fals).
n n n n− 1
1 1 1
Deci, există k ∈ ¥ * astfel încât x < + 2 + ... + k + 1 , ∀ k ≥ 1 . (1)
n n n
1 1 1
Fie k „ primul ”care verifică relaţia (1) ⇒ x > + 2 + ... + k ( x ∉ ¤ ) .
n n n

3. a) Demonstrăm prin inducţie că An X − XAn = nAn , ∀ n ≥ 1 .


Pentru n = 1 este adevărat. Presupunem P ( n ) adevărată şi demonstrăm P ( n + 1) adevărată.

Demonstrăm că An + 1 X − XAn + 1 = ( n + 1) An + 1 . Avem An X − XAn = nAn | ⋅ A (la stânga) ⇒


⇒ An + 1 X − AXAn = nAn + 1 şi rămâne să demonstrăm că
XAn + 1 + ( n + 1) An + 1 = AXAn + nAn + 1 ⇔ XAn + 1 + An + 1 = AXAn ⇔ ( XA − AX − A ) An = O3
(adevărat) ⇒ P ( n ) adevărată, ∀ n ≥ 1 .
b) Folosind An X − XAn = nAn ⇔ ( A X − nA ) =
n n
XAn ⇔ An ( X + nI 2 ) = XAn ⇒
( det A) ⋅  det ( X + nI 2 ) − det X  = 0 .
n

Presupunem că det A ≠ 0 ⇒ det ( X + nI 2 ) = det X , ∀ n ∈ ¥ * , deci ecuaţia de gradul al III-
lea det X = det ( X + zI 2 ) în necunoscuta z are o infinitate de rădăcini distincte (fals).
(Sau prin calcul direct:
a b c
 
X= d e f  ⇒ n3 + n 2 ( e + i ) + n ( ae + ai − cg − fh − bd ) = 0, ∀ n ≥ 1 , imposibil)
g h i 

1
Deci det A = 0; S ( A ) =  Tr 2 ( A ) − Tr ( A2 )  = 0 , deoarece
2 
Tr ( A) = Tr ( AX − XA ) = 0 .
Folosind relaţia lui Cayley-Hamilton obţinem:
A3 − Tr ( A ) ⋅ A2 + S ( A ) ⋅ A − det A ⋅ I 3 = O3 ⇒ A3 = O3 ⇒ An = O3 , ∀ n ≥ 3.

Clasa a XII a

1. a) A ( a, b ) ⋅ A ( c, d ) = A ( ac + bc; ad + bd ) ∈ M .
Fie A ( e1 , e2 ) elementul neutru ⇒

 ae + be1 = a
A ( a, b ) ⋅ A ( e1 , e2 ) = A ( a, b ) ⇒  1 , ∀ a, b > 0 . Ceea ce este fals : de exemplu, dacă
 ae2 + be2 = b
1
a = b = 1 ⇒ e1 = care nu verifică pentru a = 1 şi b = 2 . Deci, ( M , ⋅ ) nu este grup.
2
 1 1
b) Înmulţirea indusă pe G este asociativă şi comutativă. De la punctul a) rezultă că A  ,  ∈ G este
 2 2
element neutru.
A ' ( a, a ) ⋅ A ( a ', a ') este„inversul ” lui A ( a, a ) dacă
 1 1 1
A ( a, a ) ⋅ A ( a ', a ') = A  ,  ⇔ a ' = > 0,
 2 2 4a
deci ( G , ⋅ ) este grup abelian.

Funcţia f :G → ¡ *
+ , definită prin f ( A ( a, a ) ) = 2a este evident bijectivă şi
f ( A ( a, a ) ) ⋅ A ( b, b ) = f ( A ( 2ab, 2ab ) ) = 4ab = f ( A ( a, a ) ) ⋅ f ( A ( b, b ) ) .
c) Definim pe M legea A ( a, b ) Τ A ( c, d ) = A ( ac, bd ) . Asociativitatea este evidentă. Elementul
 1 1
neutru este A ( 1,1) ∈ M . Orice A ( a, b ) are „inversul ” A ' ( a, b ) = A  ,  ∈ M .
 a b

dt
2. Facem schimbarea de variabilă e x + 1 = t ; t > 1 ⇔ e x = t − 1 ⇔ x = ln ( t − 1) ⇒ dx = .
t−1
( t − 1) ( t − 1) ( t − 1)
n n− 1 n
n ' 1
n

Jn = ∫
t n
⋅∫
dt
t− 1
=
t n
⋅ dt =
1
n ∫ ( t − 1) ( t n ) 1
n
n 1
dt = ( t − 1) n +
t ∫ t n+ 1
1  t − 1
dt = 
n t 
 + J n+ 1 ,
nx
1 e
n (
n ≥ 1 . Obţinem I n + 1 = I n − n ⋅ x , ∀ ) n ≥ 1.
( e + 1)
ex ex ex
Avem I1
∫ ex + 1
= dx = ln ( e x
+ 1) + C , I 2 = I 1 −
ex + 1
= ln ( e x
+ 1) −
ex + 1
+ C şi

1 e2 x
I3 = I 2 −
2 ( e x + 1) 2 .

3. Fie funcţia g ( x ) = F ( x ) − F ( x + 1) ; x ∈ ¡ . Funcţia g este derivabilă, în particular continuă.


Dacă g ( x ) ≠ 0, ∀ x ∈ ¡ , atunci g ( x ) > 0, ∀ x ∈ ¡ sau g ( x ) < 0, ∀ x ∈ ¡ .
Dacă g ( x ) > 0, ∀ x ∈ ¡ ⇒ F ( x ) > F ( x + 1) , ∀ x ∈ ¡ .
Obţinem F ( n ) < F ( 1) < F ( 0 ) < F ( − n ) , ∀ n ∈ ¥ , n ≥ 2 . De aici, prin trecere la limită, rezultă
lim F ( n ) = L ≤ F ( 1) < F ( 0 ) ≤ L = lim F ( − n ) (fals). Atunci există a∈ ¡ astfel încât
n→ ∞ n→ ∞

g ( a ) = 0 ⇔ F ( a ) = F ( a + 1) . Aplicând teorema lui Rolle pe intervalul [ a, a + 1] , rezultă că


∃ x0 ∈ ( a, a + 1) astfel încât F ' ( x0 ) = f ( x0 ) = 0 . Analog dacă g ( x ) < 0, ∀ x ∈ ¡ .

CONCURSUL INTERJUDEŢEAN DE MATEMATICĂ


“ PETRU MOROŞAN -TRIDENT ”
Ediţia a V-a , Secţiunea B (M 2 ), Brăila, 23-25.11.2007

Clasa a IX a

 [ x] + { y} = 1,2
1. Să se rezolve sistemul de ecuaţii  , unde [ x ] reprezintă partea întreagă a numărului
 {x} + [ y ] = 3,3
real x, iar { x} reprezintă partea fracţionară a acestuia.

2. Fie mulţimea M= { ( x, y ) ∈ ¥ × ¥ / x 2
= y 2 + y + 51} .
a) Să se determine ( x, y ) ∈ M ştiind că x = 51.
b) Determinaţi elementele mulţimii M.

3. Se consideră pătratul cu latură 1. Se divide acest pătrat în alte nouă(9) pătrate egale şi se elimină pătratul
din mijloc.Se aplică acelaşi procedeu pentru fiecare din pătratele rămase. Dacă notăm cu S n şi Pn aria şi,
respectiv, perimetrul figurii rămase după n paşi, să se determine câte o relaţie de recurenţă pentru S n şi Pn
( S0 = 1, P0 = 4) . Exprimaţi pe S n şi Pn în funcţie de n.

Clasa a X a

1. Se consideră şirul ( xn ) n ≥ 1 , xn = 2+ 2 + ... + 2 , unde numărul radicalilor este n, şi şirul ( yn ) n ≥ 1


, definit prin yn = 2+ 2 + ... + 2 + 2 .
a) Să se calculeze y1 , y2 , y3 ,...., y2007 .
b) Să se arate că xn < y2007 , ( ∀ ) n ≥ 1.
1
c) Să se arate ca xn < ( x n − 1 + 6), (∀ )n ≥ 2 .
4
2. Fie numerele reale x, y , z cu proprietăţile: x, y , z > 1, xyz = 2 . Să se arate că:
log xy ( x + y ) + log yz ( y + z ) + log zx ( z + x ) ≥ 6 .

3. Considerăm funcţia f : ¡ → ¡ determinată din relaţia


f ( x 3 + 6 x 2 + 12 x) ≤ x ≤ f 3 ( x) + 6 f 2 ( x) + 12 f ( x)
a) Să se arate că funcţia f este inversabilă şi să se calculeze inversa ei.
b) Să se calculeze f ( ( 0,19 ) ) .
c) Să se rezolve ecuaţia f ( x) + 3
2 x + 15 = 0 .

Clasa a XI a

 a b
1. Fie A=   , A ∈ M 2 (¡ ). .
 c d
a) Arătaţi că A - (a + d ) A + ( ad - bc) I 2 = O2 . .
2

An = (a + d ) n -1 A, n ∈ ¥ , n ≥ 2.
b) Dacă A nu este inversabilă atunci
c) Dacă A2 = A , atunci a + d ∈ { 0,1, 2} .

  a b c 
   
2. Fie G =  X =  c a b  a, b, c ∈ ¡  .
  b c a 
   
a) Pentru a = b = 0 şi c = 1 arătaţi că X ∈ G este inversabilă şi calculaţi X − 1 .
b) Calculând det( X ), X ∈ G , în două moduri, demonstraţi egalitatea:
1
a 3 + b 3 + c 3 − 3abc = (a + b + c)[(a − b) 2 + (b − c) 2 + (c − a ) 2 ] .
2

3. Fie n ∈ ¥ * şi matricile A, B ∈ M n ( £ ) , cu proprietăţile:


a) ( A + B) 2 = A2 + B 2 .
b) ( A + B) 4 = A4 + B 4 .

( AB )
2
Să se arate că = On .

Clasa a XII a

1. Pe mulţimea £ se defineşte legea de compoziţie x ∗ y = xy + ax + ay + b, a, b ∈ £ . Determinaţi


a + b dacă în £ elementul neutru este 4.
x+ y 4
2. Pe ¡ se defineşte legea de compoziţie x ∗ y = xy − + şi
3 9
x∗n = 1 x ∗ 2x ∗4...4∗3 x , ( ∀ ) n ∈ ¥ ∗ .
x ∗ 44
de n ori x

a) Determinaţi numărul real a dacă (¡ \{a}, ∗ ) este grup abelian.


b) Rezolvaţi ecuaţia x∗6 + 2 x∗3 = 0 .

3. Determinaţi F (1) dacă


dx 1 2− 2
F ( x) = ∫ x x + 4x − 4
2
şi F ( 2) =
2
arccos
2
.

SOLUŢII:

Clasa a IX a

1. Adunând cele două ecuaţii se obţine: x + y = 4,5 . De asemenea, { y} = 1, 2 - [ x ] ∈ [ 0,1) ⇒ [ x ] = 1 .


Atunci { y} = y - [ y ] = 0, 2 . Cum [ x ] = 1 , avem x ∈ [ 1, 2 ) . Din x + y = 4,5 ⇒ y = 4,5 − x obţinem
2,5 < 4,5 - x = 3,5 ⇒ [ y] = 3 şi { x} = 0,3 . În concluzie x = 1,3 şi y = 3, 2 .

51 , egalitatea devine y 2 + y − 2550 = 0 . Soluţiile ecuaţiei sunt y1 = 50, y2 = -51 , dar


2. a) Cum x =
cum y ∈ ¥ rămâne doar soluţia y = 50 .
b) Avem din ipoteză că
x = y 2 + y + 51 ⇔ x 2 − y 2 − y − 51 =
2
0 /⋅ 4 ⇔ 4 x 2 − 4 y 2 − 4 y − 204 = 0 ⇔ 4 x 2 − (2 y + 1) 2 = 203 ⇔
⇔ (2 x − 2 y − 1)(2 x + 2 y + 1) = 203
Deoarece x, y ∈ ¥ avem că 2 x - 2 y -1 < 2 x + 2 y + 1 , şi cum 203 = 7 ⋅ 29 obţinem variantele:
 2x − 2 y − 1 = 1  2x − 2y − 1= 7  x = 51 x= 9
 şi  . Cu soluţiile:  şi 
 2 x + 2 y + 1 = 203  2x + 2 y + 1 = 29  y = 50 y= 5
3.


1

3

În figură am arătat primii doi paşi adică situaţiile particulare n = 1 şi n = 2 .


2
1 8 64  8  1 1
S1 = 1 − = , S2 = =   , P1 = 4 + 4 ⋅ , P2 = P1 + 4 ⋅ 2 ⋅ 8
9 9 81  9  3 3
n− 1
8 4  8
Prin inducţie matematică obţinem că S n = ⋅ S n − 1 , Pn = Pn − 1 + ⋅   ,( ∀ ) n ∈ ¥ ∗ .
9 3  3
4  8 
n n
 8
În final, tot prin inducţie matematică găsim că S n =   , Pn = 4 + ⋅    − 1 .
 9 5   3  
Clasa a X a

y1 = 2+ 2 = 2
1. a) . Se deduce uşor că y 2007 = 2 .
y2 = 2 + 2 = 2, y 3 = 2
b) Se demonstrează că xn < 2, ∀ n ∈ ¥ , n ≥ 1 .
1
c) Avem că xn = 2 + xn − 1 , n ∈ ¥ , n ≥ 2 . Înlocuind în inegalitatea xn < ( xn − 1 + 6) aceasta
4
1
devine echivalentă cu 2 + xn− 1 < ( x n − 1 + 6) . Prin prelucrare se ajunge la ( x n − 1 − 2) 2 > 0 ,
4
inegalitate adevarată deoarece x n < 2 (din b). Deci cazul de egalitate cu zero este exclus.
x+ y 4
2. Inegalitatea ≥ se logaritmează în baza xy şi se obţine :
x⋅ y x+ y
log xy ( x + y ) − 1 ≥ log xy 4 − log xy ( x + y ) ⇔ 2 log xy ( x + y ) ≥ 1 + 2 log xy 2 .
Se scriu şi analoagele acestei inegalităţi şi se adună. Folosind inegalitatea lui Nesbitt obţinem:
 1 1 1   log 2 xyz
2  log xy ( x + y ) + log yz ( y + z ) + log xz ( x + z )  ≥ 3 + 2  + +  = 3+ 2 +
 log 2 xy log 2 yz log 2 zx   log 2 xy
log 2 xyz log 2 xyz   log 2 z log 2 x log 2 y   log 2 z log 2 x
+ +  = 3 + 2 1+ + 1+ + 1+  = 9 + 2 + +
log 2 yz log 2 zx   log 2 xy log 2 yz log 2 zx   log 2 xy log 2 yz
log 2 y   log 2 z log 2 x log 2 y  3
+  = 9 + 2 + +  ≥ 9 + 2 ⋅ = 12.
log 2 zx   log 2 x + log 2 y log 2 z + log 2 y log 2 x + log 2 z  2
3. a) Inegalitatea din ipoteză , prin prelucrare, devine :
f ( ( x + 2 ) − 8 ) ≤ x ≤ ( f ( x) + 2 )
3 3
− 8 de unde se deduce că : f ( x ) = 3
x+ 8 − 2.
Funcţia f este strict crescătoare, deci injectivă. În aceste condiţii şi f ( ¡ )= ¡ , de unde f este
surjectivă. În concluzie, f este inversabilă (bijectivă) iar inversa lui f este:
f − 1 : ¡ → ¡ , f − 1 ( x) = ( x + 2 ) − 8 .
3

b) Cum f este strict crescătoare , obţinem f ( ( 0,19 ) ) = ( 0,1) .


c) Ecuaţia devine 3
x + 8 + 3 2 x + 15 = 2 şi x = -7 este soluţie unică a acestei ecuaţii, deoarece
funcţia f ( x) = 3
x + 8 + 3 2 x + 15 este strict crescătoare.

Clasa a XI a

1. a) Se verifică prin calcul direct


b) det A = 0 ⇒ ad - bc = 0 ⇒ A2 = (a + d ) ⋅ A . Prin inducţie matematică se demonstrează că:
An = (a + d ) n − 1 ⋅ A, ∀ n ∈ ¥ , n ≥ 2 .
 a 2 + bc = a  a 2 + bc = a
 
 b( a + d ) = b  b(a + d − 1) = 0
c) A = A ⇔ 
2
⇔  .
 c(a + d ) = c  c( a + d − 1) = 0
 d 2 + bc = d  d 2 + bc = d
 

Din ecuaţiile (2) şi (3) ale sistemului anterior rezultă că a + d = 1 sau b = c = 0 pentru a + d ≠ 1 .
 a 2 = a  a ∈ { 0,1}
Dacă b = c = 0 şi a + d ≠ 1 ⇒  2 ⇒  ⇒ a + d ∈ { 0, 2} , deci
 d = d  d ∈ { 0,1}
a + d ∈ { 0,1, 2} .
 0 0 1
 
2. a) Matricea X devine: X =  1 0 0  . Cum det X = 1 ≠ 0 , matricea X este inversabilă cu inversa
 0 1 0
 
 0 1 0
 
X −1 =  0 0 1
 1 0 0
 
b) det X = a 3 + b 3 + c 3 − 3abc (regula triunghiului) ( 1)
1 b c
1
det X = (a + b + c) 1 a b = (a + b + c)(a 2 + b 2 + c 2 − ab − ac − bc) = (a + b + c)  (a − b) 2 + (b − c) 2 +
2
1 c a
+ (c − a)2  ( 2 ) . Din (1) şi (2) rezultă identitatea cerută.
3. ( A + B )2 = A2 + B 2 ⇒ AB + BA = On .
( A + B ) 4 = A 4 + B 4 ⇒ ( A 2 + B 2 )( A 2 + B 2 ) = A 4 + B 4 ⇒ A 2 B 2 + B 2 A 2 = On .
Din AB + BA = On ⇒ A 2 B 2 + ABAB = On respectiv BABA + B 2 A 2 = On , de unde :
( AB) 2 + ( BA) 2 = On şi cum BA = - AB rezultă că 2( AB) 2 = On ⇒ ( AB) 2 = On .

Clasa a XII a

x ∗ 4 = x, ∀ x ∈ £ ⇔ 4 x + ax + 4a + b = x, ∀ x ∈ £ ⇔
1. Din definiţia elementului neutru avem
a+ 4= 1 a = −3
⇔ (a + 4) x + 4a + b = x, ∀ x ∈ £ ⇔  cu soluţia  . Deci a + b = 9 .
 4a + b = 0  b = 12

 1 1 1
x ∗ y =  x −   y −  + iar din existenţa elementului neutru rezultă
2. a) Legea se mai poate scrie
 3 3 3
 1  1 1  1  4 4
că  x −   e −  + = x, ( ∀ ) x ∈ ¡ ⇔  x −   e −  = 0, ( ∀ ) x ∈ ¡ , de unde e = .
 3  3 3  3  3 3
Din existenţa simetricului avem :
 1  1 1 4 1 1
 x '−   x −  + = ⇒ x '− = 1 1
 3  3 3 3 3 x − 1 , posibil doar dacă x ≠ şi deci a = .
3 3
3
2 n
 1 1  1 1
x =  x −  + şi inductiv obţinem : x∗n =  x −  + . Atunci ecuaţia
b) Din definiţie avem că 2

 3 3  3 3

6
1 1  1  1
3

devine :  x −  + + 2   x −  +  = 0 .
 3 3   3  3 
3 3
 1
Utilizând substituţia  x −  = t , avem t 2 + 2t + 1 = 0 cu t1 = t 2 = − 1 , deci,
 1
 x−  = − 1
 3  3
2
are soluţia reală x = − .
3
1 1
dx dx dx
x2 1 x2
3. Avem succesiv
F ( x) = ∫ 4 4 =∫
= ∫ =
x 2
1+ − 2  1 1  2 1 2 1  1 1
2

x x − 4 −  −  − − 
  x 2  2  2  x 2
'
 1 1 1 1
1  −  1 u' 1

− ∫  x 2 dx = − ∫ du = arccos x 2+C
.
2 1  1 1
2 2 1 2 1
− −  −u 2

2  x 2 2 2
1 1

1  1 1 1 2− 2 1
Cum F ( 2) = arccos 2  −  + C = arccos ⇒ C = 0 ⇒ F ( x) = arccos x 2
2  2 2 2 2 2 1
2
,
1
1 1 2 1 π π
F (1) = arccos 2 = arccos = ⋅ = .
2 1 2 2 2 4 8
2

CONCURSUL INTERJUDEŢEAN DE MATEMATICĂ


“VICTOR VÂLCOVICI”
Ediţia a - XVI - a,
Brăila, 5 aprilie 2008

Clasa a VII a

1. Fie x, y numere naturale mai mari sau egale ca 2. Demonstraţi că:


x 2 y 2 + 128 ( x + y ) ≥ 64 ( xy + 4 ) .

Prof. Ion Nedelcu, Ploieşti

2. Fie G centrul de greutate al V ABC . Prin G se duc două drepte oarecare, care taie AB şi AC în M şi N,
iar a doua în P, respectiv Q. Demonstraţi că:
BM 2 CN 2 BP 2 CQ 2
+ + + ≥ 1.
MA2 NA2 PA2 QA2

Prof. Dan Marinescu şi Prof. Ioan Şerdean, Hunedoara

3. Să se afle n ∈ ¥ nenul, minim, pentru care există a1 , a2 ,..., an numere naturale astfel încât
a12 + a22 + ... + an2 = 2004 .

Prof. Lucian Dragomir, Oţelu Roşu

Clasa a VIII a

1. Fie trunchiul de piramidă patrulateră regulată ABCDA ' B ' C ' D ' cu baza ABCD . Notăm

( ) (
α = m R ( ( ABB ') , ( ACC ') ) şi β = m R ( DD ', ( ABC ) ) . )
a) Demonstraţi că sin β ⋅ tgα = 1 .
b) Poate fi α = 30o ? Justificaţi răspunsul.
c) Presupunem β = 60o şi AC = 2 cm. Aflaţi volumul piramidei din care provine trunchiul.

Prof. Carmen şi Viorel Botea, Brăila

2. Să se afle x şi y numere reale pentru care avem: x4 + x2 − 2 x ( 2 + y ) + y 2 + 3 = 0 .

Prof. Dan Negulescu, Brăila

8
3. Fie a, b > 0, astfel încât a + b < . Demonstraţi că avem:
5
4a 2 + ( 3a − b − 2 ) ( b − a − 2 )  a b
≥ min  ,  .
4b + ( 3b − a − 2 ) ( a − b − 2 )
2
 b a

Prof. Carmen şi Viorel Botea, Brăila


Clasa a IX a
1. Se consideră două şiruri ( an ) n≥ 0 şi ( bn ) n ≥ 0 de numere naturale, primul formează o progresie
aritmetică, iar cel de-al doilea, o progresie geometrică. Se ştie că două şiruri au cel puţin două elemente
comune distincte. Să se arate că există o infinitate de elemente comune.

Prof. Laurenţiu Panaitopol, Bucureşti

2. Se consideră două puncte distincte B, C într-un plan.


a) Să se arate că pentru orice punct M din plan, B ≠ M , C ≠ M , numărul real
BC − MB − MC 2
2 2
α M =
2 MB ⋅ MC
este în intervalul [ − 1,1] .
b) Dacă , α ∈ [ − 1,1] să se determine locul geometric al punctelor M din plan cu proprietatea
α M =α .
Prof. univ. dr. doc. Nicu Boboc, Bucureşti

3. Să se arate că numărul real 3


10 + 3 − 3
10 − 3 este între 1 şi 2 şi nu este număr raţional.

Prof. univ. dr. doc. Nicu Boboc, Bucureşti

Clasa a X a

1. Fie z1 , z2 , z3 trei numere complexe care satisfac relaţiile:


z1 − z2 + 2 z3 = 2 z1
z2 − z3 + 2 z1 = 2 z2
z3 − z1 + 2 z2 = 2 z3
Să se arate că z1 = z2 = z3 .

Prof. Laurenţiu Panaitopol, Bucureşti

2. Să se arate că pentru orice două numere naturale k , n, cu 2 ≤ k < n are loc relaţia:
1 1 1 1
+ + ... + k <
( k + 1) ( k − 1)
k k k
k n

Prof. univ. dr. doc. Nicu Boboc, Bucureşti

3. Se consideră trei drepte paralele distincte a, b, c într-un plan.


a) Arătaţi că există un triunghi echilateral ABC, cu A ∈ a, B ∈ b, C ∈ c .
b) Determinaţi lungimea laturii triunghiului ABC ca funcţie de distanţele între dreptele a, b, c.

Prof. Soare Nicolae, Bucureşti


Clasa a XI a

1. Să se arate că există o infinitate de funcţii continue f : [ 0, ∞ ) → [ 0, ∞ ) , cu proprietăţile:


 f ( 1) = 1
( )
∗ 2 , ∀ x ∈ [ 0, ∞ ) .
 (
f f ( x ) ) (
= f ( x ) )
Să se arate că există o unică funcţie derivabilă f : [ 0, ∞ ) → [ 0, ∞ ) neconstantă cu proprietăţile ( ∗ ) .

Prof. univ. dr. doc. Nicu Boboc, Bucureşti

2. Se consideră polinomul P ( X ) = X 3 + a2 X 2 − a1 X + a0 , unde a0 , a1 , a2 sunt numere naturale cu


a0 ≠ 0 şi a1 > a02 + a2 a0 + 1 . Să se arate că P nu se poate scrie ca produsul a două polinoame cu
coeficienţii numere întregi, de grad mai mare sau egal ca 1.

Conf. dr. Grigore Gheorghe, Bucureşti

3. Se consideră două drepte paralele distincte d ', d '' într-un plan şi A, B, C trei puncte necoliniare situate
în plan între dreptele d ' şi d '' . Să se arate că există un punct M în plan, situat între dreptele d ' şi d ''
astfel încât A, B, C , M să fie vârfurile unui paralelogram.

Prof. univ. dr. doc. Nicu Boboc, Bucureşti

Clasa a XII a

1. Fie f : ( 0, ∞ ) → ¡ o funcţie reală integrabilă Riemann pe orice interval [ a, b] din ( 0, ∞ ) şi în plus cu


proprietatea f ( nx ) < f ( ( n + 1) x ) , ∀ n ∈ ¥ , x ∈ ( 0, ∞ ) , n ≥ 1 . Să se arate că f este strict crescătoare.

Conf. dr. Grigore Gheorghe, Bucureşti

2. Fie n ∈ ¥ , n ≥ 1 , M ( n ) inelul matricelor cu n linii şi n coloane de numere reale, iar I matricea


unitate din M ( n) .
a) Să se arate că dacă C ∈ M ( n ) este astfel încât matricea ( xI + C ) este inversabilă pentru orice
x ∈ ( 0, ∞ ) , atunci funcţia A : ( 0, ∞ ) → M ( n ) , dată de A( x) = ( xI + C )
−1
verifică relaţia:

A ( x ) = A ( y ) + ( y − x ) A ( x ) A ( y ) , ∀ x, y > 0 .
b) Să se arate că orice funcţie A : ( 0, ∞ ) → M ( n ) care verifică relaţia
A ( x ) = A ( y ) + ( y − x ) A ( x ) A ( y ) , ∀ x, y > 0 şi astfel încât există x0 > 0 cu proprietatea că
A ( x0 ) , este o matrice inversabilă de forma A( x) = ( xI + C ) , unde C este o matrice cu proprietatea că
−1

( xI + C ) este inversabilă pentru orice x > 0 .

Prof. univ. dr. doc. Nicu Boboc, Bucureşti

3. Fie M o mulţime formată din 3n puncte, n ∈ ¥ , n ≥ 1 , dintr-un plan. Să se arate că există n


dreptunghiuri disjuncte două câte două, situate în planul dat, astfel încât fiecare din ele să conţină exact trei
din punctele date.
Prof. dr. Bucur Gheorghe, Bucureşti
SOLUŢII:

Clasa a VII a

1. Avem x = 2 + ( x − 2 ) ≥ 2 2 ( x − 2 ) ⇒ x 2 ≥ 8 ( x − 2 ) . Analog, y 2 ≥ 8 ( y − 2 ) ⇒
⇒ x 2 y 2 ≥ 64 ( x − 2 ) ( y − 2 )
⇔ x 2 y 2 ≥ 64 xy − 128 ( x + y ) + 64 ⋅ 4 ⇔ x 2 y 2 + 128 ( x + y ) ≥ 64 ( xy + 4 )

MB GD 1 NC MB NC
2. a) Dacă MN P BC ⇒ = = = ⇒ + = 1 . Avem inegalitatea
MA AG 2 NA MA NA
2
 MB NC 
( ) , deci MB 2 NC 2  MA NA  1
2
a + b +
a 2 + b2 ≥
2 2
+ 2
≥   =
MA NA 2 2
BP CQ BM 2
CN 2
BP CQ 2 1 1
2
Analog se obţine că + = 1. Atunci + + + ≥ + = 1.
PA QA MA2 NA2 PA2 QA2 2 2

b) Dacă MN P BC , fie o paralelă prin A la BC care taie MN în { F } ; MN ∩ BC = { E} .


MB EB
VMBE : VMAF ⇒ =
MA AF
NC EC MB NC EB + EC EB + EB + 2 BD
VNCE : VNAF ⇒ = ⇒ + = = =
NA AF MA NA AF AF
2 EB + 2 BD 2 ED GD 1
= = = 2 = 2 ⋅ = 1 deoarece VEGD : V AGF .
AF AF AG 2
BP CQ BM 2 CN 2 BP 2 CQ 2 1 1
Analog se obţine că + = 1. Atunci + + + ≥ + = 1.
PA QA MA2 NA2 PA2 QA2 2 2

3. Se observă că n ≠ 1 deoarece 2004 nu este pătrat perfect. Arătăm că ecuaţia x 2 + y 2 = 2004 nu are
soluţii întregi .
Dacă am avea soluţii , x şi y ar trebui să aibă aceeaşi paritate .
a) pentru x = 2k + 1, y = 2m + 1, k , m întregi , înlocuim şi ajungem că 2002 e multiplu de 4, fals
b) pentru x = 2k , y = 2m se ajunge la k 2 + m 2 = 3 ⋅ 167 ( se arată , într-un fel sau altul , că nici
această ecuaţie nu are soluţii întregi ).Aşadar n ≠ 2
c) pentru n = 3 e suficient să observăm, de exemplu, că tripletul ( 2, 20, 40 ) e soluţie. Deci n = 3.

Clasa a VIII a

OB ⊥ AC 
1. a) Fie O şi O ' centrele bazelor. Avem  ⇒ OB ⊥ ( ACC ' A ') .
OB ⊥ OO '
Fie
OB ⊥ ( ACC ' A ' )  T . celor
OB
 ⇒ BE ⊥ AA ' ⇒ α = m ( R ( BE , OE ) ) = m ( R BEO ) ⇒ tgα = (1).
OE ⊥ AA ', E ∈ AA '  3 perpend . OE
Avem
m ( R ( DD ', ( ABCD ) ) ) = m ( R ( AA ',( ABCD ) ) ) = m ( R ( AA ', AO ) ) = m ( R A ' AO ) = m ( R EAO ) = β .

Din V AEO, m E ( ( ) )
µ = 90O , avem: sin β = EO (2). Din (1) şi (2) ⇒ tgα ⋅ sin β = OB ⋅ OE = 1
OA OE OA
, ceea ce trebuia demonstrat.
3
b) Dacă α = 30o ⇒ tgα ⋅ sin β = 1 ⇒ sin β ⋅= 1 ⇒ sin β = 3 > 1 fals.
3
c) Dacă β = 60o ⇒ m ( R A ' AC ) = 60 ⇒ VVAC este echilateral, unde V este vârful piramidei
o

AC 3 AB 2 ⋅ VO 2 3
mari ⇒ AB = 2 ; VO = = 3 . Deci, vol ( ABCD ) = = .
2 3 3
2.
( x − y)
2
x 4 + x 2 − 4 x − 2 xy + y 2 + 3 = 0 ⇔ + x4 − 4x + 3 = 0 ⇒ x4 − 4x + 3 ≤ 0 ⇒ x4 + 3 ≤ 4 x
.
Dar x
4
+ 3 = x 4 + 1 + 2 ≥ 2 x 2 + 2 = 2 ( x 2 + 1) ≥ 4 x , deci x 4 + 3 ≥ 4 x . Atunci rezultă egalitate, de
( − y + 1)
2
unde x = 1⇒ = 0 ⇒ y = 1.

3. Arătăm că numitorul fracţiei din membrul stâng este strict pozitiv:


4b 2 + ( 3b − a − 2 ) ( a − b − 2 ) > 0 ⇔ 4b 2 + 3ab − 3b 2 − 6b − a 2 + ab + 2a − 2a + 2b + 4 > 0

( b − 2) ( b − 2) + 4a ( b − 2 ) + 8a + 4a 2 − 5a 2 > 0
2 2
⇔ b 2 − 4b + 4 + 4ab − a 2 > 0 ⇔ − a 2 + 4ab > 0 ⇔
(b− 2+ 2a ) + a ( 8 − 5a ) > 0 .
2

8
Cum a< a+ b< ⇒ 8 − 5a > 0 , deci numitorul este strict pozitiv.
5
 a b b
I. Presupunem că a ≥ b > 0 ⇒ min  ,  = . Demonstrăm:
 b a a
4a 2 + 3ab − 3a 2 − 6a − b 2 + ab + 2b − 2b + 2a + 4 b a 2 + 4ab − 4a − b 2 + 4 b
≥ ⇔ ≥ ⇔
4b 2 + 3ab − 3b 2 − 6b − a 2 + ab + 2a − 2a + 2b + 4 a b 2 + 4ab − 4b − a 2 + 4 a
⇔ a 3 + 4a 2b − 4a 2 − ab 2 + 4a ≥ b3 − a 2b + 4ab 2 − 4b 2 + 4b ⇔ a 3 − b3 + 5a 2b − 5ab 2 + 4a − 4b ≥ 0 ⇔
⇔ ( a − b) ( a2 + ab + b 2 ) + 5ab ( a − b ) + 4 ( a − b ) − 4 ( a − b ) ( a + b ) ≥ 0 ⇔
⇔ ( a − b) ( a2 + ab + b 2 + 5ab + 4 − 4a − 4b ) ≥ 0
a 2 + b 2 + 6ab + 4 − 4 ( a + b ) = ( a + b ) + 4 − 4 ( a + b ) + 4ab = ( a + b − 2 ) + 4ab > 0 .
2 2
Avem

 a b a a 2 + 4ab − 4a − b2 + 4 a
⇒ min  ,  = . Demonstrăm că 2
II. Presupunem că a<b ≥ ⇔
 b a b b + 4ab − 4b − a 2 + 4 b
⇔ a 2b + 4ab 2 − 4ab − b3 + 4b ≥ ab 2 − a3 + 4a 2b − 4ab + 4a ⇔ 3ab ( b − a ) + 4 ( b − a ) + ( a 3 − b3 ) ≥ 0
( b − a ) ( 3ab + 4 − a 2 − b 2 − ab ) ≥ 0 ⇔ 4 − a 2 − b 2 + 2ab ≥ 0 ⇔ 4 > ( a − b ) ⇔
2

8
⇔ 4 > b − a = b − a ⇔ 2 > b − a ; avem b − a < a + b < < 2 , deci b − a < 2 , ceea ce trebuia
5
demonstrat.

Clasa a IX a
1. Fie r (respectiv q), raţia progresiei ( an ) n≥ 0 (respectiv a progresiei geometrice ( bn ) n ≥ 0 ). Vom arăta că r
şi q sunt numere naturale, r ≥ 1, q ≥ 2 .
Din relaţia an + 1 = an + r , ∀ n ∈ ¥ , deducem că r este un număr întreg. Nu putem avea r = 0
deoarece ar rezulta că şirul ( an ) n ≥ 0 este constant şi, deci, n-ar mai exista cel puţin doi termeni distincţi.

De asemenea, nu putem avea r < 0 , întrucât ar însemna şirul ( an ) n ≥ 0 să fie un şir descrescător de numere
naturale, contradicţie.
Întrucât ( bn ) n ≥ 0 are cel puţin doi termeni distincţi, din relaţia
bn = b0 ⋅ q n ; bn + 1 = bn ⋅ q, ∀ n ∈ ¥ deducem că bn > 0, ∀ n ∈ ¥ .
b1 p1
Din q = rezultă q este un număr raţional strict mai mare decât 2. Fie q = , p1 , p2 ∈ ¥ , cu
b0 p2
p1 , p2 prime între ele.
p1n
Din relaţia bn = b0 n , ∀ n ∈ ¥ , deducem că p2 ⋅ bn = p1 ⋅ b0 , ∀ n ∈ ¥ , ceea ce arată că orice
n n

p2
p '' al lui p2 este astfel încât ( p '') divide b0 , contradicţie. Aşadar, p2 = 1 şi,
n
divizor comun prim
deci, q este un număr natural.
Fie a,b doi termeni distincţi comuni celor două progresii. Există, deci, n, m ∈ ¥ cu b = a + rn = aq m ,
adică rn = a ( q m − 1) . De aici, deducem că pentru orice k ∈ ¥ avem a ( q km − 1) = rn ⋅ pk , unde pk

pk = 1 + q m + ( q m ) + ... + ( q m )
2 k−1
este numărul natural. . Punând nk = n ⋅ pk şi mk = k ⋅ m , avem

( )
r ⋅ nk = a q mk − 1 sau echivalent a + r ⋅ nk = a ⋅ q mk , ceea ce dovedeşte că termenii a + r ⋅ nk cu
k ∈ ¥ sunt comuni celor două progresii .

2. Fie B, C două puncte distincte într-un plan. Din teorema lui Pitagora generalizată deducem că pentru
orice punct M din plan, M ≠ B, M ≠ C , astfel încât M , B, C nu sunt coliniare, numărul α M este egal
cu cos ( θ M ) , unde θ M este măsura unghiului BMC.
Aşadar, în acest caz avem − 1< α < 1 . Dacă M , B, C sunt coliniare, atunci vom avea:
M

M este situat între B şi C ⇔ α M = 1 .


B este situat între M şi C atunci C este situat între B şi M ⇔ α M = − 1
Din cele precedente, deducem că dacă α ∈ ( − 1,1) , atunci locul geometric căutat va fi o reuniune
de două arce de cerc care sunt simetrice faţă de dreapta BC.
Dacă α = 1 , atunci locul geometric va fi segmentul BC (fără capetele B şi C), iar dacă α = − 1 , locul
geometric va fi format din cele două semidrepte ale dreptei BC, care se obţin după îndepărtarea
segmentului BC.
3. Fie x= 3
10 + 3 − 3
10 − 3 . Un calcul direct ne dă:
x3 = ( ) (
10 + 3 − 10 − 3 + 3 ) ( 3
10 + 3 ⋅ )( 3
10 − 3 )( 3
10 + 3 − 3
10 − 3 )
adică x verifică relaţia x3 = 6 − 3x .
Dacă x ar fi număr raţional, atunci ar exista p, q numere întregi, cu q > 0 , astfel încât p, q sunt prime între
p p p 3
ele şi x= . Deducem că 3 = 6 − 3 , adică p 3 = 6q 3 − 3 pq 2 .
q q q
p 3 şi deci 3 divide p, adică p = 3 p ', p ∈ ¥ . Deducem 9 ( p ') = 2q 3 − 3 p ' q 2 , şi
3
Rezultă că 3 divide
deci, 3 divide q, contradicţie. Aşadar, x nu este număr raţional.
Din definiţia lui x, rezultă x > 0 , iar din relaţia x 3 = 6 − 3 x , deducem x > 1 , întrucât în caz contrar (
5
0 < x ≤ 1 ), ar rezulta 6 − 3x = x 3 ≤ 1, 3 x ≥ 5, x ≥
, contradicţie.
3
De asemenea, avem x < 2 întrucât în caz contrar ( x ≥ 2 ), am deduce 6 − 3 x = x 3 ≥ 8, 3 x ≤ − 2 ,
contradicţie.

Clasa a X a

1. Punem z1 = a1 + ib1 , z2 = a2 + ib2 , z3 = a3 + ib3 , unde a1 , a2 , a3 , b1 , b2 , b3 sunt numere reale.


Relaţiile date devin:
( a1 − a2 + 2a3 ) + ( b1 − b2 + 2b3 ) = 4 ( a12 + b12 )
2 2

( a2 − a3 + 2a1 ) + ( b2 − b3 + 2b1 ) = 4 ( a22 + b22 )


2 2

( a3 − a1 + 2a2 ) + ( b3 − b1 + 2b2 ) = 4 ( a32 + b32 )


2 2

şi deci
( a1 − a2 + 2a3 ) + ( a2 − a3 + 2a1 ) + ( a3 − a1 + 2a2 ) + ( b1 − b2 + 2b3 ) + ( b2 − b3 + 2b1 ) + ( b3 − b1 + 2b2 ) =
2 2 2 2 2 2

= 4 ( a12 + a22 + a32 + b12 + b22 + b32 ) sau echivalent ( a1 − a2 ) + ( a2 − a3 ) + ( a3 − a1 ) + ( b1 − b2 ) + ( b2 − b3 ) +


2 2 2 2 2

( b3 − b1 ) + 4  ( a1 − a2 ) a3 + ( a2 − a3 ) a1 + ( a3 − a1 ) a2 + ( b1 − b2 ) b3 + ( b2 − b3 ) b1 + ( b3 − b1 ) b2  = 0
2

Rezultă că
0 = ( a1 − a2 ) + ( a2 − a3 ) + ( a3 − a1 ) + ( b1 − b2 ) + ( b2 − b3 ) + ( b3 − b1 ) , adică
2 2 2 2 2 2

a1 = a2 = a3 şi b1 = b2 = b3 , deci z1 = z2 = z3 .

l k − 1 − ( l − 1)
k−1
1 1
2. Fie k ∈ ¥ , 2 ≤ k < n şi l ∈ ¥ , cu k ≤ l ≤ n . Avem − = =
( l − 1) ( l − 1) l k − 1
k−1 k−1
lk−1

l k − 2 + l k − 3 ( l − 1) + ... + l ( l − 1) + ( l − 1) ( k − 1) ( l − 1) = ( k − 1) > ( k − 1)
k− 3 k− 2 k− 2

= > şi deci,
( l − 1) ( l − 1) l k − 1 ( l − 1) l k − 1 l k − 1
k−1 k−1 k−1
l
n 
n
1 1 1 1  1  1 1  1
∑l = k l k k − 1 ∑l = k  l − 1 k − 1 − l k − 1  = k − 1  k − 1 k − 1 − nk − 1  < k − 1 k
< ⋅
( )  ( )  ( )
3. Fie a, b, c trei drepte paralele diferite două câte două într-un plan. Fără a restrânge generalitatea, putem
presupune că b se află între a şi c. Se ia şi fie A ' ∈ b cu AA ' ⊥ a .
Object 1403
Se alege E un punct pe b astfel încât unghiul EAA ' să aibă măsură egală cu 60o . Se consideră D
mijlocul segmentului AE şi se alege C pe c cu proprietatea CD ⊥ AE .
Evident, C se poate afla fie de aceeaşi parte cu E faţă de dreapta AA ' , fie C nu se află de aceeaşi
parte cu E faţă de dreapta AA ' .
I. Primul caz – C se află de aceeaşi parte cu E faţă de dreapta AA ' .
Notăm cu B un punct de pe dreapta b cu AB = AC , astfel încât A ' se află între B şi E.
Vom arăta că unghiul CAB are măsură egală cu 60o şi deci, că V ABC este echilateral.
Într-adevăr, triunghiurile dreptunghice ACD şi AA ' B sunt congruente întrucât din faptul că
AE
unghiul AEA ' are 30o , atunci AA ' == AD şi AB = AC prin construcţie.
2
Deducem că unghiurile BAA ' şi CAD sunt egale, şi deci, cum unghiul A ' AD are 60o , rezultă
că unghiul CAB are de asemenea 60o .
CC ' 2β DE α
Punând AA ' = α , CC ' = β , avem CO = o
= , DO = o
= şi deci,
sin 60 3 tg30 3
2
 α 2β 
AC 2 = α 2 +  +  .
 3 3

II. Al doilea caz – C nu se află de aceeaşi parte cu E faţă de dreapta AA ' .


Luăm un punct B pe dreapta b astfel încât A ' este între B şi C şi cu AB = AC . Vom arăta că
unghiul BAC are 60o şi deci, V ABC este echilateral.
Într-adevăr, triunghiurile dreptunghice AA ' B şi CAD sunt congruente întrucât, din faptul că
AE
unghiul AEA ' are 30o , rezultă AA ' == AD şi AB = AC prin construcţie.
2
Deducem că unghiurile BAA ' şi CAD sunt egale şi deci, unghiul BAC are aceeaşi măsură cu
unghiul DAA ' adică 60o .

Clasa a XI a

1. Fie f : [ 0, ∞ ) → [ 0, ∞ ) , o funcţie continuă cu proprietăţile:


f ( 1) = 1 şi
f ( f ( x) ) = ( f ( x) ) , ∀ x ∈ [ 0, ∞ ) .
2

Notăm A= { f ( x ) x ∈ [ 0, ∞ ) } .Din ipoteză, 1∈ A şi luăm u ∈ A ⇒ f ( u ) = u2 .


Dacă f este neconstantă, atunci A va conţine cel puţin un punct u0 , cu u0 ≠ 1 .
De fapt, se va arăta că f este neconstantă şi derivabilă, atunci A = [ 0, ∞ ) , şi deci
f ( x ) = x , ∀ x ∈ [ 0, ∞ ) .
2

Să arătăm mai întâi că există o infinitate de funcţii continue f : [ 0, ∞ ) → [ 0, ∞ ) , cu proprietăţile (*).


Luăm a ∈ ( 0,1) şi considerăm funcţia f a : [ 0, ∞ ) → [ 0, ∞ ) , definită prin:
1
 a , dacă a ≤ x < a

1
f a ( x ) =  , dacă a ≤ x < 1
x
 x 2 , dacă 1 ≤ x < ∞

Se verifică simplu că f a este continuă şi că f a ( 1) = 1 . Avem, de asemenea:


2
 1  1
0 ≤ x < a ⇒ fa ( fa ( x ) ) ( f ( x) )
2
= fa   =   = a
 a  a
2
 1  1
a ≤ x < 1 ⇒ fa ( fa ( x ) ) = fa   =   = ( f ( x) )
2
a
 x  x
1 ≤ x < ∞ ⇒ fa ( fa ( x ) ) = fa ( x2 ) = ( x2 ) = ( f ( x) )
2 2
a

Evident, a ', a '' ∈ ( 0,1) ⇒ f a ' ≠ f a '' .


a '≠ a ''

Presupunem acum că f este neconstantă şi derivabilă întrucât există u0 ∈ A , cu u0 ≠ 1 , rezultă


că avem u0 > 1 sau u0 < 1 .
În primul caz ( u0 > 1 ), avem f ( u0 ) = u02 > u0 .
A conţine şirul crescător şi nemărginit (u )
2n
0 n∈ ¥
şi cum f este continuă, rezultă că A ⊃ [ u0 , ∞ ) .

Notăm {
α = inf f ( x ) x ∈ [ 0, ∞ )} .
Din ipoteză, α ≤ 1 . Nu putem α = 1 . Într-adevăr, în acest caz, punctul 1 ar fi punct de minim pentru
funcţia f şi deci, f ( 1) = 0 . Pe de altă parte, din u ≥ 1 ⇒ u ∈ A ⇒ f ( u ) = u . Deducem f ( 1) = 2
2

, contradicţie.
Aşadar, dacă f este neconstantă şi derivabilă, vom avea α < 1 . Există deci v0 ∈ A , cu v0 < 1 şi
cu v = f ( v0 ) < v0 < 1 . Prin urmare, A va conţine şirul descrescător ( v02 n ) n∈ ¥ , care converge la 0. Din
2
0

f ( 0 ) = lim f ( v02 n ) = lim v04 n = 0 , rezultă α = f ( 0 ) = 0 şi deci A = [ 0, ∞ ) .


n→ ∞ n→ ∞

Deducem de aici că f ( x ) = x 2 , ∀ x ∈ [ 0, ∞ ) . Deducem de aici că f ( x ) = x 2 , ∀ x ∈ [ 0, ∞ ) .


În al doilea caz ( u0 < 1 ), rezultă ca mai sus că A conţine şirul descrescător ( uo2n ) n , care

converge la 0 şi, deci, din f ( 0 ) = lim f ( u02 n ) = lim u04 n = 0 , deducem că A ⊃ [ 0,1] .
n→ ∞ n→ ∞

Nu putem avea A = [ 0,1] întrucât, în acest caz, punctul 1 este punct de maxim pentru funcţia derivabilă
f şi, deci, f ' ( 1) = 0 . Pe de altă parte, din x < 1 ⇒ x ∈ A ⇒ f ( x ) = x 2 , deducem că
f ' ( 1) = 2 , contradicţie.
Aşadar, în A există un punct v0 cu v0 > 1 . Din consideraţiile precedente rezultă că A ⊃ [ 1, ∞ ) şi

deci A = [ 0, ∞ ) şi f ( x ) = x 2 , ∀ x ∈ ( 0, ∞ ) .
2. Din ipoteză rezultă P ( a0 ) < 0 . De asemenea, din a1 > a02 + a2 a0 + 1 > a02 − a2 a0 − 1 deducem
P ( − a0 ) > 0 . Aşadar, polinomul P are câte o rădăcină reală în fiecare din intervalele ( − ∞ , − a0 ) ,
( 0, a0 ) , ( a0 , ∞ ) .
Să presupunem că P admite o scriere de forma P = f ⋅ g , unde f şi g sunt polinoame cu
coeficienţi numere întregi şi de grad mai mare sau egal ca 1.
Făcând eventual o alegere, vom avea f ( x ) = x + α , g ( x ) = x 2 + β x + χ , unde α , β , χ sunt numere
întregi. Rezultă, din cele precedente, că avem următoarele alternative: fie rădăcina lui f are modulul mai
mare strict decât a0 , fie rădăcinile lui g au modulul mai mare strict decât a0 .

În primul caz avem a0 = P ( 0 ) = f ( 0 ) g ( 0 ) > α > a0 , iar în al doilea caz avem


a0 = P ( 0 ) = f ( 0 ) g ( 0 ) ≥ g ( 0 ) = χ > a0 , adică relaţii contradictorii.

3. Fie d ', d '' două drepte paralele distincte într-un plan şi A, B, C trei puncte necoliniare situate între
d ' şi d '' .
Notăm cu A ', B ', C ' punctele de pe dreapta d ' cu AA ' ⊥ d ' , BB ' ⊥ d ' , CC ' ⊥ d ' şi cu
A '', B '', C '' punctele de pe dreapta d '' cu AA '' ⊥ d '' , BB '' ⊥ d '' , CC '' ⊥ d '' .
Făcând eventual o alegere vom avea AA ' < BB ' ≤ CC ' ⇔ CC '' < BB '' ≤ AA '' sau
AA ' < BB ' ≤ CC ' ⇔ CC '' ≤ BB '' < AA '' .
Notăm cu O mijlocul segmentului AC şi cu M simetricul lui B faţă de O. Evident, A, B, C , M sunt
vârfurile unui paralelogram. Rămâne să arătăm că M se află între d ' şi d '' .
Fie O ' ∈ d ' şi O '' ∈ d '' astfel încât OO ' ⊥ d ' şi OO '' ⊥ d '' . Făcând eventual o alegere, putem
presupune că OO ' ≥ BB ' . Într-adevăr, în caz contrar vom avea OO '' < BB '' , adică exact inegalitatea
dorită înlocuind d ' cu d '' .
Dacă OO ' ≥ BB ' atunci punctele O şi B sunt în semiplanul determinat de d ' care conţine pe d ''
şi din OO ' ≥ BB ' rezultă că punctul M se află în acelaşi semiplan. Pentru a arăta că M se află între
d ' şi d '' va fi suficient să arătăm că distanţa de la M la d ' este ≤ CC ' .
Fie M ' punctul de pe d ' cu MM ' ⊥ d ' . Avem de arătat că MM ' ≤ CC ' .
În trapezele dreptunghice MM ' BB ' şi AA ' BB ' , dreapta OO ' este linie mijlocie şi deci, vom avea
AA '+ CC ' BB '+ MM '
OO ' = = .
2 2
Întrucât AA ' ≤ BB ' , deducem MM ' ≤ CC ' , adică exact relaţia dorită.

Clasa a XII a

1. Fie f : [ 0, ∞ ) → ¡ o funcţie ca în enunţ. Vom arăta că γ ,γ '∈ ¤ + , 0 < γ < γ '


⇒ f ( γ x ) < f ( γ ' x ) , pentru orice x ∈ ( 0, ∞ ) .
p p'
Într-adevăr, punând γ = , γ '= , unde q ∈ ¥ , q > 0 şi p, p ' ∈ ¥ , cu p < p ' , avem
q q
 px   p'x 
f ( γ x) = f  < f  = f ( γ ' x) .
 q   q 
Fie acum a, b ∈ ( 0, α ) , cu a < b . Vom arăta că f ( a ) ≤ f ( b ) . Se ia n ∈ N , n ≥ 1 , astfel încât
1
a < b − a şi considerăm diviziunea δ n a intervalului [ a, b ] dată de a = t0 < t1 < ... < tkn + 1 = b ,
n
1 k k
unde t1 = a + a, tk = a + a, tkn = a + n a , iar kn este cel mai mare număr natural cu proprietatea
n n n
kn b − a
< .
n a
( )
Notăm ξ = t0 , t1 ,..., tkn un sistem de puncte intermediare asociat diviziunii δ n . Formăm suma Riemann

σ ( δ n ,ξ , f ) . Din cele precedente, avem f ( a ) < f ( tin ) , 1 ≤ i ≤ kn şi deci

a < b ⇒ f ( a) ≤ f ( b) .
b
∫ f ( x ) dx ≤ f ( b ) ( b − a )
a
şi deci,

1  1 
Dacă a < b şi alegem n ∈ ¥ cu a+ a < b , avem f ( a ) < f  a + a  ≤ f ( b ) , f ( a ) < f ( b ) .
n  n 

2. Fie C ∈ M ( n ) cu proprietatea x ∈ ( 0, ∞ ) ⇒ xI + C este inversabilă.


Întrucât pentru x, y ∈ ( 0, ∞ ) , matricele xI + C şi yI + C comută, deducem că inversele lor comută de
A ( x ) = ( xI + C ) . Avem yI + C = xI + C + ( y − x ) I şi deci, înmulţind ambii
−1
asemenea. Punem

membri cu A ( x ) A ( y ) , deducem A ( x ) = A ( y ) + ( y − x ) A ( x ) A ( y ) , adică relaţia dorită.


Fie acum A : ( 0, ∞ ) → M ( n ) , o funcţie cu proprietăţile
(*) A ( x ) = A ( y ) + ( y − x ) A ( x ) A ( y ) , ∀ x, y ∈ ( 0, ∞ )
şi astfel încât există x0 ∈ ( 0, ∞ ) cu proprietatea că matricea A ( x0 ) este inversabilă.
Din (*) rezultă A ( y ) A ( y ) = A ( y ) A ( x ) . Utilizând criteriul A ( x ) inversabilă ⇔ det A ( x ) ≠ 0 ,
deducem că A ( x ) este inversabilă pentru orice x ∈ ( 0, ∞ ) .

( A( x) ) ( A( y) ) = ( A( y ) ) ( A( x) )
−1 −1 −1 −1
Întrucât , deducem din relaţia (*), scrisă sub forma

A ( x ) + xA ( x ) A ( y ) = A ( y ) + yA ( y ) A ( x ) , înmulţind ambii membri cu ( A ( x ) ) ( A( y) )


−1 −1
,

( A( y) ) + xI = ( A ( x ) ) + yI şi deci matricea ( A ( x ) ) − xI nu depinde de x .


−1 −1 −1

C = ( A ( x ) ) − xI , avem ( A ( x ) ) = C + xI , adică ( xI + C ) este inversabilă pentru orice


−1 −1
Notând

x ∈ ( 0, ∞ ) A ( x ) = ( xI + C ) .
−1
şi

3. Fie { P1 , P2 ,..., P3n } o mulţime formată din 3n puncte într-un plan. Pentru orice i, j ∈ { 1, 2,...,3n} cu
i ≠ j , notăm di dreapta care uneşte punctele Pi şi Pj .
Evident, există o dreaptă d 0 în planul dat care nu este paralelă cu niciuna din dreptele d ij cu
i, j ∈ { 1, 2,...,3n} şi astfel încât toate punctele mulţimii date să fie situate într-unul din semiplanele în
care dreapta împarte planul.
Rearanjăm punctele P1 , P2 ,..., P3n în ordinea dată de distribuţia lor faţă de dreapta d 0 . Astfel, P1
va fi cel mai aproape de d 0 , apoi P2 , şi aşa mai departe, ultimul va fi P3n . Remarcăm că aceasta este
posibil, întrucât nu există două puncte Pi , Pj cu i ≠ j , care să fie la aceeaşi distanţă de d 0 . Ducem acum
dreptele distincte d 0' , d1 , d1' , d 2 , d 2' ,..., d n − 1 , d n' − 1 astfel încât fiecare din ele să fie paralelă cu dreapta d 0
şi astfel încât între d 0 şi d 0' să fie doar punctele P1 , P2 , P3 , între d1 şi d1' să fie doar punctele P4 , P5 , P6 ,
şi aşa mai departe, între d k şi d k' să fie doar punctele P3 k + 1 , P3 k + 2 , P3k + 3 , şi deci între d n − 1 şi d n' − 1 să fie
ultimele trei puncte P3 n − 2 , P3 n − 1 , P3 n .
' ' ' ' ' '
De fapt, dreptele d 0 , d1 , d1 , d 2 , d 2 ,..., d n − 1 , d n − 1 sunt astfel încât d k este între d k − 1 şi d k pentru
1 ≤ k ≤ n − 1 . Pentru fiecare 0 ≤ k ≤ n − 1 , ducem dreptele δ k , δ k' , perpendiculare pe d k astfel încât
între ele să figureze punctele P3 k + 1 , P3 k + 2 , P3k + 3 .
Notăm cu Dk dreptunghiul având laturile date de dreptele d k , d k' şi δ k , δ k' .
Prin construcţie, k ≠ l ⇒ Dk ∩ Dl = o şi fiecare dreptunghi Dk conţine în interiorul său punctele
P3 k + 1 , P3 k + 2 , P3k + 3 .
PROBLEME REZOLVATE

CLASA a V-a

1. Rezolvaţi ecuaţia: {[(7 ⋅ x − 5) + 2 ⋅ 6] : 4 + 5} ⋅ 10 − [(4 + x 2 ) ⋅ 5 − 60] = 11

Prof. Nicolae Ivăşchescu, Craiova

Soluţie:

Ecuaţia este echivalentă cu ecuaţia:


 7x + 7  70( x + 1)
 + 5  ⋅ 10 − 20 − 5 x 2 + 60 = 115 ⇔ − 5 x 2 = 25 ⇔ 70 x − 20 x 2 = 30.
 4  4
Se observă că x trebuie să fie mai mic decât 5, pentru că: x = 5 ⇒ 350 − 20 ⋅ 25 = 350 − 500 ≠ 30.
Pentru x = 3 ⇒ 70 ⋅ 3 − 20 ⋅ 32 = 210 − 180 = 30 . Deci x = 3 .

2. Scrieţi numărul 20052005 ca o sumă de două pătrate perfecte.


Prof. Monica Matei, Craiova

Soluţie:

Avem că 2005 = 1681 + 324 = 412 + 182 . Deci


20052005 = 20052004 ·2005 = 20052004 (1681 + 324) =
= 20052004 (412 + 182 ) = 20052004 ·412 + 20052004 ·182 = (20051002 ·41)2 + (20051002 ·18) 2 .

3. Reconstituiţi abbc = d 2 ⋅ aad .


Prof. Nicolae Ivăşchescu, Craiova

Soluţie:
d < 4 , pentru că dacă d = 4 ar rezulta că aad ⋅ 16 ar fi mai mare decât abbc .
Constatăm că
Pentru d = 3 ⇒ abbc = 9 ⋅ aa3 ⇒ u ( abbc) = u (9 ⋅ 3) = 7 ⇒ c = 7 .
Deci aab7 = 9 ⋅ aa3 ⇒ a = 2 ⇒ 2bb7 = 9 ⋅ 223 ⇒ 9 ⋅ 223 = 2007 ⇒ b = 0 . Deci
2
2007 = 3 ⋅ 223 .

4. Aflaţi x, y , z , u numere naturale, nenule, distincte ştiind că xyzu = yzu + xzu + xyu + xyz .

Prof. Nicolae Ivăşchescu, Craiova

Soluţie:

1 1 1 1 1 1 1 1 1 1 1 1 1 1 1
Avem că 1= + . Dar = + ⇒ 1 = + + ⇒ 1 = + + + . Dar = +
2 2 2 3 6 3 6 2 4 12 6 2 3 4 12
1 1 1 1
şi împărţind în relaţia dată cu xyzu obţinem 1 = + + + egalitatea fiind adevărată pentru
x y z u
x = 4, y = 12, z = 6, u = 2 sau orice altă permutare.

5. Fie cifrele nenulea, b, c şi numărul s = abccba + bcaacb + cabbac .


4. Să se arate că s ⋮481 .
2
5. Demonstraţi că nu există a, b, c astfel încât 37 s .

Prof. Turcu Narcis, Brăila

Soluţie:

a) s = 111111⋅ ( a + b + c ) = 481⋅ 231 ⋅ ( a + b + c ) ⋮481.


b) s = 3 ⋅ 7 ⋅ 11 ⋅ 13 ⋅ 37 ⋅ ( a + b + c ) .Cum 3 ≤ a + b + c ≤ 27 rezultă imediat concluzia .

CLASA a VI-a

1. Există n ∈ ¥ ∗ astfel încât 1!+ 2!+ 3!+ ... + n ! se divide cu 7?


Prof. Sorin Pîrlea, Dolj

Soluţie:

Se verifică pentru n ∈ {1, 2,..., 6} iar pentru n ≥ 7 presupunem că n ! ⋮ 7 şi cum pentru n = 6 ,


S= 873 ⋮/ 7 deci răspunsul este nu .

( − 1) ⋅ 1 + ( − 2 ) ⋅ 2 + ( − 3) ⋅ 3 + ... + ( − 9 ) ⋅ 9 + ( − 10 ) ⋅ 10 ∉ ¥ .
1 2 3 9 10
2. Arătaţi că

Prof. Turcu Narcis, Brăila

Soluţie:
Notăm cu n valoarea sumei de sub radical. Deoarece [ − ( 2k − 1) ] 2 k − 1 ⋅ ( 2k − 1) + ( − 2k ) 2 k ⋅ 2k
este pozitivă pentru orice k ∈ {1;2;...;10} , rezultă că n ∈ ¥ ∗ . Deci este suficient să calculăm ultima cifră
a lui n . Fie aceasta U ( n ) .
( )
Atunci U ( n ) = U − 1 + 23 − 34 + ... − 910 + 1011 = U ( − 1 + 8 − 1 + 4 − 5 + 6 − 1 + 8 − 1 + 0 ) = 7 .
De aici rezultă că n nu este pătrat perfect , adică n ∉ ¥ .

3. Scrieţi numărul 394 ca o sumă de patru termeni a, b, c, d, ştiind că


a 0, 625 d 1, ( 1) a 7,5
= , = , = .
b 0, 75 c 0, ( 6 ) d 2
Prof. Fănica Fătu, Brăila

Soluţie:
a 5 6a a 15 4a d 5 3d 3 ⋅ 4 a 4 a
Efectuăm calculele şi obţinem = ⇒ b= ⇒ = ⇒ d= , = ⇒ c= = = .
b 6 5 d 4 15 c 3 5 75 25
6a 4a 4a
Atunci a+ + + = 394 ⇒ a = 150, b = 180, c = 40, d = 24.
5 15 25
4. Să se determine numerele naturale prime a, b, c, ştiind că
a + b + c = 2000 şi b − c = 44.

Prof. Ioan Gurmăzescu,Brăila

Soluţie:

c au aceeaşi paritate deci folosind şi prima condiţie avem că a


Din a doua relaţie se observă că b şi
 b + c = 1998
este par şi prim deci a = 2 ⇒  ⇒ c = 977 ⇒ b = 1021
 b − c = 44
ab
5. Aflaţi numerele ab , cu a ≠ b , astfel încât = ba − 3 .
a− b
Prof.Nicolae Stănică,Brăila

Soluţie:

Se observă că a − b ∈ { 1, 2,3, 4,5, 6, 7,8,9} . Prin calcul direct se obţine a − b = 2 ⇒ ab = 42 .

CLASA a VII-a

x y z 1 1 1 1
1. Calculaţi + + dacă x + y + z = 2007 şi + + = .
y+ z z+ x x+ y x + y y + z z + x 446

Prof. Denisa Florică, Craiova

Soluţie:
Fie
x y z  x   y   z 
E= + + ⇒ E+ 3=  + 1 +  + 1 +  + 1 = ( x + y + z ).
y+ z z+ x x+ y  y+ z   z+ x   x+ y 
 1 1 1  1 9
 + +  = 2007 ⋅ = = 4,5 ⇒ E = 4,5 − 3 = 1,5.
 y+ z z+ x x+ y 446 2

2. Arătaţi că ecuaţia 4 x - 5 y =1 nu are soluţii în mulţimea numerelor naturale.


Prof. Ani Drăghici, Doina Chiriac, Craiova

Soluţie:

Dacă y = 0 ⇒ 4 x = 2 iar pentru y = 1 ⇒ 4 x = 6 şi nu avem soluţie x ∈ ¥ . La fel pentru


x= 0.
Presupunem y ≥ 2 şi ultimele două cifre ale lui 5 y sunt 25 ⇒ 4 x = ...26 . Cum x ≥ 1 şi 26 nu
se divide cu 4 rezultă că ..26 = 5 y + 1 nu se divide cu 4. Deci ecuaţia nu are soluţii în ¥ 2 .

−2

3. Media aritmetică a trei numere este 
2
 . Aflaţi cele trei numere ştiind că sunt direct proporţionale cu
 3
1 1 1
; ; .
0,3 0, (3) 0, 0(3)
Prof. Neculai Carnaru,Brăila

Soluţie:

a+ b+ c 9 27
Fie a, b, c cele trei numere. Din ipoteză = ⇒ a+ b+ c = ( 1) şi a, b, c sunt direct
3 4 4
1 1 1
proporţionale cu ; ; adică 0,3a = 0, ( 3) b = 0, 0 ( 3) c = k ( 2) şi se obţin a, b, c
0,3 0, (3) 0, 0(3)
prin înlocuirea în ( 1) şi apoi în ( 2 ) .
a
4. Să se determine toate fracţiile de forma ( a, b ∈ ¥ ) , cu proprietatea că, adunând şi la numărător şi

b
la numitor acelaşi număr raţional x > 0, obţinem o putere cu exponent natural a fracţiei date.

Prof. Marius Damian, Brăila

Soluţie:
n
a a a+ x
< 1 şi atunci   <
a
3. Dacă a< b⇒ <
b  b b b+ x
n
a  a a a+ x
4. Dacă a > b ⇒ > 1 şi atunci   > > .
b  b b b+ x
n
 a a+ x
Din cele două cazuri obţinem că a = b . Atunci   = adică fracţiile unitare satisfac ipoteza.
 b b+ x

CLASA a VIII-a

1. Fie x, y , z , t ∈
¡ astfel încât x + y + z + t = 0 şi A = xy + yz + zt , B = xz + xt + yt. Arătaţi că
3 A + 5 B ≤ 0 sau 5 A + 3B ≤ 0.

Prof. Ileana Mândruleanu, Craiova

Soluţie:

Ridicând relaţia din ipoteză la pătrat obţinem x 2 + y 2 + z 2 + t 2 = − 2( A + B) ⇒


x2 + y 2 + z 2 + t 2
A+ B = − . Dacă presupunem prin absurd că 3 A + 5 B > 0 şi 5 A + 3B > 0 , atunci
2
va rezulta că 8( A + B ) > 0 ⇒ A + B > 0 contradicţie, deci 3 A + 5 B ≤ 0 sau 5 A + 3B ≤ 0.
2. Rezolvaţi în mulţimea numerelor reale ecuaţia:

( 5 − x) ( 2 6− 5 = x) 2
x + 25 − 10 x .
2

Prof. Daniela Tilincă, Brăila

Soluţie:

Ecuaţia devine ( 5 − x) ( 2 )
6 − 5 = x 2 x − 5 . Dacă x ≥ 5 atunci x = 5 este soluţie a ecuaţiei, iar

pentru x < 5 ecuaţia nu are soluţii pentru că am avea −35 = x 2


214 26 4
<0

3. a) Aflaţi valoarea maximă a expresiei:


3 x + 12 x + 22
2

E ( x) = , atunci când x ∈ ¡ .
x + 4x + 6
2

b) Rezolvaţi în ¡ × ¡ ecuaţia:
3 x + 12 x + 22 = x + 4 x + 6
2 2
( ) ( 25 y 2
)
− 10 y + 6 .

Prof. Valentin Florin Damian, Brăila

Soluţie:
4 4
a) E ( x) = 3 + = 3+ are valoarea maximă
x + 4x + 6
2
( x + 2) + 2 2

⇔ x− 2= 0⇒ x = −2⇒ ⇒ Emax ( x ) = E ( − 2 ) = 5 ⇒ E ( x) ≤ 5 .
3 x + 12 x + 22
2

b) Ecuaţia devine
x + 4x + 6
2

1
= 25 y − 10 y + 6 ≤ 5 ⇒
2
( 5 y − 1) 2
≤ 0⇒ y = ⇒ x = − 2.
5
4. Fie ABCDA'B'C'D' un cub cuAB = 4 şi BC '∩ CB ' = { O} .
a) Dacă AO ∩ D ' C ' = { P} , aflaţi distanţa de la punctul P la dreapta AC.

2 5
b) Dacă punctul M este situat pe segmentul ( BB ' ) , astfel încât sin ( R BOM ) = , aflaţi
5
lungimea segmentului [ BM ] .
Prof. Nicolae Stănică, Brăila

Soluţie:

AB ⊥ ( BB ' C ') 
 T3
a) BO ⊥ B ' C  ⇒ AO ⊥ B ' C ⇒ V AOC dreptunghic în O.
BO, B ' C ⊂ ( BB ' C ') 
Fie PK distanţa de P la AC , K ∈ AC atunci VOC ' P : V AD ' P ⇒ PA = 2 AO = 2 ⋅ 2 6 = 4 6
PK PA PA ⋅ OC
. Apoi V AOC : V AKP ⇒ = ⇒ PK = = 2 6.
OC AC AC
b) Exprimăm aria VBOM în două moduri şi anume:
2 5 8
2 Aria ( VBOM ) = BM ⋅ 2 = OM ⋅ 2 2 ⋅ . Notăm BM = x ⇒ x = OM .
2 2

5 5
Din teorema cosinusului avem
2 8 2 4
OM 2 = x 2 + OB 2 − 2OB ⋅ MB
2
= x2 − 4x + 8 ⇒ x2 =
5
( )
x − 4 x + 8 ⇒ x1 = 6, nu convine şi x2 = < 2.
3
5. Fie ABCDA ' B ' C ' D ' un paralelipiped dreptunghic. Notăm cu M, N, P centrele feţelor ABCD,
BCC ' B ' , respectiv ABB ' A ' . De asemenea, notăm cu s aria triunghiului MNP şi cu S aria totală a
S
paralelipipedului ABCDA ' B ' C ' D '. Dacă = 16 3, demonstraţi că paralelipipedul ABCDA'B'C'D'
s
este cub.

Prof. Marius Damian, Brăila

Soluţie:

AB = a, BC = b, AA ' = c. Atunci PM = b2 + c2 a2 + c2 a 2 + b2
Fie , NM = , PN =
2 2 2
. În VPNK ducem înălţimea PK .Fie NK = x . În triunghiul dreptunghic VPNK avem că
a 2 + b2
PK 2 = − x2 ( ∗ ) ,
4
2
b2 + c2  a 2 + c2 
iar în triunghiul dreptunghic VPMK avem că PK 2 = − − x  ( ∗ ∗ ) . Din ( ∗ ) şi
4  2 
 
a 2
a +b
2 2
a 4
a 2b 2 + b 2 c 2 + c 2 a 2
( )
∗ ∗ obţinem că x = , atunci PK 2
=
4
− x =
4 ( a2 + c2 )
=
4 ( a2 + c2 )
.
2 a2 + c2
NM ⋅ PK a 2 + c 2 a 2b 2 + b 2 c 2 + c 2 a 2 1 a 2b 2 + b 2 c 2 + c 2 a 2
S [ PNM ] = = ⋅ ⋅ ⇒ S [ PNM ] = (∗ ∗ ∗ )
2 2 2 a2 + c2 2 8
S a 2b 2 + b 2 c 2 + c 2 a 2
( ab + bc + ac ) = 3 ( a 2b 2 + b 2 c 2 + c 2 a 2 )
2
= 16 3 ⇒ 2 ( ab + bc + ac ) = ⋅ 16 3 ⇔
s 8
( ab − bc ) + ( bc − ac ) + ( ac − ab ) = 0 ⇒ ab = ac = bc ⇒ a = b = c deci paralelipipedul este cub.
2 2 2

CLASA a IX-a

1. Fie E ( n ) = an 2 + bn + c, a, b, c ∈ ¥ , a ≠ 0, n ∈ ¥ . Să se determine a, b, c dacă:


 E (n)   E ( n + 1)   E (n + 2) 
 3  +  +  = n + 3n + 2, ∀ n ∈ ¥ .
2

3   3
Prof. Gheorghe Alexe, Brăila

Soluţie:

 E (0)   E (1)   E (2) 


1. Pentru n = 0 ⇒  + + = 2
 3   3   3 

 E (1)   E (2)   E (3)   E (3)   E (0)   9a + 3b + c   c 


2. Pentru n = 1 ⇒   +   +   = 6⇒  − = 4⇒  −  3  = 4 ⇒
 3   3   3   3   3   3
 c  c
⇒ 3a + b +   −   = 4 ⇒ 3a + b = 4, a, b ∈ ¥ , a ≠ 0 ⇒ a = b = 1.
 3  3
 E (0)   E (1)   E (2)   c  c + 2  c + 6  c  c + 2
Ştim că   +   +   = 2⇒   +  + = 2⇒ 2⋅   +  = 0⇔
 3   3   3   3   3   3   3   3 
 c  c + 2
⇔   = 0 ⇔ c ∈ {0,1, 2} şi  = 0⇔ c = 0 ⇒ c = 0. Deci a = b = 1 şi c = 0, E ( n) = n( n + 1),
 3  3 
 n( n + 1)   ( n + 1)( n + 2)   ( n + 2)( n + 3) 
atunci  +  + = n 2 + 3n + 2, ∀ n ∈ ¥ .
 3   3  3 

2. Fie 1) [ a ] x 2 + [ b] x + [ c ] = 0, a, b, c ∈ ¡ / ¢
2) [ − a ] x + [ − bx ] + [ − c ] = 0, a > 1, b < 0, c > 0 x ∈ ¡ .
2

Să se arate că :
i) [ − x] = − 1 − [ x] , ∀ x ∈ ¡ / ¢
ii) Dacă 1) nu admite rădăcinile reale atunci nici 2) nu admite rădăcinile reale, iar dacă 2) are
rădăcinile reale atunci şi 1) are rădăcinile reale.

Prof. Gheorghe Alexe, Brăila

Soluţie:

i). Ştim că x = [ x ] + { x} ⇒ − x = − [ x ] − { x} ⇒ − x = − 1 − [ x ] + 1 − { x} ⇒ [ − x ] =  − 1 − [ x ] + 1 − { x}  =
= − 1 − [ x ] +  1 − { x}  , 0 ≤ { x} < 1 ⇒ − 1 < − { x} ≤ 0 ⇒ 0 < 1 − { x} ≤ 1
Nu putem avea 1 − { x} = 1, deoarece ⇒ { x} = 0 ⇒ [ x ] = x ⇒ x ∈ ¢ fals
⇒ 0 < 1 − { x} ≤ 1 ⇒
⇒  1 − { x}  = 0 ⇒ [ − x ] = − 1 − [ x ] , ∀ x ∈ ¡ / ¢ .
∆ 1 = [ b] − 4 [ a ] [ c] .
2
ii).

∆ 2 = [ − b] − 4 [ − a ] [ − c] = ( − 1 − [ b] ) − 4 ( − 1 − [ a ] ) ( − 1 − [ c ] ) = 1 + 2 [ b] + [ b] − 4 − 4 [ c ] − 4 [ a ] −
2 2 2

− 4 [ a ] [ c] = [ b] − 4 [ a ] [ c ] − 3 + 2 [ b] − 4 [ c ] − 4 [ a ] ⇒ ∆ 2 = ∆ 1 − 3 + 2 [ b] − 4 [ c ] − 4 [ a ] .
2

a > 1 ⇒ [ a ] ≥ 1, c > 0 ⇒ [ c ] ≥ 0 , b < 0 ⇒ [ b] < 0 .


Dacă
Discuţie: 1) Dacã ∆ 1 < 0 ⇒ ∆ 2 < 0 .
2) Dacã ∆ 2 ≥ 0 ⇒ ∆ 1 ≥ 0 .

3. Determinaţi funcţia f : ¡ → ¡ , cu proprietăţile:


19. f ( x + y ) = f ( x ) f ( a − y ) + f ( y ) f ( a − x ) , ( ∀ ) x, y , ( ∀ ) a ∈ ¡ .
1
20. f ( 0 ) = .
2
***

Soluţie: Fie y = 0 şi a = 2 x .Obţinem f ( x ) = f ( x ) f ( 2 x ) + f ( 0 ) f ( x ) .Atunci f ( x ) = 0


1
sau f ( 2x) = .
2
Arătăm că f ( x ) = 0 nu convine. Dacă f ( x ) = 0 pentru un x oarecare, luând y = 0 obţinem
0 = f ( x + 0 ) = f ( x ) f ( a ) + f ( 0 ) f ( a − x ) , de unde f ( a − x ) = 0, ∀ x ∈ ¡ fals deoarece
1 1
f ( 0 ) ≠ 0 . Atunci f ( 2 x ) = , ∀ x ∈ ¡ sau f ( x ) = , ∀ x ∈ ¡ .
2 2
3. Fie n ∈ ¥ , n ≥ 2.
1) Să se arate că:
a) [ x + k ] = [ x ] + k , ∀ x ∈ ¡ , ∀ k ∈ ¢ .
b) [ nx ] ≥ [ x ] + [ ( n − 1) x ] ≥ 2 [ x ] + [ (n − 2) x ] ≥ ... ≥ n [ x ] .
2) Să se afle x ∈ ¡ astfel încât numerele întregi n [ x ] , [ ( n − 2) x ] + [ 2 x ] ,..., [ x ] + [ ( n − 1) x ] , [ nx ] să fie
consecutive.
Prof. Vasile Popa, Galaţi

Soluţie:

1. a) Fie x = m + α , m ∈ ¢ , α ∈ [ 0,1) , iar x + k = m + k + α ⇒ [ x + k ] = m + k = [ x ] + k .


b) Fie x ∈ ¡ , x = m + α . Vom demonstra că pentru ∀ t = 0, n − 1 , avem:
t [ x ] +  ( n − t ) x  ≥ ( t + 1) [ x ] +  ( n − t − 1) x  .
Fie x = m + α , avem:
(1)
⇔ nt + ( n − t ) m +  ( n − t ) α  ≥ ( t + 1) m + ( n − t − 1) m + ( n − t − 1) ⇔  ( n − t ) α  ≥  ( n − t − 1) α 
,
aceasta fiind adevărată deoarece ( n − t ) α ≥ ( n − t − 1) α ( ⇔ α ≥ 0 ) . Rezultă relaţia 1.b).
2. Avem n ⋅ [ x] ≤ ( n − 2 ) ⋅ [ x ] + [ 2 x ] + ≤ ... ≤ [ x ] +  ( n − 1) ⋅ x  ≤ [ nx ] .
Fie x = m + α . Numerele devin
nm, ( n − 2 ) m +  2 ( m + α )  ,..., m +  ( n − 1) ( m + α )  >  n ( m + α )  .
Urmează că numerele: 0, [ 2α ] , [ 3α ] ,..., [ nα ] trebuie să fie consecutive.

Aşa că [ 2α ] = 1 , [ 3α ] = 2 ,..., [ nα ] = n − 1 sau 1 ≤ 2α ≤ 2, 2 ≤ 3α ≤ 3,..., n − 1 ≤ nα < n sau


1 2 n− 1  n− 1 
≤ α < 1, ≤ α < 1,..., ≤ α < 1⇒ α ∈  ,1 .
2 3 n  n 

4. Fie x1 , x2 , x3 , x4 ∈ ¡ astfel încât x1 + x2 + x3 + x4 = 2 şi x1 , x2 , x3 , x4 ≤ 1. Arătaţi că x1 , x2 , x3 , x4 ≥ − 1.


Prof. Roxana şi Luminiţa Vasile, Craiova

Soluţie:

Presupunem prin absurd că x4 > − 1 ⇒ x1 + x2 + x3 = 2 − x4 > 3 , ceea ce este fals deoarece


x1 + x2 + x3 ≤ 3 .

n ∈ ¥ , are loc identitatea:


*
5. Să se demonstreze că, oricare ar fi
1 1 1 sin n
+ + ... + = .
sin n sin ( n + 1) sin 1sin ( n + 1)
2
sin1sin 2 sin 2sin 3

Prof. Ion Dan, Brăila


Soluţie:

Demonstrăm egalitatea prin inducţie matematică.


Pentru n = 1 egalitatea este adevărată. Presupunem că pentru un n oarecare relaţia este adevărată şi o
demonstrăm pentru n + 1 .
1 1 1 1 sin ( n + 1)
+ + ... + + = ⇒
sin n sin ( n + 1) sin ( n + 1) sin ( n + 2 ) sin 1sin ( n + 2 )
2
sin1sin 2 sin 2sin 3
sin n 1 sin ( n + 1)
+ = ⇒ sin n sin ( n + 2 )
sin 1sin ( n + 1) sin ( n + 1) sin ( n + 2 ) sin 1sin ( n + 2 )
2 2

Prin calcule elementare obţinem cos 2 − cos ( 2n + 2 ) + 1 − cos 2 = 1 − cos 2 ( n + 1) adevărat.

CLASA a X-a

1 1 1 1 1 1
1. Fie x, y, z > 0 astfel încât + + ≥ 2. Arătaţi că + + ≥ 12.
2x + 1 2 y + 1 2z + 1 x y z
În ce caz avem egalitate?
Prof. Marilena Andreescu , Podari, Dolj

Soluţie:

1 1 1
≥ xy + yz + zx + 4 xyz ≥ 4 4 4 ( xyz ) ⇒ 4 = 2 ≥ 4 ( xyz ) ⇒
3 3
4
Efectuând calculele obţinem
4 2 4
1 1 1 1 1
⇒ ( xyz ) < 18 ⇒ xyz ≥ 26 . Apoi + + ≥ 3 3
3
≥ 3 3 26 − 12 .
2 x y z xyz
1
Egalitate pentru xy = yz = zx = 4 xyz ⇔ x = y = z = .
4
2. Fie a, b, c lungimile laturilor unui triunghi. Arătaţi că: (a3 + b3 )2 + 3abc 2 (2a 2 + ab + 2b 2 ) > c 6

Mircea Tereujeanu, Craiova

Soluţie:

(a 3 + b3 ) 2 + 3abc 2 ( 2a 2 + ab + 2b 2 ) = ( a + b ) (a − ab + b 2 ) + 3abc 2 ( 2a 2 + ab + 2b 2 ) >


2 2 2

> c 2 ( a 2 − ab + b 2 ) + 3abc 2 ( 2a 2 + ab + 2b 2 ) > c 2 ( a 4 + 3a 2b 2 + b 4 − 2a 3b − 2ab3 + b 4 + 6a 3b +


2

+ 3a 2b 2 + 6ab3 ) = c 2 ( a 4 + 4a 3b + 6a 2b 2 + 4ab3 + b 4 ) = c 2 ( a + b ) > c 2 c 4 > c 6


4

 x
 log 2007 y = 2007 − 2007
y x

3. Rezolvaţi în numere reale strict pozitive sistemul de ecuaţii: 


 x 2 + y = 2007

Mircea Tereujeanu, Craiova

Soluţie:
x x
Presupunem x > y > 0⇒
> 1 ⇒ log 2007 > 0 şi 2007 y − 2007 x < 0 fals. Analog pentru
y y
x x
y > x > 0 obţinem < 1 ⇒ log 2007 < 0 şi 2007 y − 2007 x > 0 . Aşadar, x = y . Rezultă că
y y
− 1 ± 1 + 4 ⋅ 2007 8029 − 1
x 2 + x − 2007 = 0 ⇒ x1,2 = cu soluţia x = y = .
2 2

4. a) Determinaţi funcţia strict crescătoare f : ¡ → ¡ care satisface condiţia:


f ( x ) + f ( f ( x ) ) = 2 x, ( ∀ ) x ∈ ¡ .
b) Să se rezolve ecuaţia: 2 (x log 2 3
)
− 1 = x + ( x + 1)
log 3 2
.
(OLM,2007, Brăila )

Soluţie:
a) f ( a ) > a ⇒ f ( f ( a ) ) > f ( a ) ⇒ f ( f ( a ) ) + f ( a ) > 2a 
 ⇒ f ( x) = x .
f ( a ) < a ⇒ f ( f ( a ) ) < f ( a ) ⇒ f ( f ( a ) ) + f ( a ) < 2a 
1
b) Se observă că x > 1 . Notăm cu log 2 3 log 2 3 şi ecuaţia devine:
x = y > 1⇒ x = y = y log 3 2

( )
log 3 2
2 y = 2 + y log3 2 + y log 3 2 + 1 . Considerăm funcţia f : ( 1, ∞ ) → ¡ f ( y ) = y log3 2 + 1 . Atunci
avem

( )
log 3 2
f ( f ( y ) ) = y log3 2 + 1 +1
deci ecuaţia devine f ( f ( y ) ) + f ( y ) = 2 y , unde f este o funcţie strict crescătoare şi conform punctului
1 1
a) avem f ( y ) = y , adică y log3 2 + 1 = y ⇔ x + 1 = x log 2 3 ⇔ 1 + = x log 2 3− 1 ⇒ 1 + −
x x
1 log 2 3− 1
− x log 2 3− 1 = 0 ⇒ x = 2 soluţie unică deoarece funcţia g ( x) = 1+ − x este strict
x
descrescătoare.

5. Să se rezolve sistemul:

 x 2 log 2 15 + y 2 log 3 10 + z 2 log 5 6 = 2( xy + yz + zx )



 x+ y+ z= 5

Prof. Marius Damian, Brăila(OLM,2007, Brăila )

Soluţie:

Notăm cu a = log t 2, b = log t 3, c = log t 5, t > 1 , atunci ecuaţia a doua devine:


2 2 2
b+ c 2 a+ c 2 a+ b 2  b a   c a   c b 
x + y + z = 2 ( xy + xz + yz ) ⇒  x− y  +  x− z +  y− z = 0
a b c  a b   a c   b c 
Atunci
b c 5a 5b 5c
y= x, z = x ⇒ x = = 5log 30 2, y = = 5log30 3, z = = 5log30 5.
a a a+ b+ c a+ b+ c a+ b+ c

CLASA a XI-a

1. Fie A, B ∈ M 2 ( ¡ ), ε ∈ £ − ¡ rădăcină cubică complexă a unităţii şi n ∈ ¥ *arbitrar fixat.


det ( An + ε B n ) + det ( An + ε 2 B n ) = det ( An − B n ) ⇔ ( det A) = 2 ( det B )
n n
Demonstraţi că

Prof. Cătălin Cristea,Craiova

Soluţie:

Folosim următorul rezultat cunoscut:


Dacă A, B ∈ M 2 ( £ ) , atunci det ( A + B ) + det ( A − B ) = 2 ( det A + det B ) .
Avem:

det ( An + ε B n ) + det ( An + ε 2 B n ) =
1
2
(
 )
det 2 An + ( ε 2 + ε ) B n + det ( ε 2 − ε ) B n  =

1 1 1
 det ( 2 An − B n ) + det ( ε 2 − ε ) B n  =  det ( 2 An − B n ) + det ( ε 2 − ε ) B n  =  det ( 2 An − B n ) +
2
=  
2 2  2

+ ( ε − 2 + ε 2 ) det B n  = 12  det ( 2 An − B n ) − 3det Bn  = 12  det ( 2 An − Bn ) + det B n  − 2 det B n =


1
= det ( 2 A n
− 2 B n
) + det ( 2 A n
)  − 2 det B n = 1  4 det ( An − B n ) + 4 det An  − 2 det B n = det ( An − B n ) +
4  4 
+ det An − 2 det B n ⇒ det ( An + ε B n ) + det ( An + ε 2 B n ) = det ( An − B n ) ⇔ det An − 2 det B n = 0 ⇔

⇔ det An = 2 ( det B ) .
n

(n − 1) xn − nxn − 1 x
2. Fie (xn ) n≥ 1 un şir de numere reale astfel încât lim = 1. Calculaţi lim n4 .
n→ ∞ n(n − 1)(3n − 3n + 1)
2 n → ∞ n

Prof. Florentin Nicolae, Filiaşi

Soluţie:

Folosind teorema lui Cesaro-Stolz,


xn xn xn − 1

xn n
lim 4 = lim 3 = lim 3 n n − 1 = lim ( n − 1) xn − nxn − 1 = 1
n→ ∞ n n→ ∞ n n→ ∞
n − ( n − 1)
3
( )(
n → ∞ n n − 1 3n 2 − 3n + 1
)
 f (x) 
3. Fie f : (0, ∞ ) → (0, ∞ ) o funcţie cu proprietatea lui Darboux astfel încât f   = x, ∀ x > 0.
 x 
Arătaţi că f este continuă.

Prof. Ionuţ Ivănescu, Prof.Liviu Smarandache, Craiova

Soluţie:
f ( x)
Fie g : ( 0, ∞ ) → ( 0, ∞ ) , g ( x) = . Atunci
x
 f ( x) 
f 
f ( g ( x) ) x  = f ( x ) = x , ∀ x > 0 ⇒ g ° g = 1( 0,∞ ) ⇒ g este injectivă.
g ( g ( x) ) = = 
g ( x) f ( x) f ( x)
x x
Cum g are proprietatea lui Darboux ⇒ g continuă, dar f ( x ) = x ⋅ g ( x ) ⇒ f este continuă.

4. Determinaţi funcţiile continue f : ¡ → (0, ∞ ) care verifică relaţia: f ( x + y ) = f ( x − y ) ⋅ f 2 ( y ), ∀ x, y ∈ ¡ .

Prof. Gabriel Tica, Băileşti

Soluţie:
Logaritmând relaţia din ipoteză, rezultă ln ( f ( x + y ) ) = ln ( f ( x − y ) ) + 2 ln f ( y ) . Notăm
g ( x ) = ln f ( x ) ⇒ g ( x + y ) − g ( x − y ) = 2 g ( y ) , ∀ x, y ∈ ¡ . Pentru y = 0 ⇒ g ( 0 ) = 0 . Dacă
y = x ⇒ g ( 2 x ) = 2 g ( x ) ⇒ g impară. Dacă y = 2 x ⇒ g ( 3x ) = 3g ( x ) , de unde prin inducţie
matematiă obţinem g ( nx ) = ng ( x ) , ∀ n ∈ ¥ .
p
Deoarece g este impară ⇒ g ( nx ) = ng ( x ) , ∀ n ∈ ¢ . Dacă x = , p, q ∈ ¢ * ⇒ qx = p ⇒
q
p
⇒ g ( g ( x ) ) = g ( p ⋅ 1) = p ⋅ g ( 1) ⇒ g ( x ) =
g ( 1) = xg ( 1) , ∀ x ∈ ¤ .
q
Dacă x ∈ ¡ \ ¤ , alegem un şir de numere raţionale ( xn ) n ≥ 1 , xn → x . Deoarece f este continuă, g

este continuă şi g ( x ) = lim g ( xn ) = lim xn ⋅ g ( 1) = x ⋅ g ( 1) , de unde f ( x ) = e ax , cu


n→ ∞ n→ ∞

a = g ( 1) = ln f ( 1) .

2007  1 2 3 4
5. Să se determine σ ∈ S 4 cu proprietatea că σ =  .
4 3 2 1
Prof. Dan Negulescu, Brăila (OLM,2007, Brăila )

Soluţie:

 1 2 3 4  1 2 3 4
Dacă σ este o soluţie, obţinem că σ   =  σ calculând în două moduri σ 2008 .
4 3 2 1  4 3 2 1 
 1 2 3 4  = σ  1 2 3 4  σ − 1 şi luând σ =  1 2 3 4  ∈ S obţinem
Atunci   4 3 2 1 a b c d  4
4 3 2 1    
 1 2 3 4  =  1 2 3 4 1 2 3 4  a b c d ⇔  1 2 3 4  =  a b c d 
 4 3 2 1   a b c d   4 3 2 1   1 2 3 4  4 3 2 1   d c b a
         
pentru a = 1 obţinem d = 4 şi deci σ = 
 1 2 3 4  sau σ =  1 2 3 4  . Numai prima soluţie
 4 2 3 1
4 3 2 1  
verifică ecuaţia iniţială. În mod asemănător se tratează cazurile a ∈ { 2,3, 4} .

 1 − 1 1 1
6. Să se rezolve în mulţimea M 2 ( ¡ ) ecuaţia X ⋅  ⋅X=  . Discuţie după .
 −1 1  1 m
valorile parametrului real m.

Prof. Dan Negulescu, Brăila(OLM,2007, Brăila )

Soluţie:

Trecând la determinanţi obţinem: ( det X ) ⋅ 0 ⋅ ( det X ) = m − 1 ⇒ m = 1 . Deci pentru m ≠ 1 nu


există soluţii.
 1 − 1 1 1  1 − 1  0 0
 ⋅ = 
2
Fie m = 1 şi notînd Y = X ⋅  obţinem: Y =    . Luăm
 −1 1 1 1  − 1 1  0 0 
 x y  obţinem det Y = 0 şi deci 2
 z t Y = ( x + t ) Y , de unde Y = O2 ⇔
2
Y =
 
 x ( x+ t ) = 0
 y ( x+ t ) = 0

⇔  . Obţinem ( x + t ) 2 = 0 ⇔ t = − x . Din xt = yz găsim yz =
2
x . Deci t = − x
 z ( x+ t ) = 0
 t ( x + t ) = 0
2
şi yz = x .

− x2
 x y 

Pentru x = 0 luăm t = − x , z = , x, y ∈ ¡ ⇒ Y =  − x2  . Mai departe luăm
y  − x
 y 
 x1 x2 
Y = X ⋅
 1 − 1
Y =  y 2 
şi rezolvăm  . Se obţin o infinitate de soluţii.
 y1  −1 1

CLASA a XII-a

1. Pe mulţimea M = ( 0; ∞ ) se defineşte o lege de compoziţie "∗ " astfel încât

x∗
1
= y( x ∗ y) , x ∗
9
=
y
( x ∗ y ) şi ( x ∗ y ) ( x ∗ 81 y ) = ( x + 1) 2 ( ∀ ) x, y ∈ M.
,
y y 3 36 xy
a) Să se studieze asociativitatea, comutativitatea şi existenţa elementului neutru pentru operaţia "∗ " .
∗ astfel încât *
b) Demonstraţi că nu există x∈ ¥ x ∗ 81 ∈ ¥ .
Prof. Gabriel Daniilescu, Brăila(OLM,2007, Brăila )

Soluţie:

9 9y 1 x∗ y
Avem că x ∗ = ( x ∗ 9 y) ⇔ x∗ = 3y ( x ∗ 9 y) = y ⋅ ( x ∗ y) ⇒ x ∗ 9 y = ⇒
9y 3 y 3

x ∗ 9y x∗ y ( x + 1) 2 ⇒ ( x + y ) 2 = ( x + 1).2 ⇒ x+ 1
⇒ x ∗ 81şi
y= = 81 ( x ∗ y) ( x ∗ y) = x∗ y =
3 9 36 xy 9 36 xy 2 xy
y+ 1 y+ 1  3 4 
y∗ x = = deci legea nu este comutativă  2∗ 3 = ≠ 3∗ 2 =  şi analog nu
2 yx 2 xy  2 6 2 6
este asociativă. Se verifică uşor că legea nu admite element neutru.
x+ 1
. Cum 18x nu divide x + 1, ( ∀ ) x ∈ ¥ obţinem că x ∗ 81 ∉ ¥ * .

b) x ∗ 81 =
18 x
2. Să se afle f : ¡ → ¡ , care admite primitive astfel încât pentru o primitivă F , să avem
f ( x ) = 2007 ⋅ F ( x ) + sin x, ∀ x ∈ ¡ .

Prof. Carmen şi Viorel Botea, Brăila

Soluţie:

F ' ( x ) − 2007 F ( x ) = sin x ⋅ e − 2007 x ⇔ ( F ( x) ⋅ e − 2007 x


) ' = sin x ⋅ e − 2007 x
.
Fie

I = ∫ e− 2007 x sin xdx =


e− 2007 x
⋅ sin x + ∫
e− 2007 x cos x e − 2007 x e − 2007 x cos x e
− 2007 x
( − sin x ) dx ⇒
− 2007 2007
dx = −
2007
⋅ sin x −
2007 2
+ ∫ 1 4 42007
2
2 4 43
I
2007 2

 1  e − 2007 x − e − 2007 x ⋅ ( 2007 sin x + cos x )


⇒ I  1+  = − ⋅ ( 2007 sin x + cos x ) ⇒ I = , deci
 2007 2  2007 2 20072 + 1
'
 − e− 2007 x ⋅ ( 2007 sin x + cos x )  − e− 2007 x
(F ( x) ⋅ e − 2007 x
'= 
 ) 1 + 2007 2  ⇔ F ( )
x ⋅ e − 2007 x
=
1 + 2007 2 (
⋅ 2007 sin x + cos x ) + C
 
− ( 2007 sin x + cos x ) ( 2007 sin x + cos x ) .
∀ x ∈ ¡ ⇔ F ( x) = + C ⋅ e2007 x ⇒ f ( x ) = C ⋅ e2007 x ⋅ 2007 −
1 + 2007 2
1 + 2007 2
3. Determinaţi a ∈ ¡ astfel încât funcţia h : ¡ → ¡ ,

 1 1
,x≠ 0
( )
cos

h( x) = 
2 2
x +1 ln x + x +1 să admită primitive pe ¡.

 a , x= 0

Prof. Adela Dimov, Brăila(OLM,2007, Brăila )

Soluţie:

Considerăm g : ¡ → ¡ , g ( x ) = ln x + ( 2
x +1 . ) Obţinem că

1
g '( x) = , ( ∀ ) x ∈ ¡ şi
2
x +1
 g 2 x sin 1 , x ≠ 0
 ( )
g ( x ) = 0 ⇔ x = 0 . Acum funcţia F : ¡ → ¡ , F ( x ) =  g ( x) este derivabilă pe ¡ şi
 0 ,x= 0

 2 g x g ' x sin 1 + g ' x cos 1 , x ≠ 0


 ( ) ( ) ( )
F '( x) =  g ( x) g ( x) dar
 0 , x= 0
 2 g ( x ) g ' ( x ) sin 1 , x ≠ 0

u ( x) =  g ( x) este continuă pe ¡ deci are primitive. De aici obţinem că
 0 ,x= 0
 g ' x cos 1 , x ≠ 0
 ( )
h0 ( x ) =  g ( x) are primitive pe ¡ .
 0 , x= 0

Cum ( h − h ) ( x ) = { 0,a, xx =≠ 00 obţinem că h are primitive numai pentru a = 0 .


0

4. Fie ( G , ⋅ ) un grup abelian finit, G= { a1 , a2 ,..., an } în care ecuaţia t 3 = e are soluţie unică, e fiind
elementul neutru al grupului ( G , ⋅ ) . Arătaţi că ∃ b1 , b2 ,..., bn ∈ G cu b1 a1 = e, b2 a2 = e,..., bn an = e .
3 3 3

( a1a2 ...an )
4
Calculaţi .

Prof. Carmen şi Viorel Botea, Brăila

Soluţie:

t 3 = e are soluţia t = e , care este unică conform ipotezei. Fie f : G → G , f ( x ) = x ;


3
Ecuaţia
demonstrăm că f injectivă.

( xy )
G
Fie f ( x ) = f ( y ) , rezultă că x 3 = y 3 ⋅ y − 3 ⇒ x 3 y − 3 = e ⇒ −1 3
= e ⇒ xy − 1 = e (soluţie unică)
abelian

⇒ x = y ⇒ f este injectivă.
Dar cum ( G , ⋅ ) este finit, rezultă că f este şi surjectivă, deci, ţinând cont şi de demonstraţia anterioară,

rezultă că f este bijectivă ⇒ ∀ ai− 1 ∈ G, ∃ !bi ∈ G cu f ( bi ) = ai− 1 , ∀ i = 1, n .


Cum a1 , a2 ,..., an sunt distincte ⇒ a1− 1 ,..., an− 1 distincte şi f fiind bijectivă, rezultă că
b1 , b2 ,..., bn ∈ G distincte şi { b1 , b2 ,..., bn } = { a1 , a2 ,..., an } ⇒ b1b2 ⋅ ... ⋅ bn = a1a2 ⋅ ... ⋅ an .
( b1b2 ...bn ) ( a1a2 ...an )
3 4
Dar, a1a2 ...an = e ⇒ = e.

5. Să se determine toate funcţiile f : ( a; b ) → ¡ continue, ce au proprietatea că, pentru orice primitivă


F a lui f , funcţia G : ( a; b ) → ¡ , G ( x ) = F ( x ) este derivabilă pe ( a, b) .

Prof. Iulian Danielescu, Brăila(OLM,2007, Brăila )

Soluţie:

f este continuă pe ( a; b ) atunci admite primitive şi considerăm F0 : ( a; b ) → ¡ , o primitivă.


Luăm mai departe x0 ∈ ( a; b ) arbitrar, atunci F = F0 − F ( x0 ) este o primitivă pe ( a; b ) a lui f , şi
din ipoteză avem că G ( x ) = F0 − F ( x0 ) este derivabilă pe ( a, b) .
Cum F ( x0 ) = 0, f este derivabilă şi F este derivabilă obţin F ' ( x0 ) = 0 ⇔ F ' ( x0 ) = 0 .
0
Cum x0 a fost ales arbitrar f ( x0 ) = F ' ( x0 ) = 0 deci f este funcţia nulă.
0

OLIMPIADA DE MATEMATICĂ
ETAPA LOCALĂ, 26.01.2008
CLASA A V-A

1. Numerele naturale a , b, c îndeplinesc simultan condiţiile:


a + b + c = 72 şi a + 3b − 2c = 50.
a) Să se arate că 3a + 5b = 194.
b) Să se determine numerele a , b, c ştiind că b are cea mai mică valoare posibilă.

***

2. Dublul unui număr este egal cu triplul altui număr, iar diferenţa lor este 16. Aflaţi cele două numere.

Prof. Valer Pop, G.M. nr. 6/2007

3. Fie n ∈ ¥ , astfel încât 13n + 8 dă restul 13 la împărţirea cu 80, iar 8n + 5 dă restul 5 la împărţirea cu
50. Determinaţi ultimele două cifre ale lui n.

Prof. Nicolae Stănică, Brăila

4. Un număr natural se numeşte „bipătrat” dacă suma şi produsul cifrelor sale sunt pătrate perfecte nenule.
a) Determinaţi toate bipătratele de două cifre.
b) Suma tuturor bipătratelor de trei cifre se împarte exact la 37? Justificaţi răspunsul.

Prof. Valentin Damian,


Brăila

CLASA A VI A

5. Determinaţi cel mai mic număr natural care împărţit, pe rând, la numerele 24; 40 şi 56 dă, de
fiecare dată, restul 5 şi câtul diferit de 0.

***
2007
2. Determinaţi ultimele 3 zecimale nenule ale numărului raţional .
22008
Prof. Irinel Pancu,
Brăila

3. Fie B ∈ ( AC ) şi D, E două puncte de o parte şi de alta a drepte AC , astfel încât triunghiurile


ABD şi BCE să fie echilaterale. Considerăm S ∈ ( AB ) , T ∈ ( BC ) astfel încât

m ( S DSB ) = m ( S ETC ) = 90 şi DS ∩ EC = {P} , ET ∩ AD = {F } Dacă P, B, F sunt


°

coliniare şi D , B , E sunt coliniare, arătaţi că AD = BC .

Prof. Nicolae Stănică, Brăila


4. Să se afle măsura unghiului format de bisectoarele a două unghiuri adiacente, ştiind că raportul dintre
1
suplementul sumei lor şi suma suplementelor lor este .
4
Prof. Ştefan Iloaie, G.M. nr. 7/2006

CLASA A VII A

1. Demonstraţi că numărul 1⋅ 2⋅ 3⋅ ...⋅ 2007 + 2008 este iraţional.


***

 1 
2. Fie mulţimea S=  abc = x 2 + 3 x + 2, x∈ ¥  . Calculaţi suma elementelor mulţimii S .
 abc 

3. Într-un sistem de axe perpendiculare xOy , fie punctele A( − 4;2 ) şi B ( − 2;4 ) . Determinaţi
coordonatele punctului C , din sistemul de axe perpendiculare xOy , astfel încât punctul O( 0;0 ) să fie
centrul de greutate al triunghiului ABC .

***

m ( S BAC ) = 90 , M este mijlocul [ BC ] şi BD ⊥ AM ,


4. În triunghiul dreptunghic ABC , °

D ∈ ( AC ) . Demonstraţi că m( S ACB ) = 30° dacă şi numai dacă BD = 2⋅ MD.

Prof. Nicolae Stănică, Brăila

CLASA A VIII A

1. Fie a , b, c, d ∈ ( 0;∞ ) astfel încât ac = bd = 12 . Demonstraţi că: ( a + 3) ( b + 3) ( c + 4 ) ( d + 4 ) ≥ 482 .

***
2. Să se rezolve în mulţimea numerelor naturale ecuaţia:
2
( )(
y + y − 10 = x 2 + 3 x ⋅ x 2 + 3 x + 1 . )
Prof. Liliana Stoian, Brăila

3. Fie ABCD şi BCEF două paralelograme situate în plane diferite, iar punctul P mijlocul [ AB ] .
Demonstraţi că AE P( FPC ) .

***

4. În cubul ABCDA ' B ' C ' D ' , aria triunghiului DOB este 3 cm2, unde { O} = BC '∩ B ' C .
a) Calculaţi valoarea cosinusului unghiului determinat de dreptele DO şi A ' B .
b) Determinaţi distanţa de la M la ( AOC ) , unde M este mijlocul [ AB ] .

Prof. Nicolae Stănică, Brăila


CLASA A IX A

1. a) Demonstraţi că ( a+ b) n ≤ ( an + bn ) , ( ∀ ) n ∈ ¥ şi ( ∀ ) a, b ∈ ¡ + .
2
n− 1

b) Demonstraţi că ( a + b ) ( a n + bn ) ≤ 2 ( a 2n + b2 n ) ( ∀ ) n ∈ ¥ şi ( ∀ ) a , b ∈ ¡ + .
n n

Prof. Runceanu Emilian, Brăila

a 2 + 20082
2. Demonstraţi că ∈ ¡ \ ¤ ,( ∀ ) a ∈ ¢ .
3
Prof. Dan Negulescu,
Brăila

3. Dacă a , b, c sunt lungimile laturilor unui triunghi, demonstraţi că


2 2 2
9 a + 9b + 17 c − 14 ac − 14bc − 6ab > 0 .
Prof. Nicolae Stănică, Brăila

 x+ 7 x+ 1
4. Să se rezolve în ¡ ecuaţia   = 2 unde [ a] este partea întreagă a numărului a.
 2 
Prof. Vasile Tarciniu, G.M. nr. 6/2007

CLASA A X A

a x − a− b bx − a− b
1. Să se rezolve ecuaţia − = a − b , unde a ,b∈ ( 0;1) ,a ≠ b .
bx− 1 a x− 1
Prof. Marius Perianu, Slatina

2. Fie z ∈ £ cu modulul minim, care verifică relaţia z − i + z − 3 = 2 . Demonstraţi că, pentru acest
i− z
număr z , avem ∈¡ .
3− z
Prof. Carmen Botea,
Brăila

3. Să se determine n∈ ¥ pentru care log n ( n + 2 ) este număr raţional.

Prof. Dan Negulescu,


Brăila
sin A sin B sin C
4. Într-un triunghi ABC este verificată relaţia: + + = 3 . Determinaţi măsurile
sin B sin C sin A
unghiurilor triunghiului ABC .
Prof. Vasile Berghea, G.M. nr. 8/2007
CLASA A XI A

1. Să se calculeze determinantul:
sin 2 x 1 sin x + cos x
∆ = 1 sin 2 x sin x + cos x .
sin x + cos x sin x + cos x 2
G.M. nr.
8/2007

2. A∈ M2 ( ¡ ) se numeşte matrice nilpotentă dacă există k ∈ ¥ * astfel încât Ak = O2 .


 1 2
a) Să se arate că matricea  − 2 − 1 nu este nilpotentă, dar se scrie ca sumă finită de matrici
 
nilpotente distincte.
 a b
b) Să se determine matricile nilpotente de forma  c d  ∈ M 2 ( ¡ ) , dacă b ⋅ c ≤ 0 .
 
Prof. Gheorghe Alexe, Brăila
3. Fie A ∈ M n ( ¡ ) , n ∈ ¥ , n ≥ 2 cu det A ≠ 0 astfel încât A2 − A + I = O . Determinaţi n
n n

astfel încât det A ( 504


)
+ I n = 512 .
.
Prof. Roxandra Murea, Brăila

4. Fie şirul ( an ) n ≥ 1 de numere reale care verifică relaţia: ( an + 1 − an ) ⋅ ( an + 1 + an + 4 ) ≤ 0,


 1 n 
( ∀ ) n∈ ¥ * . Calculaţi lim  ∑ ak  .
n→ ∞  n2 k = 1 

Prof. Viorel Botea, Brăila

CLASA A XII A

  a b 2 2 
1. Fie G=   a + b ≠ 0 ⊂ M 2 ( ¡ ) Să se arate că:
  − b a 
a) ( G, ⋅ ) este grup abelian.
b) grupurile ( ¡ *,⋅ ) şi ( G, ⋅ ) nu sunt izomorfe.

Prof. Dan Negulescu, Brăila

sin x − sin x − 1  0; π 
2

2. Să se calculeze ∫ dx, x ∈   .
e + cos x
sin x
 2
Prof. Marius Perianu,
Brăila
π
( ∀ ) x ∈  0;  : cos x > 1 −
1
3. a) Arătaţi că .
 2 x
2

 π  cos x
b) Se consideră funcţia f :  0;  → ¡ , f ( x) = şi F o primitivă a sa. Să se arate că
 2 x

F
 e  + ln 3 < F  e  + 13 , unde e
   este numărul lui Euler.
 3  2 8
***
4. Fie legea de compoziţie pe ¡ , x ° y = xy − ax − 2ay + 4 a, a ∈ ¡ . Aflaţi a astfel încât ( 1; + ∞ )
să fie parte stabilă a lui ¡ în raport cu legea "° ".

Prof. Sergiu Romaşcu, Vaslui

PROBLEME PROPUSE

CLASA a IV-a

1. Să se afle numerele cuprinse între 30 şi 60, care împărţite la 4 să dea restul 3.


2. Ana a câştigat la „ Robingo ” 100.000 lei din care şi-a cumpărat o pereche de blugi, o pereche de ghete,
şi i-a cumpărat şi bunicii un cadou. Ghetele au fost de 2 ori mai scumpi decât blugii, iar un sfert din
diferenţa de preţ dintre ghete şi blugi înseamnă cu 2.500 lei mai puţin decât preţul cadoului. Cât a costat
cadoul bunicii ?
3. Să se calculeze „a” din egalitatea : { 3  ( 8a - 72 ) : 8 + 234 - 703} : 2 = 1.
4. Şapte băieţi din clasa a cincea au numere de telefon asemănătoare, asemănarea constând în faptul că
cinci cifre sunt identice, iar suma lor reprezintă cealaltă cifră. Să se arate că cel puţin doi băieţi sunt fraţi.
5. Diferenţa a două numere naturale este cu 240 mai mică decât suma lor şi de 4 ori mai mică decât aceeaşi
sumă. Aflaţi numerele.
6. Suma a trei numere este 100. Să se afle numerele ştiind că primul număr este de 3 ori mai mic decât al
doilea şi al treilea împreună, iar diferenţa dintre al treilea şi al doilea este jumătate din al doilea.
7. Într-o şcoală sunt 25 de clase. În fiecare clasă sunt cel puţin 30 de elevi şi cel mult 35 de elevi. Arătaţi că
există cel puţin 5 clase cu acelaşi număr de elevi.
8. Trei ciobani au 1997 oi. Al doilea are cu 8 oi mai mult decât primul, iar al treilea dublul numărului de oi
al primului cioban micşorat cu 15. Câte oi are fiecare?
9. Perimetrul unui dreptunghi este 1km. Aflaţi dimensiunile dacă un sfert din lungime este cu 34m mai
mare decât a treia parte din lăţime.
10. Fie 10 bile colorate cu câte o culoare. Arătaţi că există 4 bile colorate la fel sau 4 bile de culori diferite.
11. Să se afle x din egalitatea:  420 - 2 ⋅ 45 : ( 37 - 7 x )  : ( 4 ⋅ 4 -1) = 25.
12. Într-o cutie sunt 25 bile albe, 25 bile negre şi 25 bile verzi. Care este numărul cel mai mic de bile care
trebuie scoase, fără a privi bilele, pentru a fi siguri că am extras 13 bile de aceeaşi culoare?

CLASA a V-a

2008n + 1
1. Demonstraţi că fracţia este ireductibilă pentru orice n ∈ ¥ .
2007 n + 1

Prof. Nicolae Ivăşchescu, Craiova

2. Dacă mărim un număr natural cu 2008 se poate obţine un număr natural cu aceleaşi cifre?
Prof. Gheorghe Stoica, Petroşani

3. Se consideră numărul
n = 13 + 12 ⋅ 13 + 12 ⋅ 132 + ... + 12 ⋅ 132007
6. Demonstraţi că n = 132008
7. Scrieţi numărul n ca o sumă de trei pătrate perfecte nenule.

Prof.Boicescu Nazeli, Brăila


4. Să se găsească numerele prime de forma 9a9.

Prof. Nicolae Ivăşchescu, Craiova

5. Să se arate că dacă două numere naturale mai mari sau egale cu 2 au câte un număr par de divizori
naturali şi au produsul un număr natural pătrat perfect, atunci ele nu sunt prime între ele.

Prof. Gheorghe Stoica, Petroşani

6. Să se scrie numărul 20062005 ca o sumă de cinci pătrate nenule.

Prof. Nicolae Ivăşchescu, Craiova

7. Se dau mulţimile A = { 2 x + 2, 4 x − 6,5 x − 2, x + 2} şi B = { 3 x − 3, 2 x + 4, 4 x + 3, 2 x − 3}


. Să se afle x ∈ ¥ astfel încât cele două mulţimi să fie egale.

Prof. Nicolae Ivăşchescu, Craiova

8. Să se rezolve în ¥ × ¥ ecuaţia xy + 2 x + y = 2005.


Prof. Nicolae Ivăşchescu, Craiova
CLASA a VI-a

1. Determinaţi cifrele x, y, z diferite de 9, cu x < y < z astfel încât numărul


A = x, y ( z ) + y , z ( x ) + z , x ( y )
să fie natural.

Prof. Viorel Cornea şi Dan Ştefan Marinescu, Hunedoara

2. Demonstraţi că nu există b ∈ ¤ astfel încât ab(a + b) = 1.

Prof. Ionuţ Ivănescu, Craiova

3. Să se arate că există o infinitate de numere naturale nenule x, y, z , distincte două câte două astfel încât

x 2 + 2 y 2 = 3z 2 .

4. Fie a, b, c ∈ ¥ astfel încât 3b − 9c − 11a = 0 . Să se arate că a ( b + 8c ) ⋮33 .


Prof. Nicolae Ivăşchescu, Craiova

5. Determinaţi mulţimile X ⊂ ¢ ∗ , X =/ Φ cu proprietatea că oricare ar fi

1  x y
x , y∈ X ⇒ ⋅ +  ∈ X .
2  y x
Prof. Gheorghe Stoica, Petroşani
6. Dacă x, y, z , numere naturale cu proprietatea 2 x − 3 y − 10 z = 0, demonstraţi că y ( x + z ) este un
număr divizibil cu 6.

Prof.Boicescu Nazeli, Brăila

n ∈ ¥ , numărul
7. Să se demonstreze că oricare ar fi
( 2
)( 2
)( )( )
A = n + 2005 n + 2006 n 2 + 2007 n 2 + 2008 + 1 este pătrat perfect.

Prof. Rodica şi Dumitru Bălan, Galaţi

CLASA a VII-a

1. Aflaţi numerele naturale a, b, c diferite de 0 şi 1, care verifică relaţia:

ab + bc + ac + 6 = 3( a + b + c )

Prof. Şerban George, Brăila


a b   a b
2 2

2. Să se demonstreze că 3  2 + 2  − 5  +  + 4 ≥ 0 , ∀ a, b ∈ (0, ∞ ) .
b a   b a
Prof. Daniela Covaci, Brăila

3. Se dă V ABC cu M , N mijloacele laturilor [ AB] si [ BC ] . Să se ducă folosind o


riglă negradată paralela prin B la AT unde T este un punct oarecare de pe
( MC ) .

Prof. Nicolae Ivăşchescu, Craiova

x, y, z şi a număr real strict pozitiv.


4. Să se arate că oricare ar fi numerele reale nenule
x+ y− a y+ z− a x+ z− a 1
+ 2 ≤
x + y + xy y + z + xy x + z + xz a
2 2 2 2 2

Prof.dr.D.S.Marinescu, Prof.I.Serdean, Hunedoara

5. Fie a , b ∈ ¡ ∗ , p , q , r ∈ ¡ , r ≥ q astfel încât a p − q + b p − q ≤ r − q şi


+
a p + q + b p + q ≤ r + q . Să se arate că: a + b ≤ r .
p p

Prof. Gheorghe Stoica, Petroşani


6. În triunghiul dreptunghic ABC cu m(R A) = 90o , h lungimea înălţimii din A iar S = aria ( ABC )
are loc inegalitatea:
 a b c  2 2  1  
+ +  ⋅( a b + b c + c a )  + 2 a− b− c  ≥ 40 S 2
2 2 2 2

 bc ac ab   2  

Prof.dr.D.S.Marinescu, Prof.I.Şerdean, Hunedoara

7. Să se determine numerele naturale n ştiind că 5 n + 8 n + 12 n + 14 n se divide cu 13.

Prof. Rodica şi Dumitru Bălan, Galaţi

8. Pe ipotenuza [ BC ] a triunghiului dreptunghic ABC se consideră punctul D . Atunci are loc


inegalitatea: min ( BD, DC ) ≤ AD ≤ max ( BD, DC ) .

Prof. Gheorghe Stoica, Petroşani

9. Arătaţi că există 2007 numere naturale distincte a1 , a2 ,..., a2007 astfel încât
a1 a2 a3 a
+ + + .... + 2006 ∈ ¥
a2 a3 a4 a2007
.

Prof. Liviu Smarandache, Craiova

10. Fie a, b, c lungimile laturilor unui triunghi. Arătaţi că:


(a + b3 ) + 3abc 2 ( 2a 2 + ab + 2b 2 ) > c 6
3 2

Prof. Lucian Tuţescu, Craiova, Prof. Dumitru Săvulescu Bucureşti

CLASA a VIII a

1. Aflaţi toate numerele abcd ştiind că: (a 2 + b 2 )cd + (c 2 + d 2 )ab = 20 şi a, b, c, d sunt


nenule.

Prof. Şerban George, Brăila

3 7 11 2007 1
2. Demonstraţi că ⋅ ⋅ ⋅⋅ < .
5 9 13 2009 669

Prof Boicescu Nazeli, Brăila

3. Să se determine { 2
} {
2
m ∈ ¡ pentru care x ∈ ¢ / x − 5 x + m = 0 ∩ x ∈ ¢ / x − mx + 3 = 0 ≠ ∅ . }
Prof. Lucian Dragomir, Oţelu-Roşu, Caraş-Severin
(x − y ) + ( x − y 2 ) = ( xy − 1)
2 2 2
4. Arătaţi că ecuaţia 2
are în ¤ × ¤ o infinitate de soluţii.

Prof. Gheorghe Stoica, Petroşani

5. Dacă a, b, c, d ∈ [− 2, ∞ ) şi a + b + c + d = 16, atunci


a 3 + b 3 + c 3 + d 3 ≥ 40.
Prof.dr.D.S.Marinescu, Prof.I.Şerdean, Hunedoara
6. Să se rezolve în R următoarele ecuaţii, unde prin [a ] s-a notat partea întreagă a numărului real a:

 x − 1  2 x + 1   3x − 5   4 x + 5  5x − 8
 3  +  6  −  9  +  12  = 9 .

Prof. Rodica şi Dumitru Bălan, Galaţi

¢ × ¢ ecuaţia: x 2 = y ( y + 1)( y + 2)( y + 3) + 4 .


7. a) Rezolvaţi în
b) Determinaţi m, n ∈ ¥ pentru care m 2 = n3 − n + 2 .

Prof. Daniela Tilincă, Brăila

8. Dacă a, b, c sunt lungimile laturilor unui triunghi dreptunghic atunci


 a b c   ab ac bc 
 + +  ⋅ + +  ≥ 10.
 bc ac ab   c b a 

Prof.dr.D.S.Marinescu, Prof.I.Şerdean, Hunedoara

CLASA a IX a

1. Să se determine numerele întregi x pentru care (x 2


+ 2 x + 1) ⋅ ( x 2 + 4 x + 3) ≤ 2 x + 3 .

Prof. Lucian Dragomir, Oţelu-Roşu, Caraş-Severin

1 1 2 2 1 1
2. Să se rezolve ecuaţia + − − + + = 0.
x x + 4 x + 8 x + 12 x + 16 x + 20
Prof. Lucian Dragomir, Oţelu-Roşu, Caraş-Severin

3. Fie A = { ai ∈ N * | ai ≤ 100} astfel încât card ( A) = 51 . Demonstraţi că există cel puţin o


ecuaţie de gradul doi cu coeficienţi în mulţimea A ale cărei rădăcini sunt raţionale.

Prof. Daniel Nicoară, Hateg

 x 4 − y 4 = 15
4. Rezolvaţi sistemul de ecuaţii  3
 x y − xy = 6
3

Prof. Butaru Zizi-Iuliana, Dolj


5. Fie a, b, c lungimile laturilor unui triunghi. Arătaţi că :
a 3 + 3b 2c + c 3 b3 + 3c 2 a + a 3 c 3 + 3a 2b + b3
+ + > 3.
b3 + 3bc 2 c 3 + 3ca 2 a 3 + 3ab 2

Prof. Dumitru Săvulescu, Bucureşti


n
6. Fie x1 , x2 ,..., xn ∈ ¡ + astfel încât ∑i= 1
xi = 1 iar A1 , A2  , An mulţimi oarecare. Arătaţi că are loc:
n
card ( A1 ∩ A2 ∩ ... ∩ An ) ≤ ∑i= 1
xi ⋅ card Ai unde card A reprezintă numărul elementelor mulţimi A.

Prof. Ionuţ Ivănescu, Craiova

x y z
7. Fie x, y, z > 0 cu proprietatea că x + y + z = 3 . Să se arate că: + + ≥ 1.
y+ 2 z+ 2 x+ 2

Prof. Ion Nedelcu, Ploieşti


8. Să se rezolve în mulţimea numerelor naturale ecuaţia: x 3
− y = 3 xy + 17 .
3

Prof. Bran Nicoleta, Prof. Smarandache Liviu, Craiova

1 1 1 1 1
9. Fie an = 1+ + ... + şi bn = + + ... + , n ≥ 1 . Să se arate că oricare ar fi numărul
3 2n − 1 2 4 2n
n ≥ 1 { an } ≠ { bn } . (Prin { α } am notat partea fracţionară a numărului real α ).

Prof. Gheorghe Stoica, Petroşani

CLASA a X a

1. Rezolvaţi ecuaţia x5 x + 5
x x = 56 .
Prof. Valeria Ionescu, Dolj

2. Fie f : ( 0, ∞ ) → ¡ , f ( x ) = log9 ( 1 + x 2 + x3 ) − 2 log 4 x şi A = { x | f ( x) = 0} . Să se


determine elementele mulţimii A.
Prof. Carmen Folea, Brăila

3. Să se arate că într-un triunghi, avem:


ra rb rc 108r 3
+ + ≥ (Notaţiile sunt cele uzuale).
a b c abc
Prof. Gheorghe Stoica, Petroşani

4. Fie α ∈ ¡ şi z1 , z2 ∈ £ . Să se determine z1 şi z2 ştiind că z1 = z2 = 1 şi z1 + z2 = α z1 z2 .

Prof. Ion Nedelcu, Ploieşti


1
5. Fie a, b, c > 0 cu proprietatea că a 2 + b 2 + c 2 =
. Să se arate că :
3
1 1 1
a+ b+ c+ + + ≥ 28 .
ab ac bc
Prof. Ion Nedelcu, Ploieşti
7. Aflaţi numerele naturale nenule a căror diferenţă este egală cu puterea n a câtului lor, n ∈ ¥ ∗ .

Prof. Butaru Lili Iuliana, Dolj

2 3− 3 2 3 + 3 , să se calculeze suma
8. Ştiind că a= + i
2 2
2006
S= ∑  (a 2 + 1)k + a 2 + 2  .
 
k=1

Prof. Rodica şi Dumitru Bălan, Galaţi

9. Să se arate că într-un triunghi ascuţitunghic, avem:


a2 − ( b − c ) b2 − ( c − a ) c2 − ( a − b )
2 2 2

+ + ≥ 3 , notaţiile fiind cele uzuale.


a 2 + b2 − c 2 b2 + c 2 − a 2 c 2 + a 2 − b2
Prof. Gheorghe Stoica, Petroşani

CLASA a XI a

 1 1− x
şi şirul ( an ) n∈ ¥ , an = f ( 2 ) , ∀ n ∈ ¥ . Să se
( 2n)
1. Fie f : ¡ \  −  → ¡ , f ( x) =
 2  2x + 1
calculeze lim an .
n→ ∞
Prof. Carmen Folea, Brăila

2. Fie n ∈¥ ∗ şi A, B ∈ M n (£ ) cu proprietăţile:
6. ( A + B) 2 = A2 + B 2 ;
7. ( A + B ) 4 = A4 + B 4 ;
Să se arate că ( AB) = On ;
2

Prof. Ion Nedelcu, Ploieşti

 1 + xn + 1 
3. Fie ( xn ) n≥ 1 un şir de numere pozitive. Atunci l = lim n  − 1 ≥ 1 .
n→ ∞
 xn 

Prof. Carmen Folea, Brăila


4. Fie A, B ∈ M n (¡ ), n ∈ ¥ , n ≥ 2, astfel încât A2 = 2 A − I n şi B 2 = − B − 2 I n . Dacă
f , g : ¡ * → ¡ , f ( x) = det( xA + I n ) , g ( x) = det( xB + I n ) , calculaţi
f ( x) + g ( x ) − 1 + (− 1) n + 1
lim 1+ ( − 1) n .
x→ 0 +1
x 2

Prof. Daniel Nicoară, Haţeg

 n1+ 1 + n1+ 1 + ...+ 31n 


5. Să se calculeze lim n  e − 3
n→ ∞
 
Prof. Lucian Tuţescu, Craiova, Prof. Ion Nedelcu, Ploieşti

CLASA a XII a

1. Fie f ( x ) = cos x; f : ¡ → ¡ şi M = { f : ¡ → ¡ | ϕ continuă şi f ° ϕ = f } respectiv


G= { ϕ : ¡ → ¡ | ϕ continuă şi bijectivă f ° ϕ = f }
21. Demonstraţi că M este monoid şi G grup împreună cu operaţia de compunere a funcţiilor
22. Fie ϕ ∈ M . Demonstraţi că ϕ este derivabilă în orice punct x ≠ kπ , k ∈ ¢ .
23. Daţi un exemplu de funcţie ϕ ∈ M care să nu fie derivabilă în nici un punct de forma
x = kπ , k ∈ ¢ .

Prof. Radu Vasile, Brăila

1 − sin x  π 
2. Să se calculeze primitiva următoare: ∫ 2sin x + 3(1 + cos x)e dx , x ∈  0, 2  .
x

Prof. Moanţă Cristian, Craiova

dx
3. Calculaţi integrala: I= ∫x ( n − 1) k + 1
; k ∈ ¥ ∗ , n ∈ ¥ , x ∈ ( 0, ∞ ) .
1+ x 2k

Prof. Mărculescu Mariana, Craiova

4. Să se determine f : ¡ → ¡ , ştiind că dacă F : ¡ → ¡ este o primitivă a sa cu proprietatea că


F ( 0 ) = 0 , atunci 12¡ ⋅ F este o primitivă a funcţiei 1¡ ⋅ f .

Prof. Ion Nedelcu, Ploieşti

5. Calculaţi limita şirului:

 1 1   1 1 
an = 16n   +  +  +  + ...
  16n + 2 16n 2 + ( 4n + 1)   16n + 6 16n 2 + ( 4n + 5 ) 
2 2 2 2 2 2

 1 1 
⋅⋅⋅+  + .
 16n 2 + ( 4n − 2 ) 2 16n 2 + ( 8n − 3) 2  
 
Prof. Daniel Nicoară, Haţeg

6. Să se rezolve în S5 ecuaţia :
1 2 3 4 5
X 11 =  .
 5 4 3 1 2
Prof. Smarandache Liviu, Craiova

Rubrica rezolvitorilor

1. Judeţul BRĂILA:

Liceul Teoretic „Nicolae Iorga” Brăila:

clasa a XI a, Bogzeanu Loredana, clasa a XII a, Plopeanu Aura, Sidorof Alexandra, Enache Angelo, clasa
a XII , Ion Anca, Bratu Laura; prof. Nistor Lidia

Colegiul Naţional „Gheorghe Munteanu Murgoci” Brăila:

clasa a XI a, Boicescu Teodora , clasa a XII a Toader Bogdan, Bunea Rodica, prof. Negulescu Dan

Colegiul Naţional „Nicolae Bălcescu” Brăila:

clasa a IX a, Bălan Cristina Alexandra, Bardaş Alexandru, Boboc Iuliana, Caracaţeanu Adrian, Chiru
Andreea, Chiscăneanu Diana , Dănăilă Georgiana, Danielescu Marina Cella, Doxan Radu Mihnea,
Dumitrache Cătălin, Epifanov Denis Mirela, Gurzu Mădălina Doina, Ichim Andrei , Istrate Andrei
Valeriu, Mareş Bogdan Mihail, Muntean Ionuţ Alexandru, Neacşu Adi Mădălin, Nedelcu Cristian Vlad,
Pascu Cristina Teodora, Perianu Armand Valeriu, Pintelie Andreea Mihaela, Popa Mircea Doru, Radu
Adelina, Stancu Alexandru Daniel, Toader Horia,Tudorache Andreea Cristina, Turcu Alexandru, Voicu
Bogdan, Zodilă Emil; prof.Botea Carmen.
clasa a X a, Damian Oana, Trişcaş Oana, Bercaru Andrei clasa a XI a, Gheorghiţă Eugen, Matei Răzvan,
Vioreanu Roxana, Ghioca Iulia, prof. Botea Viorel,

2. Judeţul DOLJ:

Colegiul Naţional „Fraţii Buzeşti”, Craiova

clasa a VI a, Dănciulescu Diana Elena, Kevorchian Andreea, Puchin Iulia, Stroe Alexandra Ştefania; prof.
Ivăşchescu Nicolae, clasa a XII a, Tuţescu Anca; prof. Tuţescu Lucian.
SUMAR:

ARTICOLE MATEMATICE

1. FRACŢII CU SIMPLIFICĂRI FRAUDULOASE de Roxana Murea..........................1


2. INEGALITĂŢI ÎNTRE ELEMENTELE UNUI TRIUNGHI de Neculai Carnaru, Iugulescu
Nicolae, Turcu Narcis Gabriel……………………….................................................4
3. INEGALITATEA LUI I.E.BURKELL de Gheorghe Alexe..............................................8
4. UTILIZAREA UNOR IDENTITĂŢI COMBINATORICE ÎN REZOLVAREA UNOR
IDENTITĂŢI TRIGONOMETRICE de Ani Drăghici şi Mădălina Călinescu….12
5. ASUPRA UNEI PROBLEME DIN GAZETA MATEMATICĂ de Mihaela Giurcă...14
6. PROBLEME DATE LA OLIMPIADA JUDEŢEANĂ CU SOLUŢII INGENIOASE
de Dan Negulescu şi Radu Vasile……………………………................................................16
7. TEOREMA DE EXISTENŢĂ A PRIMITIVELOR UNEI FUNCŢII CONTINUE de
Mădălina Teodorescu...............................................................................................................18
8. PROBLEME DE BACALAUREAT CU…PROBLEME de Botea Viorel.....................23

CONCURSURI INTERJUDEŢENE

1. CONCURSUL INTERJUDEŢEAN DE MATEMATICĂ “PETRU MOROŞAN -


….TRIDENT ” EDIŢIA a V a 2007……..............................................................................27
2. CONCURSUL INTERJUDEŢEAN DE MATEMATICĂ “VICTOR VÂLCOVICI” …
EDIŢIA a XVI a 2008………..………………………………………………….………..44

PROBLEME REZOLVATE şi PROPUSE

1. PROBLEME REZOLVATE.............................................................................................55
2. OLIMPIADA DE MATEMATICĂ ETAPA LOCALĂ, 26.01.2008…………...…...…71
3. PROBLEME PROPUSE …………………………………………………..……………76
4. RUBRICA REZOLVITORILOR.....................................................................................84

REVISTA DE MATEMATICĂ DIN BRĂILA “TRIDENT” este editată semestrial cu


sprijinul S.S.M.R. Filiala Brăila şi al sponsorilor METAL BAND S.R.L., S.C.CUNAX S.R.L.. Se
adresează tuturor elevilor şi profesorilor interesaţi de concursuri, olimpiade şi rezultate
deosebite la matematică. Materialele spre publicare, precum şi soluţiile problemelor, se vor
trimite la adresa bcviorel@yahoo.com sau pe adresa revistei: str. Alexandru I. Cuza, nr.182,
Brăila, tel. / fax 0239 615333, 0722697363.
Soluţiile problemelor se vor scrie pe foi diferite şi vor fi redactate îngrijit. Expeditorii vor
menţiona :numele şi prenumele, localitatea, şcoala, clasa şi profesorul de la clasă (pentru elevi),
telefon .
Propunătorii de articole şi probleme, îşi asumă responsabilitatea pentru originalitatea
materialelor.
TEHNOREDACTARE:
Bobocea Evelyn, REDACŢIA REVISTEI, Clasa a XI a C de la C.N.N.B.BRĂILA

TIPARUL: S.C. OFFSET GRAFIC SERV S.R.L. BRĂILA


All rights reserved ©

S-ar putea să vă placă și